SlideShare una empresa de Scribd logo
1 de 123
Descargar para leer sin conexión
Guía de

ejercicios
F Í S I C A M E C Á N I C A
Presentación
Este guía de trabajo tiene como objetivo de ayudar a usted
en el conocimiento de Mecánica, en el curso de Física Mecánica
Esta guía no pretende entregar todas las herramientas para
el ramo, pero si una ruta que lo ayude a entender mejor y más
rápido los requerimientos del ramo.
Quisiera recomendar algunas acciones que pueden ayudar
en el rendimiento de este ramo:
Pasen en limpio la materia vista en clases. Siempre es bueno
tener ordenado la materia para poder entenderla.
Rehagan los cálculos de la clase y la ayudantía.
Pregunte sus dudas en la clase. No es el único que tiene
dudas.
Realizar un ejercicio a diario. Si tiene dificultad en resolver
este ejercicio, consulte al profesor o profesor ayudante lo
antes posible.
Utilice los libros de la biblioteca.
Existen enlaces de interés para el curso que pueden
ayudar al desarrollo de este
● Física con ordenador http://www.sc.ehu.es/sbweb/fisica/
● Cursos del MIT
● http://open.yale.edu/courses/ curso de Yale
Pagina 2
Física-Mecánica
Temario Ejercicios resueltos
Vectores
Cinemática
Dinámica
Profesor Miguel Bustamante S Temario
Energía
Choque: Momentun e
impulso
Dinámica de
sólidos
Estática
Pagina 4
Física-Mecánica
Indice
Vectores......................................................................................................................................................9
Problema 1.............................................................................................................................................9
Solución............................................................................................................................................9
Pregunta 2............................................................................................................................................10
Solución:.........................................................................................................................................10
Pregunta 3............................................................................................................................................11
Solución:.........................................................................................................................................12
Cinemática................................................................................................................................................15
Pregunta 1............................................................................................................................................15
Solución..........................................................................................................................................15
Pregunta 2............................................................................................................................................17
Solución..........................................................................................................................................17
Problema 3...........................................................................................................................................19
Solución..........................................................................................................................................19
Problema 4...........................................................................................................................................20
Solución..........................................................................................................................................20
Problema 5...........................................................................................................................................22
Solución..........................................................................................................................................22
Problema 6...........................................................................................................................................24
Solución..........................................................................................................................................24
Problema 7...........................................................................................................................................25
Solución..........................................................................................................................................25
Problema 8...........................................................................................................................................27
Solución:.........................................................................................................................................27
Problema 9...........................................................................................................................................28
Solución..........................................................................................................................................28
Problema 10.........................................................................................................................................29
Solución..........................................................................................................................................29
Problema 11.........................................................................................................................................30
Solución...............................................................................................................................................30
Problema 12.........................................................................................................................................31
Solución...............................................................................................................................................31
Pregunta 13...............................................................................................................................................33
Solución..........................................................................................................................................33
Pregunta 14...............................................................................................................................................34
Solución..........................................................................................................................................34
Pregunta 15..........................................................................................................................................36
Solución..........................................................................................................................................36
Problema 16.........................................................................................................................................38
Solución..........................................................................................................................................38
Problema 17.........................................................................................................................................39
Solución..........................................................................................................................................39
Problema 18.........................................................................................................................................41
Solución..........................................................................................................................................41
Problema 19.........................................................................................................................................42
Solución..........................................................................................................................................42
Dinámica..................................................................................................................................................46
Problema 1...........................................................................................................................................46
Solución:.........................................................................................................................................46
Profesor Miguel Bustamante S Temario
Pregunta 2............................................................................................................................................48
Solución..........................................................................................................................................48
Pregunta 3............................................................................................................................................51
Solución..........................................................................................................................................51
Problema 4...........................................................................................................................................55
Solución..........................................................................................................................................55
Problema 5...........................................................................................................................................56
Solución..........................................................................................................................................56
Problema 6...........................................................................................................................................59
Solución..........................................................................................................................................59
Problema 7...........................................................................................................................................61
Solución..........................................................................................................................................61
Problema 8...........................................................................................................................................62
Solución..........................................................................................................................................62
Problema 9...........................................................................................................................................63
Solución..........................................................................................................................................63
Problema 10.........................................................................................................................................64
Solución..........................................................................................................................................64
Problema 11.........................................................................................................................................66
Solución..........................................................................................................................................66
Problema 12.........................................................................................................................................67
Solución.....................................................................................................................................67
Problema 13.........................................................................................................................................68
Solución..........................................................................................................................................68
Problema 14.............................................................................................................................................70
Solución..........................................................................................................................................70
Pregunta 15..........................................................................................................................................73
Solución..........................................................................................................................................73
Problema 16.........................................................................................................................................75
Solución..........................................................................................................................................75
Problema 17.........................................................................................................................................77
Solución..........................................................................................................................................77
Problema 18.........................................................................................................................................79
Solución..........................................................................................................................................79
Energía.....................................................................................................................................................83
Problema 1...........................................................................................................................................83
Solución:.........................................................................................................................................83
Problema 2...........................................................................................................................................84
Solución..........................................................................................................................................84
Problema 3...........................................................................................................................................87
Solución..........................................................................................................................................87
Problema 4...........................................................................................................................................88
Problema 5...........................................................................................................................................89
Problema 6...........................................................................................................................................91
Solución..........................................................................................................................................91
Choque.....................................................................................................................................................94
Problema 1...........................................................................................................................................94
Solución..........................................................................................................................................94
Problema 2...........................................................................................................................................96
Solución..........................................................................................................................................96
Problema 3...........................................................................................................................................98
Solución..........................................................................................................................................98
Problema 4...........................................................................................................................................99
Solución..........................................................................................................................................99
Pagina 6
Física-Mecánica
Problema 5.........................................................................................................................................100
Solución........................................................................................................................................100
Problema 6.........................................................................................................................................101
Solución........................................................................................................................................101
Problema 7.........................................................................................................................................102
Solución........................................................................................................................................102
Problema 8.........................................................................................................................................103
Solución........................................................................................................................................103
Dinámica de Sólidos..............................................................................................................................105
Problema 1.............................................................................................................................................105
Solución:.......................................................................................................................................105
Pregunta 2..........................................................................................................................................107
Solución........................................................................................................................................107
Problema 3.........................................................................................................................................109
Solución........................................................................................................................................109
Problema 4.........................................................................................................................................110
Solución........................................................................................................................................110
Problema 5.........................................................................................................................................112
Solución........................................................................................................................................112
Problema 7.........................................................................................................................................114
Solución........................................................................................................................................114
Estática de Sólidos.................................................................................................................................117
Pregunta 1..........................................................................................................................................117
Solución........................................................................................................................................117
Pregunta 2..........................................................................................................................................118
Solución........................................................................................................................................118
Problema 3.........................................................................................................................................119
Solución........................................................................................................................................119
Profesor Miguel Bustamante S Temario
Vectores
Problema 1
Se tiene una barra de longitud L que se puede mover libremente en los ejes,
como se muestra en la figura.
La posición del punto A está descrito por la ecuación: 
At=
L
2

L
4
sinwt
i
Encuentre la ecuación que describe el ángulo (t)
Grafique el ángulo (t) en función del tiempo para los valores de L=1 y w=0.2
pata t=0 y t=10.
Solución
Por definición, el coseno del ángulo es: Cos(f
)=L/2+L/4Sin(wt))/L=1/2+1/4*sin(wt).
Por tanto f =Arccos(1/2+1/4sin(wt))
El gráfico es:
Pagina 8
A
B
L

Física-Mecánica
Pregunta 2
Se tiene un cubo de lado “a”. Una hormiga se mueve desde el punto A al
punto B.
Calcular el ángulo que
forman el vector OA y el vector
OC.
Solución:
Primero debemos describir las
posiciones de los puntos A y C
según el sistema de referencia
que se observa.
El vector OA=ai+ak y
OC=aj+ak.
Aplicando el producto interno
se tiene que: OA*OC=a2
, y los
módulo de ||OA||=a2 =
||OC||
Por tanto la ecuación nos queda:
a2
= (a 2 )2
Cos( ).
Despejando el Coseno d el ángulo se obtiene que Cos( )=(a2
)/(2a2
)=1/2. Esto
implica que  =60°.
Profesor Miguel Bustamante S Temario
x
y
z
A c
O
Pregunta 3
Supongamos que tenemos el siguiente sistema planetario.
El planeta 1 esta a una distancia de 2 ua, y le toma en recorrer toda la
circunferencia (la órbita) 1.5 año; el satélite está a una distancia de 0.1 ua del
primer planeta y gira en torno al primer planeta tomando un tiempo de 0.2 año; el
planeta 2 está a 5 ua del sol y tiene un “periodo orbital” de 5.93 año.
La posición de cada cuerpo está descrita por la siguiente ecuación:

Rt=Rcoswt,sinwt , donde R esta distancia al centro del movimiento, w es la
frecuencia angular, que es igual w=2/T, donde T es el tiempo que demora en
recorrer toda la órbita: el periodo. Todos los planetas están en t=0 según la
figura.
Suponiendo que cada cuerpo está descrito por la ecuación encuentre:
a) El vector relativo que hay entre el planeta 1 y 2. Grafique el módulo de este
vector en función del tiempo de 0 a 7 años. ¿Cuando están a la máxima
distancia?, ¿cuando a la mínima?
b) Escriba el vector posición para cualquier momento del satélite en torno al Sol.
c) Encuentre una expresión del ángulo de forman los vectores posiciones de los
planetas en función del tiempo. Grafique este ángulo entre 0 y 7 años, y diga
cuando este tiene un valor de /2 y de .
Pagina 10
Sol
Planeta 1
Satélite
Planeta 2
Física-Mecánica
Solución:
Sabemos la expresión vectorial de la posición de los cuerpos en torno su
punto de rotación. La posición del planeta 1 y 2 esta dado por las expresión
⃗
R1(t )=R1 cos(w1 t)^
i + R1 sin(w1 t) ^
j y 
R2 t =R2 cosw2 t 
iR2 sinw2 t  
j . El vector relativo
entre los planetas es R2-R1, y obteniendo el módulo se puede llegar a la siguiente
expresión: ∥ 
R2t − 
R1 t∥=R2
2
R1
2
−2R1 R2 cosw2−w1t  :la representación gráfica
del módulo en función del tiempo es:
La máxima distancia corresponde cuando los planetas están formando un ángulo
de 180° ( radianes). Esto corresponde cuando cos((w2-w1)t)=-1, y es cuando
(w2-w1)t=n, donde n es impar. tn=n

w1−w2
, t1=1.04 año, t3=3.0126, etc.
La mínima distancia ocurre cuando cos((w2-w1)t)=1 , es de decir (w2-
w1)t=m, con m par; t0=0 años, t2=2.08 años, t4=4.16 años, etc.
Pregunta b
Las posiciones respecto del
centro de rotación está descrita por la
formula presentada en el enunciado.
En este caso queremos escribir el
vector posición de la Luna respecto del Sol.
Según la representación gráfica, el vector que buscamos es la suma de los
vectores, la suma vectorial, es decir :
Profesor Miguel Bustamante S Temario
P1
L

RSL = 
P1
L . En términos cartesianos, se puede escribir como:

RSL =R1cos w1 t RL coswL t
iR1 sinw1 t RL sinwl t 
j
Pregunta c
El producto interno nos da la información que buscamos: 
R1

R2=∥
R1∥∥
R2∥cos t 
Despejando el cos((t)) se llega a la siguiente expresión:
cos(t)=cos((w1-w2)t). Los planetas están perpendiculares entre sí, cuando (w1-
w2)t=k, donde k tiene los valores de la sucesión ((2n+1)/2), donde n es un
número entero incluyendo el cero. Por tanto
tn=
2n1
2

w2−w1
, t0=0.502 años,
t1=1.5063 años, t3=7.53 años.
El valor de  es cuando (w1-w2)t=, es decir t=1.0042 años
Pagina 12
Física-Mecánica
Cinemática
Profesor Miguel Bustamante S Temario
Cinemática
Pregunta 1
Un profesor de Física (Miguel Bustamante) pasa por al esquina de Manuel
Montt y 11 de Septiembre a una velocidad de 60 Km/H en dirección Sur. En el
instante que pasa frente a un alumno que tiene una moto de 1100 cc, el
alumno acelera la mota para alcanzar al profesor. La moto puede acelerar de 0
a 100 Km/h en 4.5 seg. Sin embargo, la moto tiene un problema; cuando llega a
los 80 Km/h, esta comienza a desacelerar a razón de -0.8 m/seg2
. El
movimiento del profesor no cambia y el origen del sistema está en la esquina
de Manuel Montt con 11 de Septiembre.
Sobre la información entregada, grafique la posición de los cuerpos.
• Calcule el momento del encuentro, si es que se encuentran.
• Si no se encuentran, calcule la velocidad mínima que debe tener el
profesor para ser alcanzado.
Solución
Primero debemos escribir la ecuación de cada móvil en todo instante de tiempo.
La ecuación del profesor xp(t)=vpt=16.67t m en todo instante para t>0 seg.
El alumno tiene una ecuación en un principio xa(t)=1/2at² a=(27.7-0)/4.5
m/s²=6.17 m/s², pero cambia para un t0 donde cambia de aceleración. La
nueva ecuación tiene la forma 1/2a'(t-t0)²+v0(t-t0)+x0, donde v0 es la
velocidad que tiene en t0 y X0 es la posición en t=t0. El momento donde
cambia de aceleración es cuando el estudiante tenía una velocidad de 80
km/h=22.2 m/s; es decir, va(t)=at=22.2 m/s. El tiempo es t=3.59 seg. Este es
nuestro t0. La posición es x0=1/2a*3.59²=39.9 m. Entonces la ecuación del
estudiante tiene la siguiente estructura:
xat =
{
1
2
6.17t² ,t0
1
2
−0.8t−3.59²22.2t−3.5939.9,t3.59}
Graficando la posición de ambos móviles nos da el siguiente gráfico.
Del gráfico se observa que se juntan . Esto lo vamos a probar igualando las
ecuaciones. Existen dos tramos de interés
Pagina 14
Física-Mecánica
• t<3.59.
Esto implica xa(t)=xp(t)
1/2*6.17*t²=16.67t, obteniéndose t=5.41 s >3.59 s (no es solución) No se
juntan.
• para t>3.59
-0.5*0.8*(t-3.59)²+22.2*(t-3.59)+39.9=16.67t, se obtiene que no hay solución
real. El alumno no alcanza al profesor.
Veamos la velocidad mínima del profesor. Supongamos que xp(t)=vpt.
Igualando a la ecuación del alumno se obtiene la siguiente ecuación que debe
resolverse:
-0.4*t²+(25.072-vp)t-45.2=0
Para que se tengan soluciones el determinante debe ser mayor o igual acero.
La condición mínima es que sea igual a cero. Esto implica que (25.072-vp)²-
72.32=0. Por tanto vp=16.56 m/s=59.6 Km/h, velocidad mínima.
Profesor Miguel Bustamante S Temario
0 5 10 15 20 25
0
50
100
150
200
250
300
350
400
Título principal
Serie1
Serie3
Pregunta 2
Un cuerpo se mueve a rapidez constante en el tramo AC.
El cuerpo parte de A hacia C, a rapidez constante. Entre A y B corresponde a la de un arco de
circunferencia de radio R; entre B y C es una línea recta de pendiente cero.
Con estos datos, calcule
• El radio r(t), en función del tiempo y el ángulo en función del tiempo.
• Calcule la velocidad angular en el punto B, como la aceleración en ese punto B en coordenadas
polares.
• Calcule la velocidad en C en coordenadas polares.
Solución
La distancia en el arco de circunferencia al origen del sistema es constante e igual a R siempre que esté
entre A y B. El tiempo que toma en recorrer hasta B es t=
π R
4v0
. Posterior a este punto, la distancia
crece, ya que se mueve en una recta (1). la distancia s=v0(t−π R/(4v0)) . Luego la distancia al
origen en el tramo BC es r (t)=√R2
+(v0(t−π R/(4v0)))2
.
Esto se puede resumir como
r (t)=
(
R 0<t<
π R
4v0
√R
2
+(v0(t−π R/(4v0)))
2 π R
4 v0
<t<
π R/ 4+ R
v0
)
Pagina 16
R
R A
B
C
x
y
Física-Mecánica
la velocidad en el tramo AB es sólo tangencial, ⃗
V =r (t)θ̇ ̂
θ . Esto implica que la magnitud de la
velocidad cumple v0=θ̇ R⇒θ̇=
v0
R
. la aceleración en el punto B es sólo radial, es decir
⃗
a=−R(θ̇)2
̂
r=
−v0
2
R
̂
r
Recordemos que la velocidad en coordenadas polares es ⃗
v(t )=ṙ ̂
r+r θ̇ ̂
θ . Conocemos r(t), por tanto
conocemos su derivada, pero no así, el ángulo. El ángulo los podemos expresar como
θ(t)=π
2
+ atan(v0(t−π R/(4 v0))
R )
Luego, la derivada es
θ̇=
1
1+(v0
R
(t−π R/(4v0)))
2
v0
R
(t−π R/(4 v0))
.
En el momento que pasa por C, el tiempo es tc=
π R/4+ R
v0
. Luego evaluando en ese tiempo
las expresiones y sus derivadas, se obtiene que r (tc)=√2 R , ṙ=
√2
2
v0
y θ̇=
1
2
v0
R
. la
velocidad en el punto C es
⃗
vc=√2
2
v0 ̂
r+ √2
2
v0
̂
θ
Profesor Miguel Bustamante S Temario
Figura 1: Trayectoria, posterior a B
R
s
R
s
vr
vt
Problema 3
Se tiene un cuerpo que inicialmente está a una altura H de un disco que
gira. El disco a una distancia r de su centro tiene un orificio por donde puede
caer el objeto.
El disco gira a una frecuencia angular constante w. Calcular
la relación entre la altura H y la frecuencia angular del disco
para que cruce por el orificio. Ambos están inicialmente en
t=0 en la posición que se muestra en la figura.
Se coloca un segundo disco, separado por una distancia d,
desfasado en /2 con respecto al anterior, girando a la misma
frecuencia w y con un orificio a la misma distancia del centro.
¿ Desde que alturas se puede dejar caer el objeto para que
cruce por ambos orificios.?
Solución
Para saber desde que alturas podemos dejar la partícula,
primero debemos darnos un sistema de referencia. El origen
del sistema de referencia va estar en el centro del disco
(azul). Según el sistema de referencia, la ecuación de
posición del cuerpo que cae es: 
Rt=r ,−
1
2
gt²H  El
tiempo que demora en caer debe ser igual a un número entero n por el periodo
del disco T. En el instante que está en el origen después de un tiempo nT se
cumple que
-1/2g(nT)2
+ H=0. Esto implica que existe n alturas posibles que crucen por el
orificio, esto es: Hn=
1
2
gnT 2
.
Para que cruce por el segundo orificio la ecuación en la posición de y debe
cumplir la siguiente igualdad: −d=
−1
2
gnT 
T
4

2
H , donde T/4 es el tiempo que
demora en recorrer la distancia d. Desarrollando la ecuación y imponiendo que
H=1/2 g(nT)². El valor de n que se tiene es: n=
8d
gT 
2
−
1
32
Esto implica que la
altura es:
H=
1
2
gT 
8d
gT 
2
−
1
32
²
Pagina 18
r
H
r
d
Física-Mecánica
Problema 4
Un profesor de física sale de su casa a las 6:00 horas de la mañana a caminar
junto con su perro “fotón”. El profesor camina a 5 [km/h] y el perro “fotón” lo hace a 20
[km/h]. El perro avanza en la misma dirección que el profesor. Cuando han
transcurrido 45 minutos (punto A), el perro se devuelve con la misma rapidez y avanza
hacia el profesor hasta alcanzarlo.
Sobre la base de la información anterior,
● Calcule dónde y cuándo se produce el primer encuentro del profesor y el perro.
● Grafique, sobre un mismo diagrama, la posición de ambos en función en el tiempo.
● Grafique, para ambos, la velocidad en función del tiempo.
Solución
Escribamos al ecuación de posición del profesor. El tiempo cero para nosotros es a
las 6:00 horas. La ecuación del profesor está dado por la expresión P t =5t Km . El
perro fotón es un poco mas complicado. La ecuación del perro es:
f t = 20 t km, t0,75 h
−20t −0,7515km
Grafiquemos las posiciones de los móviles.
Según el gráfico, existe un encuentro cerca de los 6 km, es decir en torno a las
1,25 horas de viaje.
El tiempo de encuentro es mayor que 0,75 horas, por lo tanto la posición del perro
fotón está dado por la ecuación -20(t-0,75)+15 km. Este se debe igualar a la posición
Profesor Miguel Bustamante S Temario
0 0,2 0,4 0,6 0,8 1 1,2 1,4 1,6
0
2
4
6
8
10
12
14
16
Gráifco de posición vs tiempo
Serie1
Serie3
Tiempo (h)
Posición
(km)
del profesor: -20(t-0,75)+15=5t. Despejando el t se obtiene el momento del
encuentro. Este corresponde a t=6/5 h=1,2 h. Evaluando en cualquier de las
ecuaciones de posición nos da la distancia al origen. En elñ caso del profesor P(1,2
h)=1,2*5 km= 6 km, como se aprecia en el gráfico.
El gráfico de velocidad de los móviles está dado por:
Pagina 20
0 0,2 0,4 0,6 0,8 1 1,2 1,4 1,6
-25
-20
-15
-10
-5
0
5
10
15
20
25
Velocidad v/s Tiempo
Serie1
Serie3
Tiempo (h)
Velñocidad
(km/h)
Física-Mecánica
Problema 5
En el borde de un acantilado hay un cañón que dispara proyectiles a 25 [m/s]. En
el instante t = 0 [s] hace un disparo a 53º de elevación; enseguida se inclina el
cañón y se hace un segundo disparo, esta vez a 37º. Si el segundo disparo se
efectúa T segundos después que el primero,
● ¿Cuál debe ser el valor de T para que los proyectiles choquen?
● ¿En qué punto se produce el choque?
● ¿Cuál es la velocidad a la que chocan los cuerpos?
Solución
Realicemos un esquema del problema
Escribamos el vector posición de cada disparo, suponiendo el sistema de
referencia en el cañón.
El vector posición del primer disparo es:

r1 t =25cos53t ,−
1
2
g t2
25sin53∗t  m
El vector posición del segundo disparo es:

r2 t =25cos37t −T  ,−
1
2
gt−T 2
25sin37t −T 
Si se encuentran, las componentes de los vectores deben ser iguales. Estop
implica que 25cos(53)t=25cos(37)(t-T) y
Profesor Miguel Bustamante S Temario
Lanzamiento 2
Lanzamiento 1
-0,5gt2
+25sin(53)t= -0,5g(t-T)2
+25sin(37)(t-T)
Resolviendo el sistema de ecuaciones de segundo orden anterior, se obtiene que
t=0 s o t=
2∗25∗sin53−cos53
g
s=4 s y T=1 s (g=10 m/s2
)
Evaluando en cualquier de los vectores, se obtiene que la posición de encuentro
es (19,6;14,76)m.
Derivando la ecuaciones de posición con respecto al tiempo, obtenemos los
vectores velocidad. en el caso del proyectil 1

v1 t=25cos53,−gt 25sin53t  y del proyectil 2

v2 t=25cos37,−gt−T25sin37t −T  . Evaluando en t=0,98 s, se
obtiene los siguientes vectores velocidades v1(0,98)=(15,0;-191) m/s y
v2(0,98)=(19,96; 3,73) m/s.
Pagina 22
Física-Mecánica
Problema 6
Una expedición de la Universidad Adolfo Ibañez quiere medir la altura de una
caverna. Para eso dispara un proyectil, de modo que el ángulo de elevación es de
90º. El sonido del choque sobre el techo se escucha un tiempo T después que se
ha disparado. El tiempo de subida es tv (hasta chocar con el techo) y ts es el
tiempo que demora el sonido en recorrer la distancia h. La velocidad del sonido
es Vs. Se sabe que T = 3tv.
Calcule en términos de h, g, Vs:
● La velocidad de salida del proyectil del cañón
● El tiempo tv
Solución
Realicemos un esquema del problema.
Si situamos el sistema de referencia en el suelo, la ecuación que describe el
proyectil, la ecuación es: (0,-g/2t2
+vt).
Si llamamos tv al tiempo que demora en chocar con cielo de la caverna, y ts el
tiempo que el sonido demora en recorrer, se tiene la siguiente relación
T=tv+ts=3tv; del cual se obtiene ts=2tv.
Si la altura de la caverna es h, se tiene que:
h=-g/2tv2
+vtv y h=Vsts. La altura es la misma, por tanto -g/2tv2
+vtv= Vsts =
2Vstv. Por tanto ts=2tv. Esto implica h/Vs=2tv y por tanto tv=1/2 h/Vs.
Además, de la igualdad de las alturas h=-g/2tv2
+vtv =Vsts=2Vstv, se tiene
que tv=2(v-Vs)/g=1/2h/Vs, del cual se obtiene que v=Vs+gh/(4Vs).
Profesor Miguel Bustamante S Temario
Pagina 24
Física-Mecánica
Problema 7
Se tiene el siguiente gráfico de velocidad en función del tiempo de dos móviles:
Se sabe que el móvil A está en t =5 [s] a 10 [m] del origen. El móvil B, en t = 0 [s]
está a en el origen (0 [m]). Los cuerpos continúan moviéndose indefinidamente.
En base a la información entregada:
a) Realice un gráfico de posición en función del tiempo para ambos móviles
b) Escriba las ecuaciones de movimiento de cada uno de los móviles
c) Calcule dónde se encuentran los móviles
Solución
Para poder realizar el gráfico debemos escribir las posiciones en función del
tiempo.
Para el móvil A, la ecuación sería:
xAt =
10t −510 m ,0t 5 s
−10t−510 m ,5t10 s
−40 m,t 10 s
Para el móvil B
XBt = 5t m, t10 s
−10t−1050 m ,t 10 s
Según el gráfico, nunca se encuentran
Profesor Miguel Bustamante S Temario
10
5
10[m/s]
[s]
Velocidad
Móvil A
Móvil B
5[m/s]
-10[m/s]
Pagina 26
0 2 4 6 8 10 12 14
-60
-40
-20
0
20
40
60
Posición de los móviles
Serie1
Serie3
Tiempo (s)
distancia
m
Física-Mecánica
Problema 8
Un móvil está en t= 0 s en la posición 
r =10,10m . En ese instante tiene una
velocidad de 
v=5,−2m/s hasta t=10 s. En ese instante cambia su velocidad a

v=−10,5m/s y se detiene en t=30 s.
Otro móvil, tiene una velocidad 
u=10,−5m/s y en t= 0 s está

r=0,15m .
Con la información entregada:
● Dibuje las trayectorias de los móviles
● ¿Se encuentran?
Solución:
Debemos escribir las posiciones de los móviles
La ecuación del primer cuerpo

Rt=
{
5,−2t10,10m, 10t0s
−10,5t−1060,−10m , 10t50s
−340,190m, t50 s }
La ecuación del segundo cuerpo

R2
t =10,−5t0,15m
La representación gráfica en el plano XY es:
Para contestar la pregunta, si se encuentran debemos resolver la ecuación

Rt= 
R2
t  .
Al igualar las ecuaciones, se debe cumplir tanto en las componentes del eje Y
como del eje X.
En este caso, no se cumple y por tanto no se encuentran.
Profesor Miguel Bustamante S Temario
Problema 9
La aceleración de una partícula es directamente proporcional al cuadrado del tiempo t. Cuando t= 2 s ,
v=-15 m/s y cuando t=10 s, v=0,36 m/s. Sabiendo que la partícula está dos veces mas lejos del origen
en t=2 s, que en t= 10 s Determine:
a) La posición de la partícula en t= 2 s y t=10 s
b) la distancia recorrida por la partícula de t=2 s a t = 10 s
Solución
Sabemos que at =kt2 . La velocidad es, por tanto
v t=∫
2
t
atdt−15 m/ s=k
t3
3
−
8k
3
−15 m/ s (1 punto)
Por construcción v(2)=15 m/s, pero se debe satisfacer v(10)=0,36 m/s. Esto implica que el valor de k
debe satisfacer k
10
3
3
−
8 k
3
−15 m/s=0,36 m/s . el valor de k es:0.042 (1 punto).
La posición de la partícula es
xt=∫
2
t
vtdtD=D0.0038333333333333t
4
−15.12266666666667t30.184 (2 puntos)
En x(2)=D, y en x(10)=D/2= 82.70933333 m +D. Así D=82.70933 m (1 punto)
Respuesta a), x(2)=82.709333 m y x(10)=165.418666 m (1 punto)
Respuesta b) La distancia recorrida es 82.716 m
Pagina 28
Física-Mecánica
Problema 10
Un autobús está estacionado a la orilla de un autopista cuando a su lado pasa un camión que
viaja con una velocidad de 45 mi/h. Dos minutos después, el auto bus arranca y acelera de manera
uniforme hasta que alcanza una velocidad de 60 mi/h, la cual mantiene. Sabiendo que 12 minutos
después de que el camión pasó junto al autobús, éste se encuentra 0,8 mi delante del camión, determine:
a) Cuándo y donde el autobús rebasa al camión
b) la aceleración uniforme del autobús
1 mi = 1,609 km
Solución
Debemos escribir la ecuaciones de posición de cada móvil.
Si situamos en donde está estacionado el auto bus, como buestro origen, la ecuación del camión viene
dado por la ecuación de itinerario:
ct=20,1t m ( 1 punto)
Veamos el bus
bt=
0, t120 s
a
t−1202
2
, 120tt1
26,81t−t1x1, t1t
donde t1 es el tiempo demora en llegar a una velocidad de 60 mi/h=26,81 m/s y x1, es la posición en
t=t1.
La posición del camión t=12 min=720 s es 14472 m, que esta 0,8 mi=1287,2 m, luego la posición del
bus en t=720 s es 13185 m.
Luego, se debe satisfacer las siguientes ecuaciones
26,81720−t1x1=13185 m
a
t1−120
2
2
=x1
y
at1−120=26,81m/ s
Resolviendo el sistema se obtiene:
a=0.1238 m/s2 (respuesta b) ,t1=336.4117 s y x1=2901.0| m.
Según , la posición y el tiempo el bus va a una velocidad uniforme. Así, para encontrarse, deben estar al
mismo tiempo en el mismo instante, el bus y el camión. b(t)=c(t). El tiempo de intersección es
t=911,802 s y evaluando en cualquier de las posiciones se obtiene donde se encientran
c(911,864)=18327.22 m
Profesor Miguel Bustamante S Temario
Problema 11
En la carrera de de Usain Bolt, medidos que la velocidad media en los últimos 100 m que era de 41
kh/h (11,38 m/s). Suponiendo que la velocidad viene dado por la expresión v t=V0 1−e
−t
 ,
donde V0=11,38 m/s.
Sabemos que x(19,30 s)=200, y que la velocidad final era 11,38 m/s, calcule
• El valor de . (3 puntos)
• La aceleración máxima.(3 puntos)
Ayuda: Para x(19,30)=200 m asuma que et=19,30
−t
=0.0
Solución
Si integramos la velocidad, y suponiendo que parte del origen, se obtiene que
Xt =V0t
1

e
−t
 X0
Si parte del origen, se tiene que X0=
−V0

=
−11,38

Pero en X(19,30)= 200 m, t utilizando el hecho que et=19,30
−t
=0.0 , se obtiene que
X(19,30)=11,38*19,30-11,38/=200, lo que implica que =0,58 s-1
. ( 3 puntos).
Al derivar la velocidad se obtiene la aceleración at=V0  e− t
. la aceleración es máxima en t=0 s.
Lo que implica que amax=V0=6,59 m/s2
( 3 puntos)
Pagina 30
Física-Mecánica
Problema 12
Una moto detenida en un semáforo, acelera con aceleración constante cuando le da la luz verde.
Cuando llega a los 80 km/h en t= 8 s, la moto falla, y comienza a desacelerar en forma constante de
modo que recorre 150 m hasta detenerse.
Sobre la base de la información entregada, calcule:
• La desaceleración de la moto.
• La aceleración de la moto.
• La distancia total recorrida.
• Realice un bosquejo de un gráfico de posición v/s tiempo, y de velocidad v/s tiempo.
Solución
Debemos tener claro, que este movimiento, consiste en l combinación de dos movimientos acelerados.
La aceleración se puede escribir como: a(t)=
{a0, t< 8s
a1, t> 8s} . La velocidad de 80 km/h, corresponde a
una velocidad de 22.22 m/s. Luego la aceleración a0=22.22/8 m/s2= 2.78 m/s2.(1.5 puntos); por otro
lado, la velocidad desacelerando, cumple con la ecuación v( x)=√v0
2
+ 2a1 d , donde v0=22.22 m/s, y
d=150 m. En esa distancia v(150)=0, lo cual podemos despejar a1, dando a1=
−v0
2
2d
=−1.645 m/ s
2
(1.5 puntos). Como parte del reposo, la ecuación de velocidad que se obtiene es:
v(t)=∫
0
t
a(t)dt={
a0 t , t< 8s
a08+ a1(t−8) , t> 8 s
} y la ecuación de posición
x(t )=∫
0
t
v(t)dt=
{ a0
t2
2
, t< 8s
a0
8
2
2
+ a0 8(t−8)+ a1
(t−8)
2
2,
t> 8 s}Calculemos el momento cuando se detiene.
V(t)=0, que corresponde a t=8
(1−
a0
a1
)=21.519 s , cual implica que x(21.519 s)=239.30 m (1.5
puntos)
El gráfico de velocidad en función del tiempo es:
Profesor Miguel Bustamante S Temario
y la de posición en el tiempo
Pagina 32
Física-Mecánica
Pregunta 13
La forma de la función de velocidad de un movimiento unidimensional en función del tiempo viene
dado por la expresión v(t)=√400−(t−20)
2
m/ s .
Calcule:
• La velocidad máxima, y cuando se produce
• La expresión de la aceleración en función del tiempo.
• Si parte del origen, ¿en que posición está en t=40 s?.
Solución
La ecuación de velocidad se produce cuando , la derivada es cero. Es decir, la aceleración
a(t)=
20−t
√ 400−(20−t)
2
(2 puntos). Se observa que en t= 20 s se produce la velocidad máxima, que es
de 20 m/s. (2 puntos)
La posición viene dado por la expresión
x(t)=∫0
t
v(t)dt=((t−20)√(40t−t
2
)+ 400asin((t−20)/ 20))/2+ 100π y evaluado en t=40 da
π202
=1260,63 m (2 puntos) (Corresponde al área de la figura)
Profesor Miguel Bustamante S Temario
Pregunta 14
El pato Ruperto, volaba a velocidad constante v0, horizontal al suelo a una altura H. Un
manifestante, lo ve y le lanza un pequeño proyectil, de modo que el ángulo de lanzamiento
en el momento que lo ve es de a (con respecto a la horizontal) orientado hacia el pato
Ruperto. Sabiendo que la velocidad de salida de la mano del proyectil es V, ¿A que altura
volaba el Pato suponiendo que este proyectil le da ?
Solución
Debemos aplicar las ecuaciones de posición de los dos cuerpos.
La posición del pato ⃗
P(t)=(v0t+
H
tan (α)
, H )
El proyectil ⃗
r (t)=(vcos(α)t ,−
1
2
g t
2
+ vsin (α)t) . ( 2 puntos, por las ecuaciones)
Si le pega, debe cumplirse que
v0t=v cos(α)t−
H
tan(α)
y H =
−1
2
g t
2
+ vsin (α)t . ( 2 puntos, condiciones)
Despejamos el tiempo t
Pagina 34
a
v0
V
H
Física-Mecánica
t=
−H /tan(α)
v0−vcos(α)
. Reemplazamos t en la ecuación de posición vertical y(t) y nos una
ecuación para H
H =
−1
2
g H
2 tan
2
(α)
(v0−vcos(α))
2
−H V sin(α)
tan (α)
v0−vcos(α)
, lo cual nos da
H =
2
g
v0tang
2
(α)(v cos(α)−v0) ( 2 puntos).
Profesor Miguel Bustamante S Temario
Pregunta 15
Se tiene una argolla que puede deslizar sin roce por el perímetro de un
anillo de radio R. La argolla esta unido a una barra que desliza por ella sin roce.
De acuerdo a la información entregada, calcule
• La distancia OA en función de q(t)
• La velocidad de la argolla en coordenadas polares q=0° con ˙
θθ=0=
V 0
R
Solución
Utilizando el teorema del coseno la distancia OA puede ser calculada.
OA(β)=√R
2
+ R
2
−2R
2
cos(β)=R√2(1−cos(π−2θ(t)))
La velocidad viene dado por al expresión ⃗
v(t )=ṙ ̂
r+ r θ̇ ̂
θ .
Pagina 36
O
A
q
O
A
q
C
R
b
Física-Mecánica
En este caso
ecu 1
Evaluando la expresión 1 en q=0, nos da
ṙ=0 Luego, la velocidad es: ⃗
v=0 ̂
r +2R θ̇ ̂
θ=2V 0
̂
θ
Profesor Miguel Bustamante S Temario
ṙ=
sin(π−2θ)R
√(2)√(1−cos(π−2∗θ))
(−2θ̇)
Problema 16
Para estudiar un auto de carrera, se usa una cámara para registro de movimiento a alta velocidad se
coloca en el punto A. La cámara se monta sobre un mecanismo que permite registrar el movimiento
del carro conforme éste se mueve sobre la pista recta BC. Determine la velocidad del automovil en
términos de b,,̇,̈ .
Solución
La velocidad en función de la velocidad en coordenadas polares es: 
vt=ṙ 
rr ̇ 

En este caso, la magnitud r(t), depende del ángulo , de la forma r t=
b
cost
(2 puntos). La
derivada de r es : ṙ=
bsin
cos
2
̇ (2 puntos). La velocidad del automóvil es:

vt=
bsin
cos
2
̇ 
r
b
cos
̇ 
 . ( 2 puntos)
Pagina 38
(t)
b
r
Física-Mecánica
Problema 17
Un collar que desliza por un arco circular tiene un pasador que obliga a moverse por la ranura del
brazo AB. El brazo gira en el sentido anti-horario con celeridad angular (velocidad angular) de 2
rad/s.
Cuando el brazo está a 30º sobre la horizontal, determinar:
 Determinar la distancia r(t).
 Determinar la velocidad radial y tangencial (vr,v).
 Determinar la aceleración radial y tangencial.
Ayuda: El origen del sistema está en el punto A.
Solución
Por la geometría del problema se cumple que: R2
=u2
d2
−2udcos .( 1.5 puntos)
Despejamos u() de la expresión anterior.
u=d cos±R2
d2
sin 2
. Cuando =0°, el valor de u=dR. La solución es el positivo (+);
luego u=d cosR2
d2
sin2
. Como la velocidad angular es constante es decir
̇=w=2 rad/s .
Calculemos la velocidad 
v=u̇ 
ru̇ 
 .

v={
d2
wcostwsintw
R
2
d
2
sintw
2

−dw sintw}
r{d coswtR
2
d
2
sinwt
2
}w 
 (1.5 puntos)
Calculemos la aceleración 
a=ü 
r2u̇̇ 
−u̇ 
r
en este caso
Profesor Miguel Bustamante S Temario
ü=−d
2
w
2
sint w
2
/R
2
d
2
sin t w
2

d2
w2
costw2
/R2
d2
sintw2

−d4
w2
cost w2
sint w2
/R2
d2
sint w2
3/2
−d w2
cost w
(1.5 puntos)
En el caso de w= 2 rad/s, para un ángulo de 30° (/3), el tiempo t=/6.
u(/6)=200,958 mm (0.5 puntos ) u̇/6=−100.1 (0.5 puntos) y
ü/6=
10950
19∗19
−150=−17.78 (0.5 puntos)
Así:

v=−100.1
r401.916 
 mm/s,

a=−17.78
r−400.4 
−401.96 
r=−419.70
r−400.4 
 mm/s2
Pagina 40
Física-Mecánica
Problema 18
La aceleración de una partícula es directamente proporcional al cuadrado del tiempo t. Cuando
t= 2 s , v=-15 m/s y cuando t=10 s, v=0,36 m/s. Sabiendo que la partícula está dos veces mas lejos del
origen en t=2 s, que en t= 10 s Determine:
a) La posición de la partícula en t= 2 s y t=10 s
b) la distancia recorrida por la partícula de t=2 s a t = 10 s
Solución
Sabemos que at =kt2 . La velocidad es, por tanto
v t=∫
2
t
atdt−15 m/ s=k
t
3
3
−
8k
3
−15 m/ s (1 punto)
Por construcción v(2)=15 m/s, pero se debe satisfacer v(10)=0,36 m/s. Esto implica que el valor de k
debe satisfacer k
103
3
−
8 k
3
−15 m/s=0,36 m/s . el valor de k es:0.042 (1 punto).
La posición de la particula es
xt=∫
2
t
vtdtD=D0.0038333333333333t
4
−15.12266666666667t30.184 (2 puntos)
En x(2)=D, y en x(10)=D/2= 82.70933333 m +D. Así D=82.70933 m (1 punto)
Respuesta a), x(2)=82.709333 m y x(10)=165.418666 m (1 punto)
Respuesta b) La distancia recorrida es 82.716 m
Profesor Miguel Bustamante S Temario
Problema 19
Un auto-bus está estacionado a la orilla de un autopista cuando a su lado pasa un camión que
viaja con una velocidad de 45 mill/h. Dos minutos después, el auto bus arranca y acelera de manera
uniforme hasta que alcanza una velocidad de 60 mi/h, la cual mantiene. Sabiendo que 12 minutos
después de que el camión pasó junto al autobus, éste se encuentra 0,8 mi delante del camión, determine:
a) Cuándo y donde el autobus rebasa al camión
b) la aceleración uniforme del autobus
1 mi = 1,609 km
Solución
Debemos escribir la ecuaciones de posición de cada móvil.
Si situamos en donde está estacionado el auto bus, como nuestro origen, la ecuación del camión viene
dado por la ecuación de itinerario:
ct=20,1t m ( 1 punto)
Veamos el bus
bt=
0, t120 s
a
t−1202
2
, 120tt1
26,81t−t1x1, t1t
(2 puntos)
donde t1 es el tiempo demora en llegar a una velocidad de 60 mi/h=26,81 m/s y x1, es la posición en
t=t1.
La posición del camión t=12 min=720 s es 14472 m, que esta 0,8 mi=1287,2 m, luego la posición del
bus en t=720 s es 13185 m.
Luego, se debe satisfacer las siguientes ecuaciones
26,81720−t1x1=13185 m (2 puntos)
a
t1−1202
2
=x1
y
at1−120=26,81m/ s
Resolviendo el sistema se obtiene:
a=0.1238 m/s2 (respuesta b) ,t1=336.4117 s y x1=2901.0| m.
Según , la posición y el tiempo el bus va a una velocidad uniforme. Así, para encontrarse, deben estar al
mismo tiempo en el mismo instante, el bus y el camión. b(t)=c(t). El tiempo de intersección es
t=911.802 s y evaluando en cualquier de las posiciones se obtiene donde se encuentran
c(911.864)=18327.22 m.
Pagina 42
Física-Mecánica
Profesor Miguel Bustamante S Temario
Pagina 44
Física-Mecánica
Dinámica
Profesor Miguel Bustamante S Temario
Dinámica
Problema 1
Se tiene el siguiente sistema de masas:
Realice una investigación para encontrar la relación entre la aceleración de
la masa M y la fuerza F aplicada al cuerpo 2M.
Solución:
Lo primero que debemos identificar, son las fuerzas actuando en cada
masa.
La suma de las fuerzas libres la masa superior es : N+W+fr=Ma
La suma de las fuerzas sobre la masa inferior es: F+N2+W2+fr2-fr=2Ma.
Descomponiendo los vectores respecto del sistema de referencia es:
Masa Superior
En el eje X: -fr=-Ma
En el eje Y: N-Mg=0
En la masa inferior
En el eje X:-F+fr2+fr=-2Ma
En el eje Y: N2-N-2Mg=0
Pagina 46
Masa M
Coeficiente de roce
ms=0.8, mk=0.7
Masa 2M
Coeficiente de roce
m s=0.9, mk=0.8
F
fr
N
W
Fuerza actuando en el cuerpo superior
N2
-fr
F
W2
fr2
Fuerza actuando en el cuerpo inferior
i
j
-N
Física-Mecánica
De la ecuación del eje Y, cuerpo superior se tiene N=Mg, por tanto N2=3Mg (Eje
y, cuerpo inferior). Sumando las ecuaciones del eje X, se tiene que -F+fr2=-3Ma.
Si F £ msN2=ms3Mg, la aceleración es cero.
Si la fuerza es superior es esta, los cuerpos se mueven como uno sólo, es decir la
aceleración es a=(F-mk3Mg)/(3M)=F/(3M)-3m kg.
Esta ecuación válida solo cuando fr £ m s N. La aceleración crítica es cuando fr=
msMg=Ma
esto implica que a=msg. El valor de la fuerza es F= msMg+mk'2Mg-2Mmsg. Con
este valor, la masa superior está apunto de deslizar. Una fuerza F superior a esta
el cuerpo desliza y la aceleración se obtiene de la ecuación fr=mkMg.=Ma, esto
implica a=mkg.
En un gráfico esquemático nos queda de la forma:
Profesor Miguel Bustamante S Temario
a
F
a=0.7
g
0.9*3Mg Mg(2*0.7+0.8)
g(2*0.7+0.8)
Pregunta 2
Se tiene el siguiente conjunto de masa (m1, m2) sobre un plano inclinado sobre
la horizontal.
Calcule el ángulo para el cual el sistema comienza a moverse.
Aumente en 10° este ángulo y calcule la aceleración de cada masa.
Solución
Antes de comenzar a solucionar el problema, quiero llamar la atención de algo:
Fíjese que el coeficiente de roce estático de la masa m1, es mayor que el de la
masa m2. El ángulo crítico de deslizamiento para la masa m1 es 38.656° y el
ángulo crítico de la masa m2 es 30.963 °. Por tanto el ángulo que se busca debe
estar contenido entre estos dos valores; debe ser mayor a 30.9°, pero menor que
38.65°.
Veamos el diagrama del cuerpo libre:
Los vectores N1 y N2 son las normales de los cuerpos m1 y m2, fr1 y fr2 las
fuerzas de roce y W1 y W2 los pesos. La Fuerza P es la fuerza de contacto entre
los cuerpos.
Según la segunda ley de Newton, la suma de los fuerzas en el cuerpo debe ser
cero.
Pagina 48
a
m1
m2
Coeficiente de
roce:
m s=0.8 y m k=0.3
Coeficiente de roce
m s=0.6 y m k=0.2
a
m1
m2
W1
W2
N1
N2
fr1
fr2
x
y
P
-P
Física-Mecánica
Para el cuerpo m1, se tiene que debe cumplir
N1+W1-P+fr1=0
y el cuerpo m2
N2+W2+P+fr2=0.
Descomponiendo respecto al sistema de referencia, se tiene:
Cuerpo m1
N1j-Pi+fr1i-M1gcos(a )j-M1gsin(a )i=0 o
N1-M1gcos(a )=0 y
-P+fr1-M1gsin(a )=0.
Cuerpo 2
N2j+Pi+fr2-M2gsin(a )i-M2gcos(a )=0 o
N2-M2gcos(a )=0 y
P+fr2-M2gsin(a )=0
Sumando las ecuaciones 1.2 y 2.2 se obtiene fr1+fr2-(M1+M2)gsin(a)=0; es
decir la suma de las fuerzas de roce debe ser igual fr1+fr2=(M1+M2)gsin(a )
En la situación crítica la fuerza de roce es igual al coeficiente de roce estático
por la normal. En este caso, fr1=ms1N1=ms1M1gcos(a) y fr2=ms2N2=
m s2M2gcos(a).
Remplazando en la ecuación (3) se tiene que la ecuación para el ángulo
(ms1M1+ms2M2)cos(a)=(M1+M2)gsin(a)
La tangente del ángulo critico es: tang=
s1 M1s2 M2
M1M2
en el caso que el ángulo se aumente en 10°, a°=a+ 10° Las ecuaciones de
movimiento son:
Para el cuerpo m1, se tiene que debe cumplir
N1+W1-P+fr1=M1a
y el cuerpo m2
N2+W2+P+fr2=M2.
Descomponiendo respecto al sistema de referencia, se tiene:
Profesor Miguel Bustamante S Temario
1.1
1.2
2.1
2.2
( 3 )
Cuerpo m1
N1j-Pi+fr1i-M1gcos(a )j-M1gsin(a )i=0 o
N1-M1gcos(a )=0 y
-P+fr1-M1gsin(a )=-M1a.
Cuerpo 2
N2j+Pi+fr2-M2gsin(a )i-M2gcos(a )=0 o
N2-M2gcos(a )=0 y
P+fr2-M2gsin(a )=-M2a
Pagina 50
1.1
1.2
2.1
2.2
Física-Mecánica
Pregunta 3
Se tiene el siguiente esquema de un sistema de masas y plano inclinados
El sistema está en reposo, y se deja libre. Cuando el cuerpo (3) llega al suelo, no
hay mas tensión en la cuerda que liga el cuerpo (2) con (3) y queda detenido.
Sobre la información entregada calcule:
● La aceleración de la masa 1 en función del camino recorrido por la masa (1)
● Realice un gráfico (bosquejo) del punto anterior,
● Calcule la rapidez de la masa 2 en función del camino recorrido por la masa (1).
● Realice un gráfico (bosquejo) del punto anterior
Deje expresado los resultados en función de g, L, M. Recordar que sin(30)=1/2.
Solución
Para responder a todos los puntos anteriores, debemos analizar todas las fuerzas
actuando sobre cada masa.
En el caso de la masa 1, el diagrama de cuerpo libre es:
Las descomposición de los vectores, según el sistema
asociado a la masa 1, nos da:
Eje x: -T+Mgsin(30)=-Ma (Se supone que sube)
Eje y: N-Mgcos(30)=0
Profesor Miguel Bustamante S Temario
Masa M
Masa M
30
Masa M
g
L/4
L
(1)
(2)
(3)
N
Mg
T
De la masa 2
La ecuación de esta masa, según el sistema asociado:
Eje y: T-T''-Mg=-Ma (Se supone que baja).
De la masa 3
La ecuación de esta masa, según el sistema asociado:
Eje y: T''-Mg=-Ma (Se supone que baja).
Escribiendo todas las ecuaciones, se obtiene un sistema de ecuaciones, cuyas
incógnitas son: a,T y T''.
-T+Mgsin(30)=-Ma
T-T''-Mg=-Ma
T''-Mg=-Ma
Despejando las incógnitas, se obtiene que:
a=
g
3
2−sin30 , T''=Mg(1/3+sin(30)) y T=2Mg/3(1+sin(30)).
Este resultado es válido, solo cuando la masa 3 no alcance el suelo. Cuando lo alcance, el
sistema cambia; la tensión T'' no existe y las ecuaciones serán:
-T+Mgsin(30)=-Ma
T-Mg=-Ma
En este caso, la incógnita será a y T.
Se obtiene que a=
g
2
1−sin30 y T=Mg/2(1+sin(30))
Pagina 52
T
T''
Mg
T''
Mg
Física-Mecánica
Cuando la masa 2 llegue al suelo, no habrá tensión T, y por tanto la aceleración de la masa
1 va a ser igual a a=-gsin(30) (Cambia el sentido de la aceleración).
Del esquema se sabe que cuando la masa 3 recorre una distancia 3/4L, la aceleración de
la masa 1 cambia a a=
g
2
1−sin30 y que cuando la masa 2 recorre la distancia L, la
aceleración cambia a a=-gsin(30). Con esta información podemos realizar el siguiente
gráfico:
La masa 2 está sometida a la misma aceleración que la masa 1. La rapidez puede
estar expresada por la siguiente relación: v x=v0
2
2ax , donde a es la aceleración,
x es el camino recorrido y v0, es la velocidad inicial para x=0. Si los cuerpos partieron del
reposo, la rapidez de la masa 2 estaría dado por la expresión:
Profesor Miguel Bustamante S Temario
v x= 2xg
2
2−sin30 , 0x3/4L
3gL
4
2−sin30
2∗g
2
1−sin30x−
3L
4
 , Lx
3L
4
Para x>L la rapidez es cero.
El gráfico, tendría la siguiente forma:
Pagina 54
Figura 2: Velocidad en función de la posición posición
3L/4 L
V(3L/4)
V(L)
Física-Mecánica
Problema 4
Calcular la fuerza F que debe aplicarse sobre un bloque de masa =20 Kgr para evitar que el
bloque B de masa M=2 Kgr caiga. El coeficiente de roce estático entre los bloques es de 0.5
y la superficie horizontal no presenta fricción.
Solución
Debemos analizar las masas A y B por separado.
Según el sistema asociado a la a la masa MA, las fuerzas descompuestas quedan:
Eje x: F-P=MAa
Eje y: N-MAg-fr=0
La masa B.
La ecuaciones según el sistema de referencia asociado es:
Ejex: P=MBa
Eje y: fr-MBg=0
En la condición crítica fr=P=> MBg=MBa. Despejando a, se
tiene a=g/
Sumando las ecuaciones en X, se obtiene
F=(MA+MB)a=(MA+MB)g/.
Profesor Miguel Bustamante S Temario
A B
F
A
F
MA
g
N
-P
x
y
MB
g
P
fr
Problema 5
La masa m1 sobre una superficie horizontal sin fricción se conecta a la masa m2 por medio
de una polea móvil y una polea fija sin masas. a) Si a1 y a2 son magnitudes de la
aceleración de m1 y m2 respectivamente, determinar la relación entre estas aceleraciones.
Determine expresiones para b) las tensiones en la cuerdas y c) y las aceleraciones a1 y a2
en función de de m1, m2 y g.
Solución
Para resolver este problema, debemos realizar un diagrama de cuerpo libre de cada cuerpo.
Masa m1
Escribamos las fuerzas descompuesta según el sistema
de referencia asociado:
Eje x: T1-fr=m1a1
Eje y: N-m1g=0
Para la masa m2,
Pagina 56
m1
P
m2
m1
m1g
fr
T1
x
y
N
m2
T2
m2g
x
y
Física-Mecánica
Eje y:
T2-m2g=-m2a2.
Quiero hacer notar, que las tensiones son distintas, pro que son cuerdas distintas. Estas
tensiones están relacionadas por la polea del plano horizontal P.
Si la polea tiene una masa despreciable, como un momento despreciable1
, el diagrama en la
polea de las fuerzas sería:
Del diagrama se tiene que 2T1=T2. Ahora nos falta la aceleración:
Las cuerdas tienen un largo constante (Cuerdas inextensibles).
La cuerda L1 se debe cumplir la siguiente relación:
L1=x1L1+X2L1, donde x1L1 y x2L2 son las distancias entre la polea y la muralla y la masa
m1 respectivamente. De igual forma se debe cumplir que:
L2=x1L2+x2L2.
Resumiendo:
L1=x1L1+X2L1
L2=x1L2+x2L2.
1 Esta suposición no es del todo correcto. Las tensiones T1 no son iguales si tomamos en cuenta el torque sobre la polea.
Este tipo de problemas se vabn a ver en dinámica rotacional.
Profesor Miguel Bustamante S Temario
P
T2
T1
T1
m1
P
m2
Cuerda de largo L1
cuerda de largo L2
x1L2
x2L2
x1L1
x2L1
Derivando dos veces las relaciones anteriores se tiene:
d2
x1l1
dt2
=
−d2
x2L1
dt2 y
d2
x1l2
dt2
=
−d2
x2L2
dt2
Estas son las aceleraciones; en un diagrama:
En el caso de la cuerda L1, y por la aproximación tienen que tener el mismo valor de la
aceleración a1 (de lo contrario se corta la cuerda). Vectorialemente, es equivalente a 2a1=a2
ó -2a1+a2=0.
Las ecuaciones que gobiernan este sistema se reducen a:
Masa m1
Eje x: T1-fr=m1a1
Eje y: N-m1g=0
Masa m2
Eje y:
T2-m2g=-m2a2.
Polea
2T1=T2
cuerda
-2a1+a2=0
Con este sistema de ecuaciones se obtiene el siguiente resultado:
a1=g(m2-2m1)/(m1+m2)
a2=2g(m2-2m1)/(m1+m2)
T1=m1g+m1g(m2-2m1)/(m1+m2)
T2=2m1g+m1g(m2-2m1)/(m1+m2)
Pagina 58
P
a2
a1
a1
Física-Mecánica
Problema 6
Sobre un piso rugoso se lanza con rapidez v0 un bloque de masa M, sobre el
cual reposa otro bloque de masa m. Se observa que el bloque de masa m nunca
desliza respecto del bloque de masa M. El coeficiente de roce dinámico
(cinético) entre el bloque de masa M y el piso es d y el coeficiente de roce
estático entre el bloque de masa m y el bloque de masa M es s
1. Hacer un diagrama de cuerpo libre para cada bloque dibujando y
explicando cada fuerza que aparece en los diagramas
2. Escribir las ecuaciones necesarias para calcular todas las fuerzas que
aparecen en los diagramas de cuerpo libre.
3. Calcular la aceleración que experimentan los bloques.
Solución
El diagrama de cuerpo libre:
Por el diagrama del cuerpo libre:
Eje y: N-mg=0.
Eje x: -fr=-ma.
Analizando el segundo cuerpo por diagrama del cuerpo
Profesor Miguel Bustamante S Temario
M
m
v0
s
d
m
mg
N
fr
x
y
Mg
-N
fr
N'
fr'
libre
Por el diagrama del cuerpo libre:
Eje x: fr'+fr=-Ma
Eje y: -Mg+N'-N=0
Sabemos que la masa m no desliza con respecto a M. Esto implica que
ambos cuerpo experimentan la misma aceleración
Sumando las fuerzas en la dirección del eje x, de ambos cuerpos se obtiene
fr'=-(M+m)a (*)
El cuerpo inferior desliza con respecto al suelo. En este caso fr'= N'. N'=N+Mg y
N=mg, por tanto N'=(M+m)g.
De la ecuación (*), fr'=N'=(m+M)g=-(M+m)a. Simplificando se obtiene que
a=-g.
Pagina 60
Física-Mecánica
Problema 7
Una partícula es proyectada a la derecha a partir de x=0 con una velocidad inicial de 9 m/s. Si la
aceleración de la partícula está definida por la relación a=−0,6v3/2 , donde a y v están expresados
en m/s2 y m/s, respectivamente, determine:
a) la distancia que la partícula habrá recorrido cuando su velocidad es 4 m//s
b) el instante en el v= 1 m/s
c) el tiempo requerido para la partícula recorra 6 m.
Solución
Sabemos de la relación dv
dt
=
dv
dx
dx
dt
=−0,6 v
3/2
, lo que nos da
X v/s V
En v= 4
m/s, con X=8/3 m ( 2 puntos)
De la definición de aceleración, tenemos a=
dv
dt
=−0,6 v
3/2
, cual al integrar se obtiene que
V v/s t
Si v= 1 m/s, el tiempo t=20/9=2,22 s. (2 puntos).
Utilizando la ecuación X v/s V, se obtiene que v=1.44 m/s. Esta velocidad corresponde (ecuación V v/s
t) para t=10/6 s=1,666 s. (2 puntos)
Profesor Miguel Bustamante S Temario
−0,6x=2v−3
−2
v

2
3
=−0,6t
Problema 8
La aceleración debida a la gravedad a un altitud “y” sobre la superficie de la Tierra se puede expresar
como
a=
−32,2
1
y
20,9 x10
6

2
donde a está expresado en ft/s2. Usando esta expresión, calcule la altura alcanzada por un proyectil
disparado verticalmente hacia arriba desde la superficie de la Tierra si su velocidad inicial es
a) 1800 ft/s
b) 3000 ft/s
c) 36700 ft/s
Solución
ecu 2: Y v/s V ( 2 puntos)
, de cual se obtiene que
6,7298 x10
8
arctan
y
20,9 x10
9
=
v
2
2
−
v0
2
2
(2 puntos) Si v=0, luego la altura da:
Y v=20,9 x10
6
tan 
v
2
1,34 x10
9

Y(1800)=50534,4 ft. (1/3 puntos)
Y(3000)=140375,245 ft ( 1/3 puntos)
Y(36700)=32,9x10⁶ ft. (1/3 puntos)
Pagina 62
a=
dv
dt
=
dv
dy
dy
dt
=v
dv
dy
=
−32,2
1
1
y2
20,9x10
6

2 
Física-Mecánica
Problema 9
El movimiento bidimensional de una partícula está definido por las relaciones rt =32−e−t
 y
t=4t2e−t
 , donde r se expresa en metros, t en segundos y  en radianes. Determine la
velocidad y la aceleración de la partícula en t=0; en t=, ¿a qué conclusión se puede llegar respecto a
la trayectoria final de la partícula?
Solución
Primero, calculemos la primera y segunda derivada de r(t) y (t).
ṙ t=3e
−t
y r̈ t=−3e
−t
̇t=4−8e
−t
y ̈t=8e
−t
( 2 puntos)
Sabiendo que el vector velocidad en coordenadas polares es:

vt=ṙ 
rr ̇ 

Evaluado, 
v0=3
r−3x4 
=3 
r−12 
 y 
v∞=0 
r6∗4 
=24 
 ( 2 puntos)
La aceleración:

a=r̈−r ̇
r2ṙ ̇r ̈

Evaluando ⃗
a(0)=(0−3(−4)) ̂
r+ (6(−4)+ 24) ̂
θ=12 ̂
r+ 0 ̂
θ=12 ̂
r
y ⃗
a(∞)=(0−6x4)̂
r+ (2x0x4+ 6x0) ̂
θ=−24 ̂
r ( 2 puntos)
Profesor Miguel Bustamante S Temario
Problema 10
Se tiene un cuerpo de masa M, que sale en forma horizontal con una velocidad V,
que está a una altura h, sobre el suelo.
Suponiendo que la partícula de masa M, se puede en medio viscosa viene dado por la
expresión 
f =− 
v
Si M=1 kg, g=9,81 m/s2
, h=1,5 m, con V=5 m/s. Se sabe que el cuerpo cayó a 2 m del borde.
Calcule:
• La constante . (2 puntos)
• La velocidad con que llega a ese punto.(2 puntos)
• El tiempo que demora en llegar. (2 puntos)
Solución
Según la segunda ley: −
v−m
g=m
a . En coordenadas:
En x: − vx=m ˙
vx
En y: − vy−mg=m ˙
vy
De la ecuación en X se obtiene que vx t=V0x e
−
m
t
donde V0x, es la velocidad incial en x ; y en eje
Y v y t=
mg

e
−
m
t
−1 .
Al integrar la velocidad en X:, se obtiene la posición, idem en Y.
xt=∫
0
t
vx tdt=V0x
m

1−e
−
m
t
 , en el eje Y, yt=
−m² g

2
e
−
m
t
−
mg

t
m² g

2
h
Si llam,amo K=/m , y remplazando los valores de lo conocido, se obtiene que, que x debe
satisfacer
ecu 3:
y en y, K
2
g1−e
−t/K
−g K th=0 . Combinan do las dos igualdades,
se obtiene la siguiente relación entre el tiempo y K t=0,15K+0,4
Remplazando t en función de K, en la ecuación 3 se obtiene la ecuación
ecu 4: Ecuación de recurrencia de K
como se observa en el gráfico, existen dos punto de encuentro en 0, y un punto que debemos
determinar numéricamente.
Pagina 64
V
5K1−e
−t
K
=2
e−0,15K²0,4 K 
=1−0,4 K
Física-Mecánica
Buscando el punto de encuentro, distinto de cero, es K=1.609960311444378, por tanto
=1,6099 y el tiempo t=0,641 s ( Tiempo de caída )
Conocemos la constante  y t, podemos calcular en x e y: (1,782 ;1,0200) m/s
Profesor Miguel Bustamante S Temario
Figura 3: H(K) y G(K): tienen dos puntos de intersección
Problema 11
Se lanza verticalmente un cuerpo de masa M, con una velocidad inicial V0. Si parte del
origen, y esta sometida a una fuerza del tipo viscoso 
f =−∣
v∣2

v , calcule:
• La altura máxima en función de V,M y .
• El tiempo para llegar a la altura màxima
Solución
La ecuación dinámica es: −∣vy∣2
−mg=m ˙
vy . Despejando la velocidad, se obtiene
que:
v y t=
m g

tanatan
V0 
m g 
−
 g
m
t . La posición en función del tiempo se obtiene
integrando la velocidad,
yt =
m lncosatan
LV0
gm
−
gLT 
m

L

m ln LV0
2
gm
2L
−
m∗lnm
2L
−
ln gm
2L
Cuando la velocidad es cero, alcanza la altura máxima y ocurre cuando V(t)=0, ó
t=
atan
V0
mg

mmg 
si se reemplaza en la expresión de y(t), se obtiene la altura máxima
hmax=
m lnV0
2
gm
2
−
m lnm
2
−
ln gm
2
Pagina 66
V0
Física-Mecánica
Problema 12
Se tiene dos masas de valor m unidos por una cuerda inextensible. La cuerda pasa por orificio
en la mesa de modo que una de las masas cuelga; la otra gira en forma circular con radio R, libre de
roce.
• Calcule la velocidad angular necesaria para que la masa que cuelga no suba ni baje.
Solución
La tensión de la cuerda es T=mg, ya que la masa está en equilibrio.
La fuerza central del movimiento es T=mg=w2
Rm. Despejando w, se obtiene que w=
g
R
Profesor Miguel Bustamante S Temario
g
Problema 13
Se tiene un disco de radio R que gira a una velocidad angular de w. Sobre el disco hay dos masas M y
5M, unidos por una cuerda inextensible. La superficie del disco con las superficie de las masas genera
un roce estático y cinético (ms, mk). El peso es perpendicular a la superficie de rotación, entrando por la
hoja.
La distancia de la masa 5M al centro de rotación es h. Calcule:
• La frecuencia W, justo antes que comience a deslizar las masas.
• La tensión de la cuerda en ese instante.
Solución
Veamos una vista lateral de las masas
Escribamos la ecuación dinámica de la masa 5M
−Fr ̂
r+ N ̂
k−5Mg ̂
k+ T ̂
k=−5M w2
h ̂
r
La ecuación dinámica de M
−Fr' ̂
r+ N ' ̂
k−Mg ̂
k−T ̂
k=−M w2
(h+ R/ 4)̂
r
Descomponiendo por las componentes
Pagina 68
5M M
R
h
R/4
R/4
h
5mg
N
mg
N'
Fr'
Fr
Física-Mecánica
1:−Fr+ T=−5Mw2
h
2:−5Mg+ N =0
3:−Fr '−T =−Mw2
(h+ R/4)
4: N '−Mg=0.
Sumando las ecuaciones 1 y 4, se obtiene
Ecu 5
La condición crítica de equilibrio ocurre cuando Fr=μs N . Luego, la ecuación 5 se transforma
Ecu 6
Despejando w de la expresión 6 se obtiene
w=
√6μs g
6h+ R/4
Profesor Miguel Bustamante S Temario
−Fr−Fr '=−5Mw2
h−Mw2
(h+ R/4)
μs 5Mg+ μs Mg=6Mw2
h+ Mw2
R/4
Problema 14
Se tiene el siguiente sistema de masas.
Sobre la información entregada en la figura, se pregunta: ¿Se mueven las masas?
Si existe aceleración, calcule la aceleración de cada masa.
Solución
Debemos analizar las fuerzas de cada masa por separado
La masa colgando:
La masa 2M sobre la masa M
Pagina 70
Coeficiente de roce estático 0.8
Coeficiente de roce cinético 0.6
Roce estático:0.3
Roce cinético 0.2 Masa M
Masa 2M
Masa M
T
Mg
Física-Mecánica
La masa M
Nuestra hipótesis: “El sistema está en equilibrio”.
Masa colgante:
T−Mg=0
Masa 2M
N−2Mg=0
−Fr+ T =0
Masa Inferior:
−N+ N −Mg=0−Fr' + Fr=0
De las ecuaciones anteriores tenemos:
T=Mg ⇒Fr=T=Mg y Fr '=Fr
Condición de inminencia de movimiento
Fr⩽μs N ⇒Mg⩽μs 2Mg ⇒1⩽1.6 (La masa 2M no desliza sobre la mas M).
Esto no implica equilibrio, sólo implica que el conjunto se podría mover como todo (Masa 3M).
Fr '⩽μs' (3Mg)⇒Mg 1⩽3μs ' 1⩽0.9 . El sistema de masa 2M y la masa M sobre el plano se
mueve como un sólo bloque de 3M.
Así, podemos plantear las nuevas ecuaciones
Profesor Miguel Bustamante S Temario
T
N
-2Mg
fr
-fr
Fr'
N'
-N
-Mg
−Fr' + T=3Ma
T−Mg=−Ma
Luego
a=
g
4
(1−3μk )=0.025g=0.245 m/s
2
Pagina 72
Física-Mecánica
Pregunta 15
Se proyecta una carretera para un tráfico de 120 km/h. A lo largo de uno de sus tramos, el radio
de curvatura es de 270 m. La curva está peraltada como se observa en la figura de manera que no sea
necesario el rozamiento para mantener el auto en la calzada
Determine :
• El ángulo de peralte θ
• El coeficiente de roce mínimo si la carretera no estuviera peraltada.
Solución
Veamos las fuerzas actuando sobre el auto, cuya ecuación dinámica
⃗
w+ ⃗
N =m ⃗
a
En una descomposición en coordenadas cilíndricas, nos dan las ecuaciones
Eje vertical N cos(θ)−mg=0 ( 1 punto )
Eje radial −Nsin(θ)=−m
v2
R
(1 punto )
Combinando las ecuaciones se obtiene que tan (θ)=
v2
gR
=0.419 , lo que implica q=22.74ª. (1 punto)
Si está horizontal, es la fuerza de roce la responsable de mantener en órbita circular.
La ecuación dinámica de este cuerpo en coordenadas cilíndricas
− fr ̂
r+ N ̂
k−mg ̂
k=−m
v2
R
̂
r (1 punto)
Profesor Miguel Bustamante S Temario
W
N
W
fr
de lo cual se obtiene que fr =m
v2
R
En el momento de inminencia de movimiento, fr=msN=msmg. (1 punto)
Igualando las ecuaciones dr obtiene μs mg=m
V2
R
⇒μs=
V 2
gR
(1 punto)
Pagina 74
Física-Mecánica
Problema 16
Una esfera de masa m= 5 kg unida a una barra vertical mediante dos hilos, según se observa
en la figura 4. Cuando la masa gira en torno del eje de la barra, los hilos se tensan según la figura 4.
Calcular:
• Las tensiones de los hilos para w= 5 rad/s
• La velocidad angular w, para que la tensión de la cuerda unida a B sea T=0, pero estirada.
Solución
En un sistema de referencia cilíndrico, la fuerza actuando sobre la masa m son la tensión T1, la
tensión T2 y el peso W (ver figura 4).
Aplicando la segunda ley de Newton, se obtiene
⃗
T1+ ⃗
T2+ ⃗
W =m⃗
a
Descompongamos las tensiones y el peso según el sistema de referencia cilíndrico, donde el eje Z está
contenido en el eje de rotación.
⃗
T1=−T1 cos(α1) ̂
r +T1 sin(α1) ̂
k con tan(α1)=
60
60
=1⇒α1=π/4 (45°)
Profesor Miguel Bustamante S Temario
Figura 4: Esquema de la masa girando en torno al eje
⃗
T2=−T2 cos(α2) ̂
r−T2 sin(α2) ̂
k con tan(α2)=
67,5
60
=1,125 ,⇒α2=48,37°
⃗
W =−mg ̂
k y ⃗
a=−m w2
r ̂
r con r=60+10 cm =70 cm =0,7 m
Cada componente debe ser igual a cada la do de la ecuación, se obtiene que
k:
ecu 7
r:
ecu 8
Combinando las ecuaciones 7 y 8, se obtienen los siguientes resultados.
T2=
mw
2
r−
mg
tan (α1)
sin(α2)
tan(α2)+cos(α2)
Y T1=
[
mw
2
r−
mg
tan (α1)
sin (α2)
tan(α2)+cos(α2)
]sin(α2)
sin(α1)
+
mg
sin(α1)
En este caso, numéricamente hablando T1=98,123 N y T2=27,27 N.
En el caso T2=0⇒ mw2
r−
mg
tan(α1)
=0⇒ w=
√ g
r tan (α1)
=3,75rad / s
Pagina 76
T1 sin(α1)−T 2sin (α2)−mg=0
−T 1cos(α1)−T 2cos(α2)=−mw2
r
Física-Mecánica
Problema 17
Se tiene un masa M que desliza sin roce sobre una superficie. Se aplica una fuerza F, de modo
que una masa M, (Existe roce cionético k, y estático s en la superficie superior). Calcule el valor
mínimo de la fuerza F, para que la masa m está a punto de deslizar.
Solución
Debemos analizar las fuerzas que actúan en cada masa
La masa “m”
El diagrama de cuerpo libre, y descomponiendo las
fuerzas en el sistema de referencia
Y: N-mg=0
X: fr=ma
( 2 puntos)
Profesor Miguel Bustamante S Temario
Figura 1: Problema 1
M
m
40°
m
fr
N
Wm
La masa “M”
Descomponiendo las fuerzas, según el sistema
de coordenadas
Y: N'-N-Mg=0
X:Fcos(40)-fr=Ma
(2 puntos)
En la inminencia de movimiento se cumple que
fr=s N , como N=mg y fr=ma, se obtiene que
a=s g
Aplicando este resultado en el segundo cuerpo (M)
Se tiene que N=N+Mg=(m+M)g
En el eje X: Fcos40−s mg=M s g , del cual se desprende que F=s g
mM
cos40
(2 puntos)
Pagina 78
-fr
N'
WM
N
40°
Física-Mecánica
Problema 18
Determine:
● Si se mueve las masas (2M, 3/2M)
● Si acelera, calcule la aceleración. (Las poleas puntuales y no afectan el calculo).
Solución
Para resolver veste problema, primero se debe analizar las fuerzas que actúan en cada masa:
Masa “2M”
En el eje Y: T-Mg=-Ma ( 1 punto)
La masa “3/2 M”
En el eje X: -T+fr=-3/2Ma
En el eje Y: N-3/2Mg=0
( 1 punto)
Primero debemos saber si existe un movimiento. La hipótesis que vamos a usar es que “el sistema está
en equilibrio” (1 punto)
Con esta hipótesis, se tiene
T-2Mg=0
-T+fr=0
(1 punto)
Sumando las ecuaciones, se obtiene que fr=2Mg (1 punto) . La condición de equilibrio es
frs N=s 3/2Mg (1 punto). Luego, la ecuación que se obtiene es:
fr=2Mg≤s 3/2 Mg o 2≤3 , lo cumple con la hipótesis, e implica que el sistema está en
equilibrio.
Profesor Miguel Bustamante S Temario
Coeficiente de roce
s=2,0, k=0,4
Masa 2M
Masa 3/2M
x
y
Problema 19
En la figura, los bloques de masas M1= 8 kg y M2= 6 kg esta unidos por una cuerda ideal. Los
coeficientes de roce estático y dinámico entre la masa M1 y el plano inclinado son μs=0.2 y μk=0.1. El
ángulo de inclinación es de θ=30°.
Con la información entregada, calcule:
• La aceleración de cada masa
• la tensión en la cuerda.
Solución
Primero debemos identificar las fuerzas actuando en cada masa, pero sin la.
Escribamos las ecuaciones dinámicas de cada masa. La masa queda de la forma
ecu 9
La ecuación de la masa 2 es
Pagina 80
⃗
N + ⃗
w1+⃗
T=M1 ⃗
a1
30°
M1
M2
g
Existe roce
μs=0.2
μk=0.1
30°
M1
M2
g
Existe roce
μs=0.2
μk=0.1
N
T
w1
T
w2
x
y
y
s1
s2
Física-Mecánica
ecu 10
Se hace notar que las aceleraciones vectorialmente son distintas, como son las tensiones. Como
cada cuerpo tiene asociado un sistema de referencia, descomponemos las fuerzas (vectores) de acuerdo
al sistema de referencia asociado a cada masa.
Para la masa M1, la descomposición queda de la forma
ecu 11
y en la masa M2,
⃗
T=T ^
j, ⃗
w2=−M2 g ^
j , ⃗
a2=a2
^
j
Reemplazando las descomposición de los vectores en las ecuaciones 80 y 81, y separando por
direcciones , se obtienen
ecu 12
y
ecu 13
Como la cuerda es ideal, es por tanto inextensible; es decir, la suma de s1 y s2 es constante.
ecu 14
Derivando dos veces la ecuación 14, se obtiene la siguiente relación para las aceleraciones
a1+a2=0⇒a2=−a1
Combinando las ecuaciones 12, 13 y 14, resolvemos un sistema del cual se obtiene que la aceleración
a1 es
a1=
M2 g−M1 gsin(θ)
M1+ M2
Con los valores del problema, el valor de esta supuesta aceleración es (g=9.8 m/s2) 1.4 m/s2, sentido
positivo. Lo que implica que la fuerza apunta hacia el lado contrario. Este cálculo no resuelve el
problema. Sólo indica para donde va a puntar el roce.
Debemos reescribir nuevamente las ecuaciones dinámicas y descomponer las en los sistema
conocidos.
Profesor Miguel Bustamante S Temario
⃗
T+ ⃗
w2=m ⃗
a2
⃗
N=N ^
j
⃗
T=T ^
i
⃗
w1=−M1 gsin(θ)^
i−M1 gcos(θ)^
j
⃗
a1=a1
^
i
x:T−M1 gsin (θ)=M1 a1
y: N−M1 gcos(θ)=0
y:T−M2 g=M2 a2
s1+s2=L
La ecuación dinámica de la mas M1 queda ⃗
T+ ⃗
fr+ ⃗
N + ⃗
w1=M1 ⃗
a1 . La fuerza de roce, referido al
sistema de referencia asociado a M1, se puede escribir como ⃗
fr=−fr ^
i . El sistema ecuaciones 12,
queda escrita como
ecu 15
y la ecuación de la ecuación
Sin embargo, aún no podemos calcular la aceleración, ya que no sabemos si se mueve. Recordemos
que una condición de equilibrio debido a la fuerza de roce es |⃗
fr|⩽μs|⃗
N| . Para resolver esta
situación, vamos a adoptar una hipótesis :”El sistema esta en equilibrio”. Esta hipótesis implica que la
aceleración a1=0. Las ecuaciones nos quedan
x:T−fr−M1 gsin(θ)=0
y: N−M1 gcos(θ)=0
y
De la ecuación inferior se obtiene que T=M2g. Reemplazando en la ecuación de la masa M1, en la
dirección de x, se obtiene que fr=(M2−M1sin (θ)) g . Este es valor que debe tener el roce para esté
en equilibrio. Sin embargo debe satisfacer además, |⃗
fr|⩽μs|⃗
N| .
ecu 17
Veamos si la relación 17 es verdadera con los valores de este problema
Hemos llegado a una contradicción. Por tanto, las masas se mueven. Como se mueven, sabemos que la
fuerza de roce viene descrita por fr=μk N . Reemplazando esta relación en las ecuaciones 15 y 16, y
recordando que a2=-a1 se obtiene que la aceleración a1 es
Pagina 82
x:T−fr−M1 gsin(θ)=M1 a1
y: N−M1 gcos(θ)=0
fr=(M2−M1sin (θ)) g⩽μs|N|=μs M1 gcos(θ)
30°
M1
M2
g
Existe roce
μs=0.2
μk=0.1
N
T
w1
T
w2
x
y
y
s1
s2
fr
ecu 16
y:T−M2 g=M2 a2
y:T−M2 g=0
fr=(M2−M1sin(θ)) g=19,6⩽μs|N|=μs M1 gcos(θ)=0,2x 67,89=13,579
Física-Mecánica
a1=
(M2−M1(sin(θ)+μk cos(θ)))g
M1+M2
=0,92 m/ s
2
y la tensión T es T=M2 g−M2
(M2−M1(sin(θ)+μk cos(θ))) g
M1+ M2
=53,28N
Profesor Miguel Bustamante S Temario
Pagina 84
Física-Mecánica
Energía
Profesor Miguel Bustamante S Temario
Energía
Problema 1
A un cuerpo de masa m=1 Kgr se le aplica una fuerza F, de modo que éste se
mantiene comprimiendo al resorte en 0.3 m. En t=0 deja de actuar la fuerza F y el
cuerpo comienza a recorrer la pista ABCDEF , pasando por los puntos B y E con
una rapidez de 6 m/s. Si en los tramos AB y DE son tramos rugosos y la
constante de elasticidad del resorte es k=1000 N/m, calcule
a) Trabajo realizado por las fuerzas conservativas en el tramo AD
b) Trabajo realizado por el roce en los tramos rugosos
c) Sobre la base de los datos entregados, calcule los coeficiente de roce de los
tramos AB y DE.
Solución:
a)La fuerza conservativa en el tramo AD es el peso, el trabajo es
mgh=1*9.8*1.5 J= 14.7 J
b) El trabajo de la fuerza de roce es: W=Uf-Ui+Ekf-Eki. En el tramo AB el
trabajo de la fuera de roce es: W=14.7-1/2*1000*(03)²+1/2*1*6²-0 J=-12.3 J=-
m kMgcos(23) 1.5/sin(23) J
El trabajo de DE, W=Uf-Ui+Ekf-Eki=-Mg1.5=-45 J=-m kMgCos(50) 1.5 /Sin(50)
c). Los coeficientes de rocen son: m k(AB)=0.499 y m k(DE)=3.6
Pagina 86
1.5 m
B
C
1.7 m
D
E
37°
A
Fuerza F
g
1.5 m
23° 50 °
F
Física-Mecánica
Problema 2
Se tiene una pelotita de masa m que este en el punto B, en reposo. Un alumno golpea la
bolita dando una velocidad v=30 m/s y sube por un arco circunferencial de diámetro 60 m.
Sobre la información entregada:
·       ¿A qué altura se desprende la bolita del arco de semicircunferencia?
·       ¿Dónde cae la bolita? (Recuerde lanzamiento de proyectiles)
Solución
La solución de este problema involucra tanto la energía, como las fuerzas actando
sobre el cuerpo.
Si la persona le dio energía inicial, esta al subir por la rampa, se transforma en
potencial y cinética.
Ei=1/2mv2
=Ed=1/2mvd
2
+mghd (+)
donde vd es la velocidad en el momento que se desprende y hd es la altura ala cual se
desprende.
Realicemos un diagrama de cuerpo libre, justo en el momento que se desprende:
Profesor Miguel Bustamante S Temario
2R=60
m
V
2R=60
m
V

hd
La altura hd
se relaciona con el radio R y el ángulo  por
la ecuación hd
=R(1+sin())
La aceleración radial está en la misma dirección que la norma.
En esta dirección tenemos:
N+mgsin()=-mar
Cuando se desprende el objeto, la normal es cero (N=0). La ecuación anterior queda
mgsin()=mar
En este punto, podemos decir que la aceleración radial ar = Vd
2
/R. Con esta relación
tenemos que mgsin()=mVd
2
/R. (*)
De la expresión (*) despejamos la velocidad y la remplazamos en la expresión (+).
1/2mv2
=1/2mRgsin()+mgR(1+sin())
Despejando sin(), se tiene la siguiente igualdad:
sin=
1
3

v
2
gR
−2=0,35 Lo que implica que =20,487º. La altura hd=1,35R. La magnitud
de la velocidad a esa altura es: Vd=0,35Rg .
Desde este momento, el cuerpo se comporta como un proyectil.
Pagina 88
mg
N

2R=60
m
V

hd
La altura hd
se relaciona con el radio R y el ángulo  por
la ecuación hd
=R(1+sin())
V
Rcos()
O
Física-Mecánica
La posición inicial del proyectil es (-Rcos(),5R/4). La velocidad

Vd =
Rg
4
sin ,cos=
Rg
4

1
4,
15
4
 .
La posición en el tiempo del proyectil, tomando como origen el punto O, que descrito
por 
r t =
1
4 RG
4
t−R
15
4
,−
1
2
g t2

Rg
4
15
4
t 
5R
4

El saber donde cayó es equivalente a hacer r(t)=(x(t),0).
El tiempo que demora en llegar al suelo una ves que se ha desprendido:
t=
57 Rg15Rg
8g
=
Rg
8g
5 715=2,13
R
g
Luego la distancia a la que cae respecto del origen en donde cae es:

r t =−15
R
4
5715
R
64
,0=−0.701R,0
Profesor Miguel Bustamante S Temario
Problema 3
En el peligroso deporte del salto Bungee, un osado estudiante de Física salta desde un
puente con una cuerda elástica atado a sus tobillos especial sujeta a sus tobillos. La
longitud de la cuerda sin alargamiento es de 5H/7. El estudiante pesa Mg y el puente
está a H por encima de la superficie de un rio “seco”. Calcule la constante elástica de la
cuerda, suponiendo que es un resorte de modo que el estudiante se salve, no “mojándose”.
Solución
Realicemos un esquema del problema
Si el origen esta en el borde del puente, la energía inicial es cero Ei=0.
Cuando llega a máximo alargamiento la velocidad es cero (Energía cinética), y la energía
potencial U=-mgH y la potencial elástica 1/2k(H-5H/7)2
. La energía total en este punto es
0=-mgH+1/2K(2H/7)2
. Despejando K nos da K=49mg/(2H).
Pagina 90
Física-Mecánica
Problema 4
Los bloques representados en la figura, de igual masa, están unidos a los extremos de un
resorte ideal de constante k. Los bloques que partieron juntos del reposo desde la posición
en que x=0, se deslizan sin roce por los planos que tienen el mismo ángulo de inclinación. Si
el resorte está sin deformar en la posición en que x=0, y parte del reposo determine: La
posición de los bloques cuando el resorte alcanza la máxima compresión
Sugerencia: Tome la altura h=0 m en el borde superior.
Solución
Situemos nuestro origen del sistema en la parte superior del artefacto.
En este caso la energía total inicial es cero (Se supone que en x=0 el resorte no está
comprimido).
Cuando baja y llega a su máxima compresión, la velocidad es cero, es decir su
energía cinética es cero. Sólo quedan energía potencial y cinética.
En ese instante se cumple :
0=-2mgxcos()+1/2K(2xsin())2
.
Despejando x de esta ecuación nos da:
x=
mgcos 
ksin2
Profesor Miguel Bustamante S Temario
x x
α

Problema 5
Se tiene un resorte en posición vertical, con una constante elástica k=Mg/h y en presencia
de gravedad está unido a una masa M. Se comprime el resorte una distancia 3h, y se
suelta.
Calcule:
• La elongación máxima del resorte (La masa no se desprende: unido al resorte)
• La velocidad que tiene la masa M, cuando pasa por el punto de elongación cero del
resorte.
Solución
La energía inicial, es solo potencial es:
ecu 18:
.
Cuando alcanza la elongación máxima, la energía potencial es:
ecu 19:
Igualando la ecuación 18 con 19, podemos encontrar x, que tiene dos soluciones:
x+:(3hK-2gm)/K
x-: -3h.
La solución X- corresponde a la situación inicial. La solución es x+
Cuando, el resorte está en elongación x=0, sólo hay energía cinética.
Pagina 92
Figura 5: Resorte comprimido, en cero elongación, elongación máxima
Masa
Masa
3h
Masa
U=−mg3h
1
2
K 3h
2
Uf=
1
2
K x
2
mgx.
Física-Mecánica
La velocidad viene dado por la expresión: v=
9h2
K
m
−6hg .
Profesor Miguel Bustamante S Temario
ecu 20:
U=−mg3h
1
2
K 3h
2
=
1
2
m v
2
Problema 6
Una masa M, esta a una altura H sobre el suelo y se suelta (parte del reposo).
Sabiendo que K=2Mg/H, calcular:
• Cuanto se comprime el resorte. (Si este llega a comprimir)
• Donde se detiene.
Solución
La energía inicial, con el resorte en su elongación natural es Ui=MgH. Sólo en la zona de
roce disipa la energía, existe trabajo; además comprime el resorte una distancia x.
Suponiendo que se comprime el resorte, el trabajo será W =−μk Mg(H + x) ; la energía
potencial del resorte será U R=
1
2
K(H −x)
2
. Luego, aplicando que el trabajo es iguala
al cambio de las energía potencial y cinética y además en el punto x se detiene el cuerpo,
se obtiene la siguiente igualdad
ecu 21
, donde nuestra
incógnita es x.
Ordenando la ecuación 21 se
del cual x da dos valores: x=H/2 y x=-H. En este caso, la solución es la positiva.
LA fuerza que experimenta el resorte al comprimirse H/2, es
⃗
F=−K( H−H /2) ̂
i=
−2Mg
H
H
2
̂
i=−Mg ̂
i .
Pagina 94
H
M
K
2 H
H
Coeficiente de roce estático 0.8
Coeficiente de roce cinético 0.5
g
ecu 22
μk Mg( H+ x)+
1
2
K(H + x)
2
−MgH=0
W =−μk Mg(H + x)=
1
2
K( H+ x)
2
−MgH
Física-Mecánica
La fuerza máxima que el roce podría realizar para detenerlo es
Froce=μs Mg=0.8 Mg< Mg . Esto implica que se mueve.
Aplicando, a partir de este punto, que el trabajo es igual al cambio de energía, se obtiene
la siguiente ecuación
Wroce=−μk Mgd=0−
1
2
K ( H /2)
2
=
−MgH
4
que implica que d= H/2.
Profesor Miguel Bustamante S Temario
Chouqe: Impulso y conservación del momento
Pagina 96
Física-Mecánica
Choque
Problema 1
Dos bloques de masas m1=2.0 Kgr. Y m2 =4.0 Kgr. Se sueltan desde de una altura de
5.0 m sobre la pista sin fricción, como la que se muestra en la figura. Los bloques sufren un
choque frontal elástico.
• Determine las dos velocidades justo antes del choque
• Determine las velocidades después del choque.
• Determine la altura máxima a la cual sube cada bloque después del choque.
(El choque se produce cuando la altura es H=0 m)
Solución
En todo el sistema no hay pérdida de energía, se conserva. Si las masas m1 y m2
están a la misma altura y parten del reposo, llegando a nivel de altura cero, lo harán con la
misma velocidad. En este caso v=(2hg)1/2
=10 m/s.
En el caso de este choque, se conserva el momentun y la energía.
La ecuación del momentun antes del choque es:

pantes=m1 v 
i−m2 v 
i
El momentun después del choque

Pdespues=m1 v1

im2 v2

i
En este caso, el momentun se conserva, lo que implica
m1 v−m2 v=m1 v1m2 v2
La conservación de la energía, nos da la ecuación:
Einicial =
1
2
m1 v2

1
2
m2 v2
=Edespues=
1
2
m1 v1
2

1
2
m2 v2
2
Con esta última igualdad tenemos dos sistema de y dos incógnitas. Resolviendo el sistema
nos da las siguientes soluciones
v1=v=10 m/s y v1=(m1-3m2)v/(m1+m2)=-18 m/s, v2=-v=-10 m/s y
v2=(3m1-m2)v/(m1+m2)=3.33 m/s Cada masa posteriormente sale en sentidos contrarios.
Profesor Miguel Bustamante S Temario
M1=2.0Kgr
M2=4.0Kgr
H H
Como no hay roce, la energía cinética de cada masa se transforma en energía potencial. La
altura será h=v2
/(2g). en el caso de la masa m1, la altura es h1=16,2 m y h2=0,55 m (g=10
m/s2
).
Problema 2
Magila el gorila de masa M, come unas bananas con insecticida. Alucina y ataca a un
guardia, sobre el cual se abalanza con velocidad v (constante). El guardia, con el propósito
de detenerlo, reacciona disparándole con una metralleta de proyectiles de goma de masa m
= M/50.000 y cuya velocidad es u = 100v. Suponga que una vez que Magila disminuye su
velocidad como consecuencia del impacto de un proyectil la mantiene constante hasta el
siguiente impacto.
Calcule:
a. ¿Cuántos proyectiles debe disparar el guardia para detener al gorila? Suponga que
las balas chocan y caen cuando alcanzan al gorila.
b. Si la cadencia de tiro es T, ¿cuánto avanza Magila entre el tercer y cuarto impacto?
Solución
Un esquema de esta situación será:
Justo antes del choque , el momentun inicial es:
pi=Mv −mu
Después del primer choque
pf1=Mv1 . Igualando las dos cantidades, se tiene que
v1=v−
m
M
u=v−
1
50000
100v=v1−
1
500
 .
Para el segundo disparo, el momentun inicial antes del segundo impacto es:
pi2=Mv1−mu
El momentun final después del segundo choque pf2=Mv2. Igualando, se obtiene que v2=v(1-
2/500).
Pagina 98
No hay roce
V
u
Física-Mecánica
En el tercer impacto, el momento inicial es pi3=Mv2-mu, el momento final después del tercer
impacto pf3=Mv3, igualando y despejando, se tiene que la velocidad de Magila es v3=v(1-
3/500).
En forma sucesiva se obtiene que la velocidad enésima es: vn=v(1-n/500); por tanto con 500
balas podemos detener a Maguila.
Si la cadencia1
de Tiro es T, que la cantidad de tiros que salen por unidad de tiempo.
Por tanto el tiempo entre tiro y tiro es 1/T.
La velocidad después del tercer impacto es: v3=497/500u. La distancia que recorre hasta el
cuarto impacto es d=
497
500
u
1
T
1 Cadencia: Repetición de un fenómeno que sucede regularmente.
Profesor Miguel Bustamante S Temario
Problema 3
Dos bloques A y B de igual masa M, están unidos mediante un resorte de constante de
dureza k y largo natural L0. El sistema formado por los dos bloques descansa sobre un
plano horizontal liso. El resorte está en su largo natural. De pronto una bala de masa m y
rapidez v0 se incrusta en el bloque A (choque plástico e instantáneo), tal como muestra la
figura.
a) Calcular la rapidez v1 que adquiere el bloque A inmediatamente después que la
bala se ha incrustado en él
b) Cuando el resorte está comprimido el máximo los bloques viajan con la misma
rapidez v2. Calcular v2
c) Calcular la distancia a entre los bloques cuando el resorte se encuentra
comprimido al máximo
Solución
En este choque hay conservación del momentun, mas no de la energía.
El mometun inicial es 
pi=m v0

i . El momentun posterior al choque, instantáneo es:

pf =Mmv1

i . Como hay conservación, podemos obtener la velocidad v1,

v1=
m
Mm
v0

i
Posteriormente, se produce un segundo choque pero es elástico; el momemtun se conserva
y la energía.
El momentun justo antes del choque elástico es 
pf =Mmv1

i . Cuando alcanza
la máxima compresión el resorte, ambos cuerpos se mueven a la misma velocidad, por tanto
el momentun en esa situación es 
p=2MmV2

i . De la conservación de momentun
pi=pf, se tiene que V2 es igual a 
V2=
Mm
2Mm
V1
2

i=
m
2Mm
V0

i
La energía inicial después del primer choque es cinética e igual a
E1=
1
2
Mmv1
2
. Al comprimirse el resorte, la energía en ese momento es cinética y
potencial elástica. E=
1
2
KL0 −a2
1/22MmV2
2
. Remplazando el valor de v2 en
función de V0 y V1 en función de V0, el valor de a da: a=L0−
mV0
K  M
Mm2Mm
Pagina 100
Física-Mecánica
Problema 4
-Se tiene una barra de largo L, de ancho H y profundidad S. Esta barra tiene un orificio
circular de radio R (R<H/2 y R<L/2), coincidiendo el origen con el eje de simetría vertical y
cuyo centro está a una distancia d del centro sin orificio de la barra..
Calcule el centro de masa de sistema
Solución
Este problema se puede pensar que es un
rectángulo de la do L y H, que se ha extraído una
circunferencia.
La circunferencia debe tener la misma densidad
del objeto real. Si la masa de objeto inicial es M, la densidad es =
M
LH− R
2
El centro de masa, de acuerdo a un sistema en el punto O es:

rcm=
H/2 LH 
i−H/2  R
2

L
2
 LH 
j−L/2d  R
2
LH− R
2

=
L
2
−
d R2
LH− R
2
 
j
Profesor Miguel Bustamante S Temario
-
=
H
L
R
O
d
Problema 5
Utilizando la misma técnica, calcule el centro de masa de:
Solución
En este caso, la densidad es:
=
M
LH−

2

H
2

2
En esta caso, el centro de masa va estar contenido en la
linea de simetría. Sólo calculemos en el eje vertical, respecto de la
esquina inferior izquierda.
ycm=
L/2 LH−L−
4
3
H
2


2

H
2

2
 LH−

2

H
2

2

Pagina 102
L
H
Física-Mecánica
Problema 6
Una bala de 300 gr con una velocidad de 330 m/s impacta con un péndulo de masa 100 kg, que cuelga
desde un techo que está a 2 m. Si la bala queda incrustada en el péndulo, calcule:
• La velocidad justo después del impacto.
• Calcule la la altura que llega el péndulo.
Solución
Sabemos que el momento inicial ses 
p=mv 
i . Justo después del choque el momento

Pf =mMVf

i . Por conservación del momento, en el eje x, se obtiene Vf =
m
mM
v .
Una vez, efectuado el choque, la energía se conserva. La energía cinética se transforma en
energí potencial
1
2
mM  M
mM 
2
v2
=mMgH . Luego, despejando H=
m2
2gmM2
v
2
(3
puntos)
Profesor Miguel Bustamante S Temario
Guía de ejercicios de Física Mecánica
Guía de ejercicios de Física Mecánica
Guía de ejercicios de Física Mecánica
Guía de ejercicios de Física Mecánica
Guía de ejercicios de Física Mecánica
Guía de ejercicios de Física Mecánica
Guía de ejercicios de Física Mecánica
Guía de ejercicios de Física Mecánica
Guía de ejercicios de Física Mecánica
Guía de ejercicios de Física Mecánica
Guía de ejercicios de Física Mecánica
Guía de ejercicios de Física Mecánica
Guía de ejercicios de Física Mecánica
Guía de ejercicios de Física Mecánica
Guía de ejercicios de Física Mecánica
Guía de ejercicios de Física Mecánica
Guía de ejercicios de Física Mecánica
Guía de ejercicios de Física Mecánica
Guía de ejercicios de Física Mecánica
Guía de ejercicios de Física Mecánica

Más contenido relacionado

La actualidad más candente

Recta tangente normal y binormal
Recta tangente normal y binormalRecta tangente normal y binormal
Recta tangente normal y binormalmoisesdhp
 
Cours de diagnostic électronique automobile
Cours de diagnostic électronique automobileCours de diagnostic électronique automobile
Cours de diagnostic électronique automobilescorpios92
 
Le dimensionnement d'une batterie en installation solaire
Le dimensionnement d'une batterie en installation solaireLe dimensionnement d'une batterie en installation solaire
Le dimensionnement d'une batterie en installation solairesalem ben moussa
 
Cours --hydraulique-thorique-dbit-vitesse
Cours --hydraulique-thorique-dbit-vitesseCours --hydraulique-thorique-dbit-vitesse
Cours --hydraulique-thorique-dbit-vitesseiro2
 
3.1. perfil de egreso
3.1. perfil de egreso3.1. perfil de egreso
3.1. perfil de egresofigempa uce
 
solucionario mecanica vectorial para ingenieros - beer & johnston (dinamica)...
solucionario mecanica vectorial para ingenieros - beer  & johnston (dinamica)...solucionario mecanica vectorial para ingenieros - beer  & johnston (dinamica)...
solucionario mecanica vectorial para ingenieros - beer & johnston (dinamica)...Sohar Carr
 
Chapter 13 solutions_to_exercises (engineering circuit analysis 7th)
Chapter 13 solutions_to_exercises (engineering circuit analysis 7th)Chapter 13 solutions_to_exercises (engineering circuit analysis 7th)
Chapter 13 solutions_to_exercises (engineering circuit analysis 7th)Maamoun Hennache
 
Tp informatique industrielle
Tp informatique industrielleTp informatique industrielle
Tp informatique industrielleHajer Dahech
 
Conversiones sistema internacional-metrico
Conversiones sistema internacional-metricoConversiones sistema internacional-metrico
Conversiones sistema internacional-metricoAbisai Orduña
 
Diode & Diode Zener : Exercices Corrigés
Diode & Diode Zener : Exercices CorrigésDiode & Diode Zener : Exercices Corrigés
Diode & Diode Zener : Exercices CorrigésRAMZI EL IDRISSI
 
Aplicaciones de leyes de newton
Aplicaciones de leyes de newtonAplicaciones de leyes de newton
Aplicaciones de leyes de newtonMery Melendez
 
Devenir technicien en diagnostic et maintenance automobile
Devenir technicien en diagnostic et maintenance automobileDevenir technicien en diagnostic et maintenance automobile
Devenir technicien en diagnostic et maintenance automobileFatima AZOUZI
 
Matlab simulink partie 2
Matlab simulink partie 2Matlab simulink partie 2
Matlab simulink partie 2babaoui mohamed
 
Cours3 machine-courant-continu.pdf par www.lfaculte.com
Cours3 machine-courant-continu.pdf  par www.lfaculte.comCours3 machine-courant-continu.pdf  par www.lfaculte.com
Cours3 machine-courant-continu.pdf par www.lfaculte.comالحسين بوعيدا
 

La actualidad más candente (20)

Recta tangente normal y binormal
Recta tangente normal y binormalRecta tangente normal y binormal
Recta tangente normal y binormal
 
Cours de diagnostic électronique automobile
Cours de diagnostic électronique automobileCours de diagnostic électronique automobile
Cours de diagnostic électronique automobile
 
Le dimensionnement d'une batterie en installation solaire
Le dimensionnement d'une batterie en installation solaireLe dimensionnement d'une batterie en installation solaire
Le dimensionnement d'une batterie en installation solaire
 
Resumen laplace
Resumen laplaceResumen laplace
Resumen laplace
 
Cours --hydraulique-thorique-dbit-vitesse
Cours --hydraulique-thorique-dbit-vitesseCours --hydraulique-thorique-dbit-vitesse
Cours --hydraulique-thorique-dbit-vitesse
 
3.1. perfil de egreso
3.1. perfil de egreso3.1. perfil de egreso
3.1. perfil de egreso
 
Uii estatica
Uii estaticaUii estatica
Uii estatica
 
solucionario mecanica vectorial para ingenieros - beer & johnston (dinamica)...
solucionario mecanica vectorial para ingenieros - beer  & johnston (dinamica)...solucionario mecanica vectorial para ingenieros - beer  & johnston (dinamica)...
solucionario mecanica vectorial para ingenieros - beer & johnston (dinamica)...
 
Chapter 13 solutions_to_exercises (engineering circuit analysis 7th)
Chapter 13 solutions_to_exercises (engineering circuit analysis 7th)Chapter 13 solutions_to_exercises (engineering circuit analysis 7th)
Chapter 13 solutions_to_exercises (engineering circuit analysis 7th)
 
Tp informatique industrielle
Tp informatique industrielleTp informatique industrielle
Tp informatique industrielle
 
Conversiones sistema internacional-metrico
Conversiones sistema internacional-metricoConversiones sistema internacional-metrico
Conversiones sistema internacional-metrico
 
Zemansky
ZemanskyZemansky
Zemansky
 
Icfes
IcfesIcfes
Icfes
 
Diode & Diode Zener : Exercices Corrigés
Diode & Diode Zener : Exercices CorrigésDiode & Diode Zener : Exercices Corrigés
Diode & Diode Zener : Exercices Corrigés
 
Redresseurs
RedresseursRedresseurs
Redresseurs
 
Aplicaciones de leyes de newton
Aplicaciones de leyes de newtonAplicaciones de leyes de newton
Aplicaciones de leyes de newton
 
Devenir technicien en diagnostic et maintenance automobile
Devenir technicien en diagnostic et maintenance automobileDevenir technicien en diagnostic et maintenance automobile
Devenir technicien en diagnostic et maintenance automobile
 
Matlab simulink partie 2
Matlab simulink partie 2Matlab simulink partie 2
Matlab simulink partie 2
 
Cours3 machine-courant-continu.pdf par www.lfaculte.com
Cours3 machine-courant-continu.pdf  par www.lfaculte.comCours3 machine-courant-continu.pdf  par www.lfaculte.com
Cours3 machine-courant-continu.pdf par www.lfaculte.com
 
Trabajo y energia
Trabajo y energiaTrabajo y energia
Trabajo y energia
 

Similar a Guía de ejercicios de Física Mecánica

Álgebra y Trigonometría - Sullivan - 07.pdf
Álgebra y Trigonometría - Sullivan - 07.pdfÁlgebra y Trigonometría - Sullivan - 07.pdf
Álgebra y Trigonometría - Sullivan - 07.pdfManuel Ortiz
 
Diseño instruccional curso fae
Diseño instruccional curso faeDiseño instruccional curso fae
Diseño instruccional curso faeNestor Pedraza
 
Chela ivan sesion#19_20
Chela ivan sesion#19_20Chela ivan sesion#19_20
Chela ivan sesion#19_20Ivan Vyn
 
Álgebra - Arturo Márquez, Fabián Valapai Bravo - 1ra Edición.pdf
Álgebra - Arturo Márquez, Fabián Valapai Bravo - 1ra Edición.pdfÁlgebra - Arturo Márquez, Fabián Valapai Bravo - 1ra Edición.pdf
Álgebra - Arturo Márquez, Fabián Valapai Bravo - 1ra Edición.pdfPinta p
 
Álgebra - Arturo Márquez, Fabián Valapai Bravo - 1ra Edición.pdf
Álgebra - Arturo Márquez, Fabián Valapai Bravo - 1ra Edición.pdfÁlgebra - Arturo Márquez, Fabián Valapai Bravo - 1ra Edición.pdf
Álgebra - Arturo Márquez, Fabián Valapai Bravo - 1ra Edición.pdfPinta p
 
Gestor plantillaproyectoactividad maggy
Gestor plantillaproyectoactividad maggyGestor plantillaproyectoactividad maggy
Gestor plantillaproyectoactividad maggyLemus Angelica
 
Unidad didáctica y adaptación curricular "Potencias y raíces"
Unidad didáctica y adaptación curricular "Potencias y raíces"Unidad didáctica y adaptación curricular "Potencias y raíces"
Unidad didáctica y adaptación curricular "Potencias y raíces"xoseamar
 
Unidad didáctica y adaptación curricular "Potencias y raíces"
Unidad didáctica y adaptación curricular "Potencias y raíces"Unidad didáctica y adaptación curricular "Potencias y raíces"
Unidad didáctica y adaptación curricular "Potencias y raíces"xoseamar
 
Lineamientos para realización de actividad pbl
Lineamientos para realización de actividad pblLineamientos para realización de actividad pbl
Lineamientos para realización de actividad pblEdgar Mata
 

Similar a Guía de ejercicios de Física Mecánica (20)

sullivan 7 edicion.pdf
sullivan 7 edicion.pdfsullivan 7 edicion.pdf
sullivan 7 edicion.pdf
 
Álgebra y Trigonometría - Sullivan - 07.pdf
Álgebra y Trigonometría - Sullivan - 07.pdfÁlgebra y Trigonometría - Sullivan - 07.pdf
Álgebra y Trigonometría - Sullivan - 07.pdf
 
Blog
BlogBlog
Blog
 
Plan de clase 3-Probabilidad y Estadística.docx
Plan de clase 3-Probabilidad y Estadística.docxPlan de clase 3-Probabilidad y Estadística.docx
Plan de clase 3-Probabilidad y Estadística.docx
 
Diseño instruccional curso fae
Diseño instruccional curso faeDiseño instruccional curso fae
Diseño instruccional curso fae
 
Experiencia de e-learning
Experiencia de e-learningExperiencia de e-learning
Experiencia de e-learning
 
Chela ivan sesion#19_20
Chela ivan sesion#19_20Chela ivan sesion#19_20
Chela ivan sesion#19_20
 
Álgebra - Arturo Márquez, Fabián Valapai Bravo - 1ra Edición.pdf
Álgebra - Arturo Márquez, Fabián Valapai Bravo - 1ra Edición.pdfÁlgebra - Arturo Márquez, Fabián Valapai Bravo - 1ra Edición.pdf
Álgebra - Arturo Márquez, Fabián Valapai Bravo - 1ra Edición.pdf
 
Álgebra - Arturo Márquez, Fabián Valapai Bravo - 1ra Edición.pdf
Álgebra - Arturo Márquez, Fabián Valapai Bravo - 1ra Edición.pdfÁlgebra - Arturo Márquez, Fabián Valapai Bravo - 1ra Edición.pdf
Álgebra - Arturo Márquez, Fabián Valapai Bravo - 1ra Edición.pdf
 
Gestor plantillaproyectoactividad maggy
Gestor plantillaproyectoactividad maggyGestor plantillaproyectoactividad maggy
Gestor plantillaproyectoactividad maggy
 
Matematica basica
Matematica basicaMatematica basica
Matematica basica
 
Mate bas
Mate basMate bas
Mate bas
 
Mate bas
Mate basMate bas
Mate bas
 
Unidad didáctica y adaptación curricular "Potencias y raíces"
Unidad didáctica y adaptación curricular "Potencias y raíces"Unidad didáctica y adaptación curricular "Potencias y raíces"
Unidad didáctica y adaptación curricular "Potencias y raíces"
 
Unidad didáctica y adaptación curricular "Potencias y raíces"
Unidad didáctica y adaptación curricular "Potencias y raíces"Unidad didáctica y adaptación curricular "Potencias y raíces"
Unidad didáctica y adaptación curricular "Potencias y raíces"
 
Lineamientos para realización de actividad pbl
Lineamientos para realización de actividad pblLineamientos para realización de actividad pbl
Lineamientos para realización de actividad pbl
 
Tecn estudio
Tecn estudioTecn estudio
Tecn estudio
 
100000 x103 nivelaciondematematicahumanidades
100000 x103 nivelaciondematematicahumanidades100000 x103 nivelaciondematematicahumanidades
100000 x103 nivelaciondematematicahumanidades
 
Fisica i medina h.
Fisica i   medina h.Fisica i   medina h.
Fisica i medina h.
 
Metodologia del aprendizaje
Metodologia del aprendizajeMetodologia del aprendizaje
Metodologia del aprendizaje
 

Más de Independiente

Problemas resueltos de Electromagnetismo: Desayunos de Electro
Problemas resueltos de Electromagnetismo: Desayunos de ElectroProblemas resueltos de Electromagnetismo: Desayunos de Electro
Problemas resueltos de Electromagnetismo: Desayunos de ElectroIndependiente
 
Problemas de Mecánica: Desayuno, el alimento importante.
Problemas de Mecánica:  Desayuno, el alimento importante.Problemas de Mecánica:  Desayuno, el alimento importante.
Problemas de Mecánica: Desayuno, el alimento importante.Independiente
 
colision-satelite.pdf
colision-satelite.pdfcolision-satelite.pdf
colision-satelite.pdfIndependiente
 
Ejercicios resueltos de Electromagnetismo
Ejercicios resueltos de ElectromagnetismoEjercicios resueltos de Electromagnetismo
Ejercicios resueltos de ElectromagnetismoIndependiente
 
Medición del momento magnético de un imán usando un teléfono inteligente.
	Medición del momento magnético de un imán usando un  teléfono inteligente.	Medición del momento magnético de un imán usando un  teléfono inteligente.
Medición del momento magnético de un imán usando un teléfono inteligente.Independiente
 
Teoria de errores gráfico-ajuste-2020
Teoria de errores gráfico-ajuste-2020Teoria de errores gráfico-ajuste-2020
Teoria de errores gráfico-ajuste-2020Independiente
 
Paper 101 bustamante lefranc osorio lefranc ieee ica-acca2016
Paper 101 bustamante lefranc osorio lefranc ieee ica-acca2016Paper 101 bustamante lefranc osorio lefranc ieee ica-acca2016
Paper 101 bustamante lefranc osorio lefranc ieee ica-acca2016Independiente
 

Más de Independiente (20)

Problemas resueltos de Electromagnetismo: Desayunos de Electro
Problemas resueltos de Electromagnetismo: Desayunos de ElectroProblemas resueltos de Electromagnetismo: Desayunos de Electro
Problemas resueltos de Electromagnetismo: Desayunos de Electro
 
Problemas de Mecánica: Desayuno, el alimento importante.
Problemas de Mecánica:  Desayuno, el alimento importante.Problemas de Mecánica:  Desayuno, el alimento importante.
Problemas de Mecánica: Desayuno, el alimento importante.
 
IA_mayo-2023.pdf
IA_mayo-2023.pdfIA_mayo-2023.pdf
IA_mayo-2023.pdf
 
caida-agua.pdf
caida-agua.pdfcaida-agua.pdf
caida-agua.pdf
 
colision-satelite.pdf
colision-satelite.pdfcolision-satelite.pdf
colision-satelite.pdf
 
Fisica-wmaxima1
Fisica-wmaxima1Fisica-wmaxima1
Fisica-wmaxima1
 
IMagen-Tec.pdf
IMagen-Tec.pdfIMagen-Tec.pdf
IMagen-Tec.pdf
 
Ejercicios resueltos de Electromagnetismo
Ejercicios resueltos de ElectromagnetismoEjercicios resueltos de Electromagnetismo
Ejercicios resueltos de Electromagnetismo
 
Medición del momento magnético de un imán usando un teléfono inteligente.
	Medición del momento magnético de un imán usando un  teléfono inteligente.	Medición del momento magnético de un imán usando un  teléfono inteligente.
Medición del momento magnético de un imán usando un teléfono inteligente.
 
Puntoslagrnge2
Puntoslagrnge2Puntoslagrnge2
Puntoslagrnge2
 
Teoria de errores gráfico-ajuste-2020
Teoria de errores gráfico-ajuste-2020Teoria de errores gráfico-ajuste-2020
Teoria de errores gráfico-ajuste-2020
 
CIECS 2021
CIECS 2021CIECS 2021
CIECS 2021
 
Calculadora
CalculadoraCalculadora
Calculadora
 
Ondas 1
Ondas 1Ondas 1
Ondas 1
 
Mec calor
Mec calorMec calor
Mec calor
 
Proyectil viscoso
Proyectil viscosoProyectil viscoso
Proyectil viscoso
 
Laboratorio2017
Laboratorio2017Laboratorio2017
Laboratorio2017
 
Ondas 2017
Ondas 2017Ondas 2017
Ondas 2017
 
Paper 101 bustamante lefranc osorio lefranc ieee ica-acca2016
Paper 101 bustamante lefranc osorio lefranc ieee ica-acca2016Paper 101 bustamante lefranc osorio lefranc ieee ica-acca2016
Paper 101 bustamante lefranc osorio lefranc ieee ica-acca2016
 
Clase fisicanuclear
Clase fisicanuclearClase fisicanuclear
Clase fisicanuclear
 

Último

Manual - ABAS II completo 263 hojas .pdf
Manual - ABAS II completo 263 hojas .pdfManual - ABAS II completo 263 hojas .pdf
Manual - ABAS II completo 263 hojas .pdfMaryRotonda1
 
codigos HTML para blogs y paginas web Karina
codigos HTML para blogs y paginas web Karinacodigos HTML para blogs y paginas web Karina
codigos HTML para blogs y paginas web Karinavergarakarina022
 
Movimientos Precursores de La Independencia en Venezuela
Movimientos Precursores de La Independencia en VenezuelaMovimientos Precursores de La Independencia en Venezuela
Movimientos Precursores de La Independencia en Venezuelacocuyelquemao
 
Identificación de componentes Hardware del PC
Identificación de componentes Hardware del PCIdentificación de componentes Hardware del PC
Identificación de componentes Hardware del PCCesarFernandez937857
 
NARRACIONES SOBRE LA VIDA DEL GENERAL ELOY ALFARO
NARRACIONES SOBRE LA VIDA DEL GENERAL ELOY ALFARONARRACIONES SOBRE LA VIDA DEL GENERAL ELOY ALFARO
NARRACIONES SOBRE LA VIDA DEL GENERAL ELOY ALFAROJosé Luis Palma
 
SINTAXIS DE LA ORACIÓN SIMPLE 2023-2024.pptx
SINTAXIS DE LA ORACIÓN SIMPLE 2023-2024.pptxSINTAXIS DE LA ORACIÓN SIMPLE 2023-2024.pptx
SINTAXIS DE LA ORACIÓN SIMPLE 2023-2024.pptxlclcarmen
 
Factores ecosistemas: interacciones, energia y dinamica
Factores ecosistemas: interacciones, energia y dinamicaFactores ecosistemas: interacciones, energia y dinamica
Factores ecosistemas: interacciones, energia y dinamicaFlor Idalia Espinoza Ortega
 
MAYO 1 PROYECTO día de la madre el amor más grande
MAYO 1 PROYECTO día de la madre el amor más grandeMAYO 1 PROYECTO día de la madre el amor más grande
MAYO 1 PROYECTO día de la madre el amor más grandeMarjorie Burga
 
PRIMER SEMESTRE 2024 ASAMBLEA DEPARTAMENTAL.pptx
PRIMER SEMESTRE 2024 ASAMBLEA DEPARTAMENTAL.pptxPRIMER SEMESTRE 2024 ASAMBLEA DEPARTAMENTAL.pptx
PRIMER SEMESTRE 2024 ASAMBLEA DEPARTAMENTAL.pptxinformacionasapespu
 
RAIZ CUADRADA Y CUBICA PARA NIÑOS DE PRIMARIA
RAIZ CUADRADA Y CUBICA PARA NIÑOS DE PRIMARIARAIZ CUADRADA Y CUBICA PARA NIÑOS DE PRIMARIA
RAIZ CUADRADA Y CUBICA PARA NIÑOS DE PRIMARIACarlos Campaña Montenegro
 
cortes de luz abril 2024 en la provincia de tungurahua
cortes de luz abril 2024 en la provincia de tungurahuacortes de luz abril 2024 en la provincia de tungurahua
cortes de luz abril 2024 en la provincia de tungurahuaDANNYISAACCARVAJALGA
 
Planificacion Anual 2do Grado Educacion Primaria 2024 Ccesa007.pdf
Planificacion Anual 2do Grado Educacion Primaria   2024   Ccesa007.pdfPlanificacion Anual 2do Grado Educacion Primaria   2024   Ccesa007.pdf
Planificacion Anual 2do Grado Educacion Primaria 2024 Ccesa007.pdfDemetrio Ccesa Rayme
 
PPT GESTIÓN ESCOLAR 2024 Comités y Compromisos.pptx
PPT GESTIÓN ESCOLAR 2024 Comités y Compromisos.pptxPPT GESTIÓN ESCOLAR 2024 Comités y Compromisos.pptx
PPT GESTIÓN ESCOLAR 2024 Comités y Compromisos.pptxOscarEduardoSanchezC
 
Introducción:Los objetivos de Desarrollo Sostenible
Introducción:Los objetivos de Desarrollo SostenibleIntroducción:Los objetivos de Desarrollo Sostenible
Introducción:Los objetivos de Desarrollo SostenibleJonathanCovena1
 
Herramientas de Inteligencia Artificial.pdf
Herramientas de Inteligencia Artificial.pdfHerramientas de Inteligencia Artificial.pdf
Herramientas de Inteligencia Artificial.pdfMARIAPAULAMAHECHAMOR
 
RETO MES DE ABRIL .............................docx
RETO MES DE ABRIL .............................docxRETO MES DE ABRIL .............................docx
RETO MES DE ABRIL .............................docxAna Fernandez
 
EXPECTATIVAS vs PERSPECTIVA en la vida.
EXPECTATIVAS vs PERSPECTIVA  en la vida.EXPECTATIVAS vs PERSPECTIVA  en la vida.
EXPECTATIVAS vs PERSPECTIVA en la vida.DaluiMonasterio
 
la unidad de s sesion edussssssssssssssscacio fisca
la unidad de s sesion edussssssssssssssscacio fiscala unidad de s sesion edussssssssssssssscacio fisca
la unidad de s sesion edussssssssssssssscacio fiscaeliseo91
 

Último (20)

Manual - ABAS II completo 263 hojas .pdf
Manual - ABAS II completo 263 hojas .pdfManual - ABAS II completo 263 hojas .pdf
Manual - ABAS II completo 263 hojas .pdf
 
codigos HTML para blogs y paginas web Karina
codigos HTML para blogs y paginas web Karinacodigos HTML para blogs y paginas web Karina
codigos HTML para blogs y paginas web Karina
 
Movimientos Precursores de La Independencia en Venezuela
Movimientos Precursores de La Independencia en VenezuelaMovimientos Precursores de La Independencia en Venezuela
Movimientos Precursores de La Independencia en Venezuela
 
Unidad 4 | Teorías de las Comunicación | MCDI
Unidad 4 | Teorías de las Comunicación | MCDIUnidad 4 | Teorías de las Comunicación | MCDI
Unidad 4 | Teorías de las Comunicación | MCDI
 
Identificación de componentes Hardware del PC
Identificación de componentes Hardware del PCIdentificación de componentes Hardware del PC
Identificación de componentes Hardware del PC
 
NARRACIONES SOBRE LA VIDA DEL GENERAL ELOY ALFARO
NARRACIONES SOBRE LA VIDA DEL GENERAL ELOY ALFARONARRACIONES SOBRE LA VIDA DEL GENERAL ELOY ALFARO
NARRACIONES SOBRE LA VIDA DEL GENERAL ELOY ALFARO
 
SINTAXIS DE LA ORACIÓN SIMPLE 2023-2024.pptx
SINTAXIS DE LA ORACIÓN SIMPLE 2023-2024.pptxSINTAXIS DE LA ORACIÓN SIMPLE 2023-2024.pptx
SINTAXIS DE LA ORACIÓN SIMPLE 2023-2024.pptx
 
Power Point: "Defendamos la verdad".pptx
Power Point: "Defendamos la verdad".pptxPower Point: "Defendamos la verdad".pptx
Power Point: "Defendamos la verdad".pptx
 
Factores ecosistemas: interacciones, energia y dinamica
Factores ecosistemas: interacciones, energia y dinamicaFactores ecosistemas: interacciones, energia y dinamica
Factores ecosistemas: interacciones, energia y dinamica
 
MAYO 1 PROYECTO día de la madre el amor más grande
MAYO 1 PROYECTO día de la madre el amor más grandeMAYO 1 PROYECTO día de la madre el amor más grande
MAYO 1 PROYECTO día de la madre el amor más grande
 
PRIMER SEMESTRE 2024 ASAMBLEA DEPARTAMENTAL.pptx
PRIMER SEMESTRE 2024 ASAMBLEA DEPARTAMENTAL.pptxPRIMER SEMESTRE 2024 ASAMBLEA DEPARTAMENTAL.pptx
PRIMER SEMESTRE 2024 ASAMBLEA DEPARTAMENTAL.pptx
 
RAIZ CUADRADA Y CUBICA PARA NIÑOS DE PRIMARIA
RAIZ CUADRADA Y CUBICA PARA NIÑOS DE PRIMARIARAIZ CUADRADA Y CUBICA PARA NIÑOS DE PRIMARIA
RAIZ CUADRADA Y CUBICA PARA NIÑOS DE PRIMARIA
 
cortes de luz abril 2024 en la provincia de tungurahua
cortes de luz abril 2024 en la provincia de tungurahuacortes de luz abril 2024 en la provincia de tungurahua
cortes de luz abril 2024 en la provincia de tungurahua
 
Planificacion Anual 2do Grado Educacion Primaria 2024 Ccesa007.pdf
Planificacion Anual 2do Grado Educacion Primaria   2024   Ccesa007.pdfPlanificacion Anual 2do Grado Educacion Primaria   2024   Ccesa007.pdf
Planificacion Anual 2do Grado Educacion Primaria 2024 Ccesa007.pdf
 
PPT GESTIÓN ESCOLAR 2024 Comités y Compromisos.pptx
PPT GESTIÓN ESCOLAR 2024 Comités y Compromisos.pptxPPT GESTIÓN ESCOLAR 2024 Comités y Compromisos.pptx
PPT GESTIÓN ESCOLAR 2024 Comités y Compromisos.pptx
 
Introducción:Los objetivos de Desarrollo Sostenible
Introducción:Los objetivos de Desarrollo SostenibleIntroducción:Los objetivos de Desarrollo Sostenible
Introducción:Los objetivos de Desarrollo Sostenible
 
Herramientas de Inteligencia Artificial.pdf
Herramientas de Inteligencia Artificial.pdfHerramientas de Inteligencia Artificial.pdf
Herramientas de Inteligencia Artificial.pdf
 
RETO MES DE ABRIL .............................docx
RETO MES DE ABRIL .............................docxRETO MES DE ABRIL .............................docx
RETO MES DE ABRIL .............................docx
 
EXPECTATIVAS vs PERSPECTIVA en la vida.
EXPECTATIVAS vs PERSPECTIVA  en la vida.EXPECTATIVAS vs PERSPECTIVA  en la vida.
EXPECTATIVAS vs PERSPECTIVA en la vida.
 
la unidad de s sesion edussssssssssssssscacio fisca
la unidad de s sesion edussssssssssssssscacio fiscala unidad de s sesion edussssssssssssssscacio fisca
la unidad de s sesion edussssssssssssssscacio fisca
 

Guía de ejercicios de Física Mecánica

  • 1. Guía de ejercicios F Í S I C A M E C Á N I C A
  • 2. Presentación Este guía de trabajo tiene como objetivo de ayudar a usted en el conocimiento de Mecánica, en el curso de Física Mecánica Esta guía no pretende entregar todas las herramientas para el ramo, pero si una ruta que lo ayude a entender mejor y más rápido los requerimientos del ramo. Quisiera recomendar algunas acciones que pueden ayudar en el rendimiento de este ramo: Pasen en limpio la materia vista en clases. Siempre es bueno tener ordenado la materia para poder entenderla. Rehagan los cálculos de la clase y la ayudantía. Pregunte sus dudas en la clase. No es el único que tiene dudas. Realizar un ejercicio a diario. Si tiene dificultad en resolver este ejercicio, consulte al profesor o profesor ayudante lo antes posible. Utilice los libros de la biblioteca. Existen enlaces de interés para el curso que pueden ayudar al desarrollo de este ● Física con ordenador http://www.sc.ehu.es/sbweb/fisica/ ● Cursos del MIT ● http://open.yale.edu/courses/ curso de Yale Pagina 2
  • 4. Energía Choque: Momentun e impulso Dinámica de sólidos Estática Pagina 4
  • 5. Física-Mecánica Indice Vectores......................................................................................................................................................9 Problema 1.............................................................................................................................................9 Solución............................................................................................................................................9 Pregunta 2............................................................................................................................................10 Solución:.........................................................................................................................................10 Pregunta 3............................................................................................................................................11 Solución:.........................................................................................................................................12 Cinemática................................................................................................................................................15 Pregunta 1............................................................................................................................................15 Solución..........................................................................................................................................15 Pregunta 2............................................................................................................................................17 Solución..........................................................................................................................................17 Problema 3...........................................................................................................................................19 Solución..........................................................................................................................................19 Problema 4...........................................................................................................................................20 Solución..........................................................................................................................................20 Problema 5...........................................................................................................................................22 Solución..........................................................................................................................................22 Problema 6...........................................................................................................................................24 Solución..........................................................................................................................................24 Problema 7...........................................................................................................................................25 Solución..........................................................................................................................................25 Problema 8...........................................................................................................................................27 Solución:.........................................................................................................................................27 Problema 9...........................................................................................................................................28 Solución..........................................................................................................................................28 Problema 10.........................................................................................................................................29 Solución..........................................................................................................................................29 Problema 11.........................................................................................................................................30 Solución...............................................................................................................................................30 Problema 12.........................................................................................................................................31 Solución...............................................................................................................................................31 Pregunta 13...............................................................................................................................................33 Solución..........................................................................................................................................33 Pregunta 14...............................................................................................................................................34 Solución..........................................................................................................................................34 Pregunta 15..........................................................................................................................................36 Solución..........................................................................................................................................36 Problema 16.........................................................................................................................................38 Solución..........................................................................................................................................38 Problema 17.........................................................................................................................................39 Solución..........................................................................................................................................39 Problema 18.........................................................................................................................................41 Solución..........................................................................................................................................41 Problema 19.........................................................................................................................................42 Solución..........................................................................................................................................42 Dinámica..................................................................................................................................................46 Problema 1...........................................................................................................................................46 Solución:.........................................................................................................................................46 Profesor Miguel Bustamante S Temario
  • 6. Pregunta 2............................................................................................................................................48 Solución..........................................................................................................................................48 Pregunta 3............................................................................................................................................51 Solución..........................................................................................................................................51 Problema 4...........................................................................................................................................55 Solución..........................................................................................................................................55 Problema 5...........................................................................................................................................56 Solución..........................................................................................................................................56 Problema 6...........................................................................................................................................59 Solución..........................................................................................................................................59 Problema 7...........................................................................................................................................61 Solución..........................................................................................................................................61 Problema 8...........................................................................................................................................62 Solución..........................................................................................................................................62 Problema 9...........................................................................................................................................63 Solución..........................................................................................................................................63 Problema 10.........................................................................................................................................64 Solución..........................................................................................................................................64 Problema 11.........................................................................................................................................66 Solución..........................................................................................................................................66 Problema 12.........................................................................................................................................67 Solución.....................................................................................................................................67 Problema 13.........................................................................................................................................68 Solución..........................................................................................................................................68 Problema 14.............................................................................................................................................70 Solución..........................................................................................................................................70 Pregunta 15..........................................................................................................................................73 Solución..........................................................................................................................................73 Problema 16.........................................................................................................................................75 Solución..........................................................................................................................................75 Problema 17.........................................................................................................................................77 Solución..........................................................................................................................................77 Problema 18.........................................................................................................................................79 Solución..........................................................................................................................................79 Energía.....................................................................................................................................................83 Problema 1...........................................................................................................................................83 Solución:.........................................................................................................................................83 Problema 2...........................................................................................................................................84 Solución..........................................................................................................................................84 Problema 3...........................................................................................................................................87 Solución..........................................................................................................................................87 Problema 4...........................................................................................................................................88 Problema 5...........................................................................................................................................89 Problema 6...........................................................................................................................................91 Solución..........................................................................................................................................91 Choque.....................................................................................................................................................94 Problema 1...........................................................................................................................................94 Solución..........................................................................................................................................94 Problema 2...........................................................................................................................................96 Solución..........................................................................................................................................96 Problema 3...........................................................................................................................................98 Solución..........................................................................................................................................98 Problema 4...........................................................................................................................................99 Solución..........................................................................................................................................99 Pagina 6
  • 7. Física-Mecánica Problema 5.........................................................................................................................................100 Solución........................................................................................................................................100 Problema 6.........................................................................................................................................101 Solución........................................................................................................................................101 Problema 7.........................................................................................................................................102 Solución........................................................................................................................................102 Problema 8.........................................................................................................................................103 Solución........................................................................................................................................103 Dinámica de Sólidos..............................................................................................................................105 Problema 1.............................................................................................................................................105 Solución:.......................................................................................................................................105 Pregunta 2..........................................................................................................................................107 Solución........................................................................................................................................107 Problema 3.........................................................................................................................................109 Solución........................................................................................................................................109 Problema 4.........................................................................................................................................110 Solución........................................................................................................................................110 Problema 5.........................................................................................................................................112 Solución........................................................................................................................................112 Problema 7.........................................................................................................................................114 Solución........................................................................................................................................114 Estática de Sólidos.................................................................................................................................117 Pregunta 1..........................................................................................................................................117 Solución........................................................................................................................................117 Pregunta 2..........................................................................................................................................118 Solución........................................................................................................................................118 Problema 3.........................................................................................................................................119 Solución........................................................................................................................................119 Profesor Miguel Bustamante S Temario
  • 8. Vectores Problema 1 Se tiene una barra de longitud L que se puede mover libremente en los ejes, como se muestra en la figura. La posición del punto A está descrito por la ecuación:  At= L 2  L 4 sinwt i Encuentre la ecuación que describe el ángulo (t) Grafique el ángulo (t) en función del tiempo para los valores de L=1 y w=0.2 pata t=0 y t=10. Solución Por definición, el coseno del ángulo es: Cos(f )=L/2+L/4Sin(wt))/L=1/2+1/4*sin(wt). Por tanto f =Arccos(1/2+1/4sin(wt)) El gráfico es: Pagina 8 A B L 
  • 9. Física-Mecánica Pregunta 2 Se tiene un cubo de lado “a”. Una hormiga se mueve desde el punto A al punto B. Calcular el ángulo que forman el vector OA y el vector OC. Solución: Primero debemos describir las posiciones de los puntos A y C según el sistema de referencia que se observa. El vector OA=ai+ak y OC=aj+ak. Aplicando el producto interno se tiene que: OA*OC=a2 , y los módulo de ||OA||=a2 = ||OC|| Por tanto la ecuación nos queda: a2 = (a 2 )2 Cos( ). Despejando el Coseno d el ángulo se obtiene que Cos( )=(a2 )/(2a2 )=1/2. Esto implica que  =60°. Profesor Miguel Bustamante S Temario x y z A c O
  • 10. Pregunta 3 Supongamos que tenemos el siguiente sistema planetario. El planeta 1 esta a una distancia de 2 ua, y le toma en recorrer toda la circunferencia (la órbita) 1.5 año; el satélite está a una distancia de 0.1 ua del primer planeta y gira en torno al primer planeta tomando un tiempo de 0.2 año; el planeta 2 está a 5 ua del sol y tiene un “periodo orbital” de 5.93 año. La posición de cada cuerpo está descrita por la siguiente ecuación:  Rt=Rcoswt,sinwt , donde R esta distancia al centro del movimiento, w es la frecuencia angular, que es igual w=2/T, donde T es el tiempo que demora en recorrer toda la órbita: el periodo. Todos los planetas están en t=0 según la figura. Suponiendo que cada cuerpo está descrito por la ecuación encuentre: a) El vector relativo que hay entre el planeta 1 y 2. Grafique el módulo de este vector en función del tiempo de 0 a 7 años. ¿Cuando están a la máxima distancia?, ¿cuando a la mínima? b) Escriba el vector posición para cualquier momento del satélite en torno al Sol. c) Encuentre una expresión del ángulo de forman los vectores posiciones de los planetas en función del tiempo. Grafique este ángulo entre 0 y 7 años, y diga cuando este tiene un valor de /2 y de . Pagina 10 Sol Planeta 1 Satélite Planeta 2
  • 11. Física-Mecánica Solución: Sabemos la expresión vectorial de la posición de los cuerpos en torno su punto de rotación. La posición del planeta 1 y 2 esta dado por las expresión ⃗ R1(t )=R1 cos(w1 t)^ i + R1 sin(w1 t) ^ j y  R2 t =R2 cosw2 t  iR2 sinw2 t   j . El vector relativo entre los planetas es R2-R1, y obteniendo el módulo se puede llegar a la siguiente expresión: ∥  R2t −  R1 t∥=R2 2 R1 2 −2R1 R2 cosw2−w1t  :la representación gráfica del módulo en función del tiempo es: La máxima distancia corresponde cuando los planetas están formando un ángulo de 180° ( radianes). Esto corresponde cuando cos((w2-w1)t)=-1, y es cuando (w2-w1)t=n, donde n es impar. tn=n  w1−w2 , t1=1.04 año, t3=3.0126, etc. La mínima distancia ocurre cuando cos((w2-w1)t)=1 , es de decir (w2- w1)t=m, con m par; t0=0 años, t2=2.08 años, t4=4.16 años, etc. Pregunta b Las posiciones respecto del centro de rotación está descrita por la formula presentada en el enunciado. En este caso queremos escribir el vector posición de la Luna respecto del Sol. Según la representación gráfica, el vector que buscamos es la suma de los vectores, la suma vectorial, es decir : Profesor Miguel Bustamante S Temario P1 L
  • 12.  RSL =  P1 L . En términos cartesianos, se puede escribir como:  RSL =R1cos w1 t RL coswL t iR1 sinw1 t RL sinwl t  j Pregunta c El producto interno nos da la información que buscamos:  R1  R2=∥ R1∥∥ R2∥cos t  Despejando el cos((t)) se llega a la siguiente expresión: cos(t)=cos((w1-w2)t). Los planetas están perpendiculares entre sí, cuando (w1- w2)t=k, donde k tiene los valores de la sucesión ((2n+1)/2), donde n es un número entero incluyendo el cero. Por tanto tn= 2n1 2  w2−w1 , t0=0.502 años, t1=1.5063 años, t3=7.53 años. El valor de  es cuando (w1-w2)t=, es decir t=1.0042 años Pagina 12
  • 14. Cinemática Pregunta 1 Un profesor de Física (Miguel Bustamante) pasa por al esquina de Manuel Montt y 11 de Septiembre a una velocidad de 60 Km/H en dirección Sur. En el instante que pasa frente a un alumno que tiene una moto de 1100 cc, el alumno acelera la mota para alcanzar al profesor. La moto puede acelerar de 0 a 100 Km/h en 4.5 seg. Sin embargo, la moto tiene un problema; cuando llega a los 80 Km/h, esta comienza a desacelerar a razón de -0.8 m/seg2 . El movimiento del profesor no cambia y el origen del sistema está en la esquina de Manuel Montt con 11 de Septiembre. Sobre la información entregada, grafique la posición de los cuerpos. • Calcule el momento del encuentro, si es que se encuentran. • Si no se encuentran, calcule la velocidad mínima que debe tener el profesor para ser alcanzado. Solución Primero debemos escribir la ecuación de cada móvil en todo instante de tiempo. La ecuación del profesor xp(t)=vpt=16.67t m en todo instante para t>0 seg. El alumno tiene una ecuación en un principio xa(t)=1/2at² a=(27.7-0)/4.5 m/s²=6.17 m/s², pero cambia para un t0 donde cambia de aceleración. La nueva ecuación tiene la forma 1/2a'(t-t0)²+v0(t-t0)+x0, donde v0 es la velocidad que tiene en t0 y X0 es la posición en t=t0. El momento donde cambia de aceleración es cuando el estudiante tenía una velocidad de 80 km/h=22.2 m/s; es decir, va(t)=at=22.2 m/s. El tiempo es t=3.59 seg. Este es nuestro t0. La posición es x0=1/2a*3.59²=39.9 m. Entonces la ecuación del estudiante tiene la siguiente estructura: xat = { 1 2 6.17t² ,t0 1 2 −0.8t−3.59²22.2t−3.5939.9,t3.59} Graficando la posición de ambos móviles nos da el siguiente gráfico. Del gráfico se observa que se juntan . Esto lo vamos a probar igualando las ecuaciones. Existen dos tramos de interés Pagina 14
  • 15. Física-Mecánica • t<3.59. Esto implica xa(t)=xp(t) 1/2*6.17*t²=16.67t, obteniéndose t=5.41 s >3.59 s (no es solución) No se juntan. • para t>3.59 -0.5*0.8*(t-3.59)²+22.2*(t-3.59)+39.9=16.67t, se obtiene que no hay solución real. El alumno no alcanza al profesor. Veamos la velocidad mínima del profesor. Supongamos que xp(t)=vpt. Igualando a la ecuación del alumno se obtiene la siguiente ecuación que debe resolverse: -0.4*t²+(25.072-vp)t-45.2=0 Para que se tengan soluciones el determinante debe ser mayor o igual acero. La condición mínima es que sea igual a cero. Esto implica que (25.072-vp)²- 72.32=0. Por tanto vp=16.56 m/s=59.6 Km/h, velocidad mínima. Profesor Miguel Bustamante S Temario 0 5 10 15 20 25 0 50 100 150 200 250 300 350 400 Título principal Serie1 Serie3
  • 16. Pregunta 2 Un cuerpo se mueve a rapidez constante en el tramo AC. El cuerpo parte de A hacia C, a rapidez constante. Entre A y B corresponde a la de un arco de circunferencia de radio R; entre B y C es una línea recta de pendiente cero. Con estos datos, calcule • El radio r(t), en función del tiempo y el ángulo en función del tiempo. • Calcule la velocidad angular en el punto B, como la aceleración en ese punto B en coordenadas polares. • Calcule la velocidad en C en coordenadas polares. Solución La distancia en el arco de circunferencia al origen del sistema es constante e igual a R siempre que esté entre A y B. El tiempo que toma en recorrer hasta B es t= π R 4v0 . Posterior a este punto, la distancia crece, ya que se mueve en una recta (1). la distancia s=v0(t−π R/(4v0)) . Luego la distancia al origen en el tramo BC es r (t)=√R2 +(v0(t−π R/(4v0)))2 . Esto se puede resumir como r (t)= ( R 0<t< π R 4v0 √R 2 +(v0(t−π R/(4v0))) 2 π R 4 v0 <t< π R/ 4+ R v0 ) Pagina 16 R R A B C x y
  • 17. Física-Mecánica la velocidad en el tramo AB es sólo tangencial, ⃗ V =r (t)θ̇ ̂ θ . Esto implica que la magnitud de la velocidad cumple v0=θ̇ R⇒θ̇= v0 R . la aceleración en el punto B es sólo radial, es decir ⃗ a=−R(θ̇)2 ̂ r= −v0 2 R ̂ r Recordemos que la velocidad en coordenadas polares es ⃗ v(t )=ṙ ̂ r+r θ̇ ̂ θ . Conocemos r(t), por tanto conocemos su derivada, pero no así, el ángulo. El ángulo los podemos expresar como θ(t)=π 2 + atan(v0(t−π R/(4 v0)) R ) Luego, la derivada es θ̇= 1 1+(v0 R (t−π R/(4v0))) 2 v0 R (t−π R/(4 v0)) . En el momento que pasa por C, el tiempo es tc= π R/4+ R v0 . Luego evaluando en ese tiempo las expresiones y sus derivadas, se obtiene que r (tc)=√2 R , ṙ= √2 2 v0 y θ̇= 1 2 v0 R . la velocidad en el punto C es ⃗ vc=√2 2 v0 ̂ r+ √2 2 v0 ̂ θ Profesor Miguel Bustamante S Temario Figura 1: Trayectoria, posterior a B R s R s vr vt
  • 18. Problema 3 Se tiene un cuerpo que inicialmente está a una altura H de un disco que gira. El disco a una distancia r de su centro tiene un orificio por donde puede caer el objeto. El disco gira a una frecuencia angular constante w. Calcular la relación entre la altura H y la frecuencia angular del disco para que cruce por el orificio. Ambos están inicialmente en t=0 en la posición que se muestra en la figura. Se coloca un segundo disco, separado por una distancia d, desfasado en /2 con respecto al anterior, girando a la misma frecuencia w y con un orificio a la misma distancia del centro. ¿ Desde que alturas se puede dejar caer el objeto para que cruce por ambos orificios.? Solución Para saber desde que alturas podemos dejar la partícula, primero debemos darnos un sistema de referencia. El origen del sistema de referencia va estar en el centro del disco (azul). Según el sistema de referencia, la ecuación de posición del cuerpo que cae es:  Rt=r ,− 1 2 gt²H  El tiempo que demora en caer debe ser igual a un número entero n por el periodo del disco T. En el instante que está en el origen después de un tiempo nT se cumple que -1/2g(nT)2 + H=0. Esto implica que existe n alturas posibles que crucen por el orificio, esto es: Hn= 1 2 gnT 2 . Para que cruce por el segundo orificio la ecuación en la posición de y debe cumplir la siguiente igualdad: −d= −1 2 gnT  T 4  2 H , donde T/4 es el tiempo que demora en recorrer la distancia d. Desarrollando la ecuación y imponiendo que H=1/2 g(nT)². El valor de n que se tiene es: n= 8d gT  2 − 1 32 Esto implica que la altura es: H= 1 2 gT  8d gT  2 − 1 32 ² Pagina 18 r H r d
  • 19. Física-Mecánica Problema 4 Un profesor de física sale de su casa a las 6:00 horas de la mañana a caminar junto con su perro “fotón”. El profesor camina a 5 [km/h] y el perro “fotón” lo hace a 20 [km/h]. El perro avanza en la misma dirección que el profesor. Cuando han transcurrido 45 minutos (punto A), el perro se devuelve con la misma rapidez y avanza hacia el profesor hasta alcanzarlo. Sobre la base de la información anterior, ● Calcule dónde y cuándo se produce el primer encuentro del profesor y el perro. ● Grafique, sobre un mismo diagrama, la posición de ambos en función en el tiempo. ● Grafique, para ambos, la velocidad en función del tiempo. Solución Escribamos al ecuación de posición del profesor. El tiempo cero para nosotros es a las 6:00 horas. La ecuación del profesor está dado por la expresión P t =5t Km . El perro fotón es un poco mas complicado. La ecuación del perro es: f t = 20 t km, t0,75 h −20t −0,7515km Grafiquemos las posiciones de los móviles. Según el gráfico, existe un encuentro cerca de los 6 km, es decir en torno a las 1,25 horas de viaje. El tiempo de encuentro es mayor que 0,75 horas, por lo tanto la posición del perro fotón está dado por la ecuación -20(t-0,75)+15 km. Este se debe igualar a la posición Profesor Miguel Bustamante S Temario 0 0,2 0,4 0,6 0,8 1 1,2 1,4 1,6 0 2 4 6 8 10 12 14 16 Gráifco de posición vs tiempo Serie1 Serie3 Tiempo (h) Posición (km)
  • 20. del profesor: -20(t-0,75)+15=5t. Despejando el t se obtiene el momento del encuentro. Este corresponde a t=6/5 h=1,2 h. Evaluando en cualquier de las ecuaciones de posición nos da la distancia al origen. En elñ caso del profesor P(1,2 h)=1,2*5 km= 6 km, como se aprecia en el gráfico. El gráfico de velocidad de los móviles está dado por: Pagina 20 0 0,2 0,4 0,6 0,8 1 1,2 1,4 1,6 -25 -20 -15 -10 -5 0 5 10 15 20 25 Velocidad v/s Tiempo Serie1 Serie3 Tiempo (h) Velñocidad (km/h)
  • 21. Física-Mecánica Problema 5 En el borde de un acantilado hay un cañón que dispara proyectiles a 25 [m/s]. En el instante t = 0 [s] hace un disparo a 53º de elevación; enseguida se inclina el cañón y se hace un segundo disparo, esta vez a 37º. Si el segundo disparo se efectúa T segundos después que el primero, ● ¿Cuál debe ser el valor de T para que los proyectiles choquen? ● ¿En qué punto se produce el choque? ● ¿Cuál es la velocidad a la que chocan los cuerpos? Solución Realicemos un esquema del problema Escribamos el vector posición de cada disparo, suponiendo el sistema de referencia en el cañón. El vector posición del primer disparo es:  r1 t =25cos53t ,− 1 2 g t2 25sin53∗t  m El vector posición del segundo disparo es:  r2 t =25cos37t −T  ,− 1 2 gt−T 2 25sin37t −T  Si se encuentran, las componentes de los vectores deben ser iguales. Estop implica que 25cos(53)t=25cos(37)(t-T) y Profesor Miguel Bustamante S Temario Lanzamiento 2 Lanzamiento 1
  • 22. -0,5gt2 +25sin(53)t= -0,5g(t-T)2 +25sin(37)(t-T) Resolviendo el sistema de ecuaciones de segundo orden anterior, se obtiene que t=0 s o t= 2∗25∗sin53−cos53 g s=4 s y T=1 s (g=10 m/s2 ) Evaluando en cualquier de los vectores, se obtiene que la posición de encuentro es (19,6;14,76)m. Derivando la ecuaciones de posición con respecto al tiempo, obtenemos los vectores velocidad. en el caso del proyectil 1  v1 t=25cos53,−gt 25sin53t  y del proyectil 2  v2 t=25cos37,−gt−T25sin37t −T  . Evaluando en t=0,98 s, se obtiene los siguientes vectores velocidades v1(0,98)=(15,0;-191) m/s y v2(0,98)=(19,96; 3,73) m/s. Pagina 22
  • 23. Física-Mecánica Problema 6 Una expedición de la Universidad Adolfo Ibañez quiere medir la altura de una caverna. Para eso dispara un proyectil, de modo que el ángulo de elevación es de 90º. El sonido del choque sobre el techo se escucha un tiempo T después que se ha disparado. El tiempo de subida es tv (hasta chocar con el techo) y ts es el tiempo que demora el sonido en recorrer la distancia h. La velocidad del sonido es Vs. Se sabe que T = 3tv. Calcule en términos de h, g, Vs: ● La velocidad de salida del proyectil del cañón ● El tiempo tv Solución Realicemos un esquema del problema. Si situamos el sistema de referencia en el suelo, la ecuación que describe el proyectil, la ecuación es: (0,-g/2t2 +vt). Si llamamos tv al tiempo que demora en chocar con cielo de la caverna, y ts el tiempo que el sonido demora en recorrer, se tiene la siguiente relación T=tv+ts=3tv; del cual se obtiene ts=2tv. Si la altura de la caverna es h, se tiene que: h=-g/2tv2 +vtv y h=Vsts. La altura es la misma, por tanto -g/2tv2 +vtv= Vsts = 2Vstv. Por tanto ts=2tv. Esto implica h/Vs=2tv y por tanto tv=1/2 h/Vs. Además, de la igualdad de las alturas h=-g/2tv2 +vtv =Vsts=2Vstv, se tiene que tv=2(v-Vs)/g=1/2h/Vs, del cual se obtiene que v=Vs+gh/(4Vs). Profesor Miguel Bustamante S Temario
  • 25. Física-Mecánica Problema 7 Se tiene el siguiente gráfico de velocidad en función del tiempo de dos móviles: Se sabe que el móvil A está en t =5 [s] a 10 [m] del origen. El móvil B, en t = 0 [s] está a en el origen (0 [m]). Los cuerpos continúan moviéndose indefinidamente. En base a la información entregada: a) Realice un gráfico de posición en función del tiempo para ambos móviles b) Escriba las ecuaciones de movimiento de cada uno de los móviles c) Calcule dónde se encuentran los móviles Solución Para poder realizar el gráfico debemos escribir las posiciones en función del tiempo. Para el móvil A, la ecuación sería: xAt = 10t −510 m ,0t 5 s −10t−510 m ,5t10 s −40 m,t 10 s Para el móvil B XBt = 5t m, t10 s −10t−1050 m ,t 10 s Según el gráfico, nunca se encuentran Profesor Miguel Bustamante S Temario 10 5 10[m/s] [s] Velocidad Móvil A Móvil B 5[m/s] -10[m/s]
  • 26. Pagina 26 0 2 4 6 8 10 12 14 -60 -40 -20 0 20 40 60 Posición de los móviles Serie1 Serie3 Tiempo (s) distancia m
  • 27. Física-Mecánica Problema 8 Un móvil está en t= 0 s en la posición  r =10,10m . En ese instante tiene una velocidad de  v=5,−2m/s hasta t=10 s. En ese instante cambia su velocidad a  v=−10,5m/s y se detiene en t=30 s. Otro móvil, tiene una velocidad  u=10,−5m/s y en t= 0 s está  r=0,15m . Con la información entregada: ● Dibuje las trayectorias de los móviles ● ¿Se encuentran? Solución: Debemos escribir las posiciones de los móviles La ecuación del primer cuerpo  Rt= { 5,−2t10,10m, 10t0s −10,5t−1060,−10m , 10t50s −340,190m, t50 s } La ecuación del segundo cuerpo  R2 t =10,−5t0,15m La representación gráfica en el plano XY es: Para contestar la pregunta, si se encuentran debemos resolver la ecuación  Rt=  R2 t  . Al igualar las ecuaciones, se debe cumplir tanto en las componentes del eje Y como del eje X. En este caso, no se cumple y por tanto no se encuentran. Profesor Miguel Bustamante S Temario
  • 28. Problema 9 La aceleración de una partícula es directamente proporcional al cuadrado del tiempo t. Cuando t= 2 s , v=-15 m/s y cuando t=10 s, v=0,36 m/s. Sabiendo que la partícula está dos veces mas lejos del origen en t=2 s, que en t= 10 s Determine: a) La posición de la partícula en t= 2 s y t=10 s b) la distancia recorrida por la partícula de t=2 s a t = 10 s Solución Sabemos que at =kt2 . La velocidad es, por tanto v t=∫ 2 t atdt−15 m/ s=k t3 3 − 8k 3 −15 m/ s (1 punto) Por construcción v(2)=15 m/s, pero se debe satisfacer v(10)=0,36 m/s. Esto implica que el valor de k debe satisfacer k 10 3 3 − 8 k 3 −15 m/s=0,36 m/s . el valor de k es:0.042 (1 punto). La posición de la partícula es xt=∫ 2 t vtdtD=D0.0038333333333333t 4 −15.12266666666667t30.184 (2 puntos) En x(2)=D, y en x(10)=D/2= 82.70933333 m +D. Así D=82.70933 m (1 punto) Respuesta a), x(2)=82.709333 m y x(10)=165.418666 m (1 punto) Respuesta b) La distancia recorrida es 82.716 m Pagina 28
  • 29. Física-Mecánica Problema 10 Un autobús está estacionado a la orilla de un autopista cuando a su lado pasa un camión que viaja con una velocidad de 45 mi/h. Dos minutos después, el auto bus arranca y acelera de manera uniforme hasta que alcanza una velocidad de 60 mi/h, la cual mantiene. Sabiendo que 12 minutos después de que el camión pasó junto al autobús, éste se encuentra 0,8 mi delante del camión, determine: a) Cuándo y donde el autobús rebasa al camión b) la aceleración uniforme del autobús 1 mi = 1,609 km Solución Debemos escribir la ecuaciones de posición de cada móvil. Si situamos en donde está estacionado el auto bus, como buestro origen, la ecuación del camión viene dado por la ecuación de itinerario: ct=20,1t m ( 1 punto) Veamos el bus bt= 0, t120 s a t−1202 2 , 120tt1 26,81t−t1x1, t1t donde t1 es el tiempo demora en llegar a una velocidad de 60 mi/h=26,81 m/s y x1, es la posición en t=t1. La posición del camión t=12 min=720 s es 14472 m, que esta 0,8 mi=1287,2 m, luego la posición del bus en t=720 s es 13185 m. Luego, se debe satisfacer las siguientes ecuaciones 26,81720−t1x1=13185 m a t1−120 2 2 =x1 y at1−120=26,81m/ s Resolviendo el sistema se obtiene: a=0.1238 m/s2 (respuesta b) ,t1=336.4117 s y x1=2901.0| m. Según , la posición y el tiempo el bus va a una velocidad uniforme. Así, para encontrarse, deben estar al mismo tiempo en el mismo instante, el bus y el camión. b(t)=c(t). El tiempo de intersección es t=911,802 s y evaluando en cualquier de las posiciones se obtiene donde se encientran c(911,864)=18327.22 m Profesor Miguel Bustamante S Temario
  • 30. Problema 11 En la carrera de de Usain Bolt, medidos que la velocidad media en los últimos 100 m que era de 41 kh/h (11,38 m/s). Suponiendo que la velocidad viene dado por la expresión v t=V0 1−e −t  , donde V0=11,38 m/s. Sabemos que x(19,30 s)=200, y que la velocidad final era 11,38 m/s, calcule • El valor de . (3 puntos) • La aceleración máxima.(3 puntos) Ayuda: Para x(19,30)=200 m asuma que et=19,30 −t =0.0 Solución Si integramos la velocidad, y suponiendo que parte del origen, se obtiene que Xt =V0t 1  e −t  X0 Si parte del origen, se tiene que X0= −V0  = −11,38  Pero en X(19,30)= 200 m, t utilizando el hecho que et=19,30 −t =0.0 , se obtiene que X(19,30)=11,38*19,30-11,38/=200, lo que implica que =0,58 s-1 . ( 3 puntos). Al derivar la velocidad se obtiene la aceleración at=V0  e− t . la aceleración es máxima en t=0 s. Lo que implica que amax=V0=6,59 m/s2 ( 3 puntos) Pagina 30
  • 31. Física-Mecánica Problema 12 Una moto detenida en un semáforo, acelera con aceleración constante cuando le da la luz verde. Cuando llega a los 80 km/h en t= 8 s, la moto falla, y comienza a desacelerar en forma constante de modo que recorre 150 m hasta detenerse. Sobre la base de la información entregada, calcule: • La desaceleración de la moto. • La aceleración de la moto. • La distancia total recorrida. • Realice un bosquejo de un gráfico de posición v/s tiempo, y de velocidad v/s tiempo. Solución Debemos tener claro, que este movimiento, consiste en l combinación de dos movimientos acelerados. La aceleración se puede escribir como: a(t)= {a0, t< 8s a1, t> 8s} . La velocidad de 80 km/h, corresponde a una velocidad de 22.22 m/s. Luego la aceleración a0=22.22/8 m/s2= 2.78 m/s2.(1.5 puntos); por otro lado, la velocidad desacelerando, cumple con la ecuación v( x)=√v0 2 + 2a1 d , donde v0=22.22 m/s, y d=150 m. En esa distancia v(150)=0, lo cual podemos despejar a1, dando a1= −v0 2 2d =−1.645 m/ s 2 (1.5 puntos). Como parte del reposo, la ecuación de velocidad que se obtiene es: v(t)=∫ 0 t a(t)dt={ a0 t , t< 8s a08+ a1(t−8) , t> 8 s } y la ecuación de posición x(t )=∫ 0 t v(t)dt= { a0 t2 2 , t< 8s a0 8 2 2 + a0 8(t−8)+ a1 (t−8) 2 2, t> 8 s}Calculemos el momento cuando se detiene. V(t)=0, que corresponde a t=8 (1− a0 a1 )=21.519 s , cual implica que x(21.519 s)=239.30 m (1.5 puntos) El gráfico de velocidad en función del tiempo es: Profesor Miguel Bustamante S Temario
  • 32. y la de posición en el tiempo Pagina 32
  • 33. Física-Mecánica Pregunta 13 La forma de la función de velocidad de un movimiento unidimensional en función del tiempo viene dado por la expresión v(t)=√400−(t−20) 2 m/ s . Calcule: • La velocidad máxima, y cuando se produce • La expresión de la aceleración en función del tiempo. • Si parte del origen, ¿en que posición está en t=40 s?. Solución La ecuación de velocidad se produce cuando , la derivada es cero. Es decir, la aceleración a(t)= 20−t √ 400−(20−t) 2 (2 puntos). Se observa que en t= 20 s se produce la velocidad máxima, que es de 20 m/s. (2 puntos) La posición viene dado por la expresión x(t)=∫0 t v(t)dt=((t−20)√(40t−t 2 )+ 400asin((t−20)/ 20))/2+ 100π y evaluado en t=40 da π202 =1260,63 m (2 puntos) (Corresponde al área de la figura) Profesor Miguel Bustamante S Temario
  • 34. Pregunta 14 El pato Ruperto, volaba a velocidad constante v0, horizontal al suelo a una altura H. Un manifestante, lo ve y le lanza un pequeño proyectil, de modo que el ángulo de lanzamiento en el momento que lo ve es de a (con respecto a la horizontal) orientado hacia el pato Ruperto. Sabiendo que la velocidad de salida de la mano del proyectil es V, ¿A que altura volaba el Pato suponiendo que este proyectil le da ? Solución Debemos aplicar las ecuaciones de posición de los dos cuerpos. La posición del pato ⃗ P(t)=(v0t+ H tan (α) , H ) El proyectil ⃗ r (t)=(vcos(α)t ,− 1 2 g t 2 + vsin (α)t) . ( 2 puntos, por las ecuaciones) Si le pega, debe cumplirse que v0t=v cos(α)t− H tan(α) y H = −1 2 g t 2 + vsin (α)t . ( 2 puntos, condiciones) Despejamos el tiempo t Pagina 34 a v0 V H
  • 35. Física-Mecánica t= −H /tan(α) v0−vcos(α) . Reemplazamos t en la ecuación de posición vertical y(t) y nos una ecuación para H H = −1 2 g H 2 tan 2 (α) (v0−vcos(α)) 2 −H V sin(α) tan (α) v0−vcos(α) , lo cual nos da H = 2 g v0tang 2 (α)(v cos(α)−v0) ( 2 puntos). Profesor Miguel Bustamante S Temario
  • 36. Pregunta 15 Se tiene una argolla que puede deslizar sin roce por el perímetro de un anillo de radio R. La argolla esta unido a una barra que desliza por ella sin roce. De acuerdo a la información entregada, calcule • La distancia OA en función de q(t) • La velocidad de la argolla en coordenadas polares q=0° con ˙ θθ=0= V 0 R Solución Utilizando el teorema del coseno la distancia OA puede ser calculada. OA(β)=√R 2 + R 2 −2R 2 cos(β)=R√2(1−cos(π−2θ(t))) La velocidad viene dado por al expresión ⃗ v(t )=ṙ ̂ r+ r θ̇ ̂ θ . Pagina 36 O A q O A q C R b
  • 37. Física-Mecánica En este caso ecu 1 Evaluando la expresión 1 en q=0, nos da ṙ=0 Luego, la velocidad es: ⃗ v=0 ̂ r +2R θ̇ ̂ θ=2V 0 ̂ θ Profesor Miguel Bustamante S Temario ṙ= sin(π−2θ)R √(2)√(1−cos(π−2∗θ)) (−2θ̇)
  • 38. Problema 16 Para estudiar un auto de carrera, se usa una cámara para registro de movimiento a alta velocidad se coloca en el punto A. La cámara se monta sobre un mecanismo que permite registrar el movimiento del carro conforme éste se mueve sobre la pista recta BC. Determine la velocidad del automovil en términos de b,,̇,̈ . Solución La velocidad en función de la velocidad en coordenadas polares es:  vt=ṙ  rr ̇   En este caso, la magnitud r(t), depende del ángulo , de la forma r t= b cost (2 puntos). La derivada de r es : ṙ= bsin cos 2 ̇ (2 puntos). La velocidad del automóvil es:  vt= bsin cos 2 ̇  r b cos ̇   . ( 2 puntos) Pagina 38 (t) b r
  • 39. Física-Mecánica Problema 17 Un collar que desliza por un arco circular tiene un pasador que obliga a moverse por la ranura del brazo AB. El brazo gira en el sentido anti-horario con celeridad angular (velocidad angular) de 2 rad/s. Cuando el brazo está a 30º sobre la horizontal, determinar:  Determinar la distancia r(t).  Determinar la velocidad radial y tangencial (vr,v).  Determinar la aceleración radial y tangencial. Ayuda: El origen del sistema está en el punto A. Solución Por la geometría del problema se cumple que: R2 =u2 d2 −2udcos .( 1.5 puntos) Despejamos u() de la expresión anterior. u=d cos±R2 d2 sin 2 . Cuando =0°, el valor de u=dR. La solución es el positivo (+); luego u=d cosR2 d2 sin2 . Como la velocidad angular es constante es decir ̇=w=2 rad/s . Calculemos la velocidad  v=u̇  ru̇   .  v={ d2 wcostwsintw R 2 d 2 sintw 2  −dw sintw} r{d coswtR 2 d 2 sinwt 2 }w   (1.5 puntos) Calculemos la aceleración  a=ü  r2u̇̇  −u̇  r en este caso Profesor Miguel Bustamante S Temario
  • 40. ü=−d 2 w 2 sint w 2 /R 2 d 2 sin t w 2  d2 w2 costw2 /R2 d2 sintw2  −d4 w2 cost w2 sint w2 /R2 d2 sint w2 3/2 −d w2 cost w (1.5 puntos) En el caso de w= 2 rad/s, para un ángulo de 30° (/3), el tiempo t=/6. u(/6)=200,958 mm (0.5 puntos ) u̇/6=−100.1 (0.5 puntos) y ü/6= 10950 19∗19 −150=−17.78 (0.5 puntos) Así:  v=−100.1 r401.916   mm/s,  a=−17.78 r−400.4  −401.96  r=−419.70 r−400.4   mm/s2 Pagina 40
  • 41. Física-Mecánica Problema 18 La aceleración de una partícula es directamente proporcional al cuadrado del tiempo t. Cuando t= 2 s , v=-15 m/s y cuando t=10 s, v=0,36 m/s. Sabiendo que la partícula está dos veces mas lejos del origen en t=2 s, que en t= 10 s Determine: a) La posición de la partícula en t= 2 s y t=10 s b) la distancia recorrida por la partícula de t=2 s a t = 10 s Solución Sabemos que at =kt2 . La velocidad es, por tanto v t=∫ 2 t atdt−15 m/ s=k t 3 3 − 8k 3 −15 m/ s (1 punto) Por construcción v(2)=15 m/s, pero se debe satisfacer v(10)=0,36 m/s. Esto implica que el valor de k debe satisfacer k 103 3 − 8 k 3 −15 m/s=0,36 m/s . el valor de k es:0.042 (1 punto). La posición de la particula es xt=∫ 2 t vtdtD=D0.0038333333333333t 4 −15.12266666666667t30.184 (2 puntos) En x(2)=D, y en x(10)=D/2= 82.70933333 m +D. Así D=82.70933 m (1 punto) Respuesta a), x(2)=82.709333 m y x(10)=165.418666 m (1 punto) Respuesta b) La distancia recorrida es 82.716 m Profesor Miguel Bustamante S Temario
  • 42. Problema 19 Un auto-bus está estacionado a la orilla de un autopista cuando a su lado pasa un camión que viaja con una velocidad de 45 mill/h. Dos minutos después, el auto bus arranca y acelera de manera uniforme hasta que alcanza una velocidad de 60 mi/h, la cual mantiene. Sabiendo que 12 minutos después de que el camión pasó junto al autobus, éste se encuentra 0,8 mi delante del camión, determine: a) Cuándo y donde el autobus rebasa al camión b) la aceleración uniforme del autobus 1 mi = 1,609 km Solución Debemos escribir la ecuaciones de posición de cada móvil. Si situamos en donde está estacionado el auto bus, como nuestro origen, la ecuación del camión viene dado por la ecuación de itinerario: ct=20,1t m ( 1 punto) Veamos el bus bt= 0, t120 s a t−1202 2 , 120tt1 26,81t−t1x1, t1t (2 puntos) donde t1 es el tiempo demora en llegar a una velocidad de 60 mi/h=26,81 m/s y x1, es la posición en t=t1. La posición del camión t=12 min=720 s es 14472 m, que esta 0,8 mi=1287,2 m, luego la posición del bus en t=720 s es 13185 m. Luego, se debe satisfacer las siguientes ecuaciones 26,81720−t1x1=13185 m (2 puntos) a t1−1202 2 =x1 y at1−120=26,81m/ s Resolviendo el sistema se obtiene: a=0.1238 m/s2 (respuesta b) ,t1=336.4117 s y x1=2901.0| m. Según , la posición y el tiempo el bus va a una velocidad uniforme. Así, para encontrarse, deben estar al mismo tiempo en el mismo instante, el bus y el camión. b(t)=c(t). El tiempo de intersección es t=911.802 s y evaluando en cualquier de las posiciones se obtiene donde se encuentran c(911.864)=18327.22 m. Pagina 42
  • 46. Dinámica Problema 1 Se tiene el siguiente sistema de masas: Realice una investigación para encontrar la relación entre la aceleración de la masa M y la fuerza F aplicada al cuerpo 2M. Solución: Lo primero que debemos identificar, son las fuerzas actuando en cada masa. La suma de las fuerzas libres la masa superior es : N+W+fr=Ma La suma de las fuerzas sobre la masa inferior es: F+N2+W2+fr2-fr=2Ma. Descomponiendo los vectores respecto del sistema de referencia es: Masa Superior En el eje X: -fr=-Ma En el eje Y: N-Mg=0 En la masa inferior En el eje X:-F+fr2+fr=-2Ma En el eje Y: N2-N-2Mg=0 Pagina 46 Masa M Coeficiente de roce ms=0.8, mk=0.7 Masa 2M Coeficiente de roce m s=0.9, mk=0.8 F fr N W Fuerza actuando en el cuerpo superior N2 -fr F W2 fr2 Fuerza actuando en el cuerpo inferior i j -N
  • 47. Física-Mecánica De la ecuación del eje Y, cuerpo superior se tiene N=Mg, por tanto N2=3Mg (Eje y, cuerpo inferior). Sumando las ecuaciones del eje X, se tiene que -F+fr2=-3Ma. Si F £ msN2=ms3Mg, la aceleración es cero. Si la fuerza es superior es esta, los cuerpos se mueven como uno sólo, es decir la aceleración es a=(F-mk3Mg)/(3M)=F/(3M)-3m kg. Esta ecuación válida solo cuando fr £ m s N. La aceleración crítica es cuando fr= msMg=Ma esto implica que a=msg. El valor de la fuerza es F= msMg+mk'2Mg-2Mmsg. Con este valor, la masa superior está apunto de deslizar. Una fuerza F superior a esta el cuerpo desliza y la aceleración se obtiene de la ecuación fr=mkMg.=Ma, esto implica a=mkg. En un gráfico esquemático nos queda de la forma: Profesor Miguel Bustamante S Temario a F a=0.7 g 0.9*3Mg Mg(2*0.7+0.8) g(2*0.7+0.8)
  • 48. Pregunta 2 Se tiene el siguiente conjunto de masa (m1, m2) sobre un plano inclinado sobre la horizontal. Calcule el ángulo para el cual el sistema comienza a moverse. Aumente en 10° este ángulo y calcule la aceleración de cada masa. Solución Antes de comenzar a solucionar el problema, quiero llamar la atención de algo: Fíjese que el coeficiente de roce estático de la masa m1, es mayor que el de la masa m2. El ángulo crítico de deslizamiento para la masa m1 es 38.656° y el ángulo crítico de la masa m2 es 30.963 °. Por tanto el ángulo que se busca debe estar contenido entre estos dos valores; debe ser mayor a 30.9°, pero menor que 38.65°. Veamos el diagrama del cuerpo libre: Los vectores N1 y N2 son las normales de los cuerpos m1 y m2, fr1 y fr2 las fuerzas de roce y W1 y W2 los pesos. La Fuerza P es la fuerza de contacto entre los cuerpos. Según la segunda ley de Newton, la suma de los fuerzas en el cuerpo debe ser cero. Pagina 48 a m1 m2 Coeficiente de roce: m s=0.8 y m k=0.3 Coeficiente de roce m s=0.6 y m k=0.2 a m1 m2 W1 W2 N1 N2 fr1 fr2 x y P -P
  • 49. Física-Mecánica Para el cuerpo m1, se tiene que debe cumplir N1+W1-P+fr1=0 y el cuerpo m2 N2+W2+P+fr2=0. Descomponiendo respecto al sistema de referencia, se tiene: Cuerpo m1 N1j-Pi+fr1i-M1gcos(a )j-M1gsin(a )i=0 o N1-M1gcos(a )=0 y -P+fr1-M1gsin(a )=0. Cuerpo 2 N2j+Pi+fr2-M2gsin(a )i-M2gcos(a )=0 o N2-M2gcos(a )=0 y P+fr2-M2gsin(a )=0 Sumando las ecuaciones 1.2 y 2.2 se obtiene fr1+fr2-(M1+M2)gsin(a)=0; es decir la suma de las fuerzas de roce debe ser igual fr1+fr2=(M1+M2)gsin(a ) En la situación crítica la fuerza de roce es igual al coeficiente de roce estático por la normal. En este caso, fr1=ms1N1=ms1M1gcos(a) y fr2=ms2N2= m s2M2gcos(a). Remplazando en la ecuación (3) se tiene que la ecuación para el ángulo (ms1M1+ms2M2)cos(a)=(M1+M2)gsin(a) La tangente del ángulo critico es: tang= s1 M1s2 M2 M1M2 en el caso que el ángulo se aumente en 10°, a°=a+ 10° Las ecuaciones de movimiento son: Para el cuerpo m1, se tiene que debe cumplir N1+W1-P+fr1=M1a y el cuerpo m2 N2+W2+P+fr2=M2. Descomponiendo respecto al sistema de referencia, se tiene: Profesor Miguel Bustamante S Temario 1.1 1.2 2.1 2.2 ( 3 )
  • 50. Cuerpo m1 N1j-Pi+fr1i-M1gcos(a )j-M1gsin(a )i=0 o N1-M1gcos(a )=0 y -P+fr1-M1gsin(a )=-M1a. Cuerpo 2 N2j+Pi+fr2-M2gsin(a )i-M2gcos(a )=0 o N2-M2gcos(a )=0 y P+fr2-M2gsin(a )=-M2a Pagina 50 1.1 1.2 2.1 2.2
  • 51. Física-Mecánica Pregunta 3 Se tiene el siguiente esquema de un sistema de masas y plano inclinados El sistema está en reposo, y se deja libre. Cuando el cuerpo (3) llega al suelo, no hay mas tensión en la cuerda que liga el cuerpo (2) con (3) y queda detenido. Sobre la información entregada calcule: ● La aceleración de la masa 1 en función del camino recorrido por la masa (1) ● Realice un gráfico (bosquejo) del punto anterior, ● Calcule la rapidez de la masa 2 en función del camino recorrido por la masa (1). ● Realice un gráfico (bosquejo) del punto anterior Deje expresado los resultados en función de g, L, M. Recordar que sin(30)=1/2. Solución Para responder a todos los puntos anteriores, debemos analizar todas las fuerzas actuando sobre cada masa. En el caso de la masa 1, el diagrama de cuerpo libre es: Las descomposición de los vectores, según el sistema asociado a la masa 1, nos da: Eje x: -T+Mgsin(30)=-Ma (Se supone que sube) Eje y: N-Mgcos(30)=0 Profesor Miguel Bustamante S Temario Masa M Masa M 30 Masa M g L/4 L (1) (2) (3) N Mg T
  • 52. De la masa 2 La ecuación de esta masa, según el sistema asociado: Eje y: T-T''-Mg=-Ma (Se supone que baja). De la masa 3 La ecuación de esta masa, según el sistema asociado: Eje y: T''-Mg=-Ma (Se supone que baja). Escribiendo todas las ecuaciones, se obtiene un sistema de ecuaciones, cuyas incógnitas son: a,T y T''. -T+Mgsin(30)=-Ma T-T''-Mg=-Ma T''-Mg=-Ma Despejando las incógnitas, se obtiene que: a= g 3 2−sin30 , T''=Mg(1/3+sin(30)) y T=2Mg/3(1+sin(30)). Este resultado es válido, solo cuando la masa 3 no alcance el suelo. Cuando lo alcance, el sistema cambia; la tensión T'' no existe y las ecuaciones serán: -T+Mgsin(30)=-Ma T-Mg=-Ma En este caso, la incógnita será a y T. Se obtiene que a= g 2 1−sin30 y T=Mg/2(1+sin(30)) Pagina 52 T T'' Mg T'' Mg
  • 53. Física-Mecánica Cuando la masa 2 llegue al suelo, no habrá tensión T, y por tanto la aceleración de la masa 1 va a ser igual a a=-gsin(30) (Cambia el sentido de la aceleración). Del esquema se sabe que cuando la masa 3 recorre una distancia 3/4L, la aceleración de la masa 1 cambia a a= g 2 1−sin30 y que cuando la masa 2 recorre la distancia L, la aceleración cambia a a=-gsin(30). Con esta información podemos realizar el siguiente gráfico: La masa 2 está sometida a la misma aceleración que la masa 1. La rapidez puede estar expresada por la siguiente relación: v x=v0 2 2ax , donde a es la aceleración, x es el camino recorrido y v0, es la velocidad inicial para x=0. Si los cuerpos partieron del reposo, la rapidez de la masa 2 estaría dado por la expresión: Profesor Miguel Bustamante S Temario
  • 54. v x= 2xg 2 2−sin30 , 0x3/4L 3gL 4 2−sin30 2∗g 2 1−sin30x− 3L 4  , Lx 3L 4 Para x>L la rapidez es cero. El gráfico, tendría la siguiente forma: Pagina 54 Figura 2: Velocidad en función de la posición posición 3L/4 L V(3L/4) V(L)
  • 55. Física-Mecánica Problema 4 Calcular la fuerza F que debe aplicarse sobre un bloque de masa =20 Kgr para evitar que el bloque B de masa M=2 Kgr caiga. El coeficiente de roce estático entre los bloques es de 0.5 y la superficie horizontal no presenta fricción. Solución Debemos analizar las masas A y B por separado. Según el sistema asociado a la a la masa MA, las fuerzas descompuestas quedan: Eje x: F-P=MAa Eje y: N-MAg-fr=0 La masa B. La ecuaciones según el sistema de referencia asociado es: Ejex: P=MBa Eje y: fr-MBg=0 En la condición crítica fr=P=> MBg=MBa. Despejando a, se tiene a=g/ Sumando las ecuaciones en X, se obtiene F=(MA+MB)a=(MA+MB)g/. Profesor Miguel Bustamante S Temario A B F A F MA g N -P x y MB g P fr
  • 56. Problema 5 La masa m1 sobre una superficie horizontal sin fricción se conecta a la masa m2 por medio de una polea móvil y una polea fija sin masas. a) Si a1 y a2 son magnitudes de la aceleración de m1 y m2 respectivamente, determinar la relación entre estas aceleraciones. Determine expresiones para b) las tensiones en la cuerdas y c) y las aceleraciones a1 y a2 en función de de m1, m2 y g. Solución Para resolver este problema, debemos realizar un diagrama de cuerpo libre de cada cuerpo. Masa m1 Escribamos las fuerzas descompuesta según el sistema de referencia asociado: Eje x: T1-fr=m1a1 Eje y: N-m1g=0 Para la masa m2, Pagina 56 m1 P m2 m1 m1g fr T1 x y N m2 T2 m2g x y
  • 57. Física-Mecánica Eje y: T2-m2g=-m2a2. Quiero hacer notar, que las tensiones son distintas, pro que son cuerdas distintas. Estas tensiones están relacionadas por la polea del plano horizontal P. Si la polea tiene una masa despreciable, como un momento despreciable1 , el diagrama en la polea de las fuerzas sería: Del diagrama se tiene que 2T1=T2. Ahora nos falta la aceleración: Las cuerdas tienen un largo constante (Cuerdas inextensibles). La cuerda L1 se debe cumplir la siguiente relación: L1=x1L1+X2L1, donde x1L1 y x2L2 son las distancias entre la polea y la muralla y la masa m1 respectivamente. De igual forma se debe cumplir que: L2=x1L2+x2L2. Resumiendo: L1=x1L1+X2L1 L2=x1L2+x2L2. 1 Esta suposición no es del todo correcto. Las tensiones T1 no son iguales si tomamos en cuenta el torque sobre la polea. Este tipo de problemas se vabn a ver en dinámica rotacional. Profesor Miguel Bustamante S Temario P T2 T1 T1 m1 P m2 Cuerda de largo L1 cuerda de largo L2 x1L2 x2L2 x1L1 x2L1
  • 58. Derivando dos veces las relaciones anteriores se tiene: d2 x1l1 dt2 = −d2 x2L1 dt2 y d2 x1l2 dt2 = −d2 x2L2 dt2 Estas son las aceleraciones; en un diagrama: En el caso de la cuerda L1, y por la aproximación tienen que tener el mismo valor de la aceleración a1 (de lo contrario se corta la cuerda). Vectorialemente, es equivalente a 2a1=a2 ó -2a1+a2=0. Las ecuaciones que gobiernan este sistema se reducen a: Masa m1 Eje x: T1-fr=m1a1 Eje y: N-m1g=0 Masa m2 Eje y: T2-m2g=-m2a2. Polea 2T1=T2 cuerda -2a1+a2=0 Con este sistema de ecuaciones se obtiene el siguiente resultado: a1=g(m2-2m1)/(m1+m2) a2=2g(m2-2m1)/(m1+m2) T1=m1g+m1g(m2-2m1)/(m1+m2) T2=2m1g+m1g(m2-2m1)/(m1+m2) Pagina 58 P a2 a1 a1
  • 59. Física-Mecánica Problema 6 Sobre un piso rugoso se lanza con rapidez v0 un bloque de masa M, sobre el cual reposa otro bloque de masa m. Se observa que el bloque de masa m nunca desliza respecto del bloque de masa M. El coeficiente de roce dinámico (cinético) entre el bloque de masa M y el piso es d y el coeficiente de roce estático entre el bloque de masa m y el bloque de masa M es s 1. Hacer un diagrama de cuerpo libre para cada bloque dibujando y explicando cada fuerza que aparece en los diagramas 2. Escribir las ecuaciones necesarias para calcular todas las fuerzas que aparecen en los diagramas de cuerpo libre. 3. Calcular la aceleración que experimentan los bloques. Solución El diagrama de cuerpo libre: Por el diagrama del cuerpo libre: Eje y: N-mg=0. Eje x: -fr=-ma. Analizando el segundo cuerpo por diagrama del cuerpo Profesor Miguel Bustamante S Temario M m v0 s d m mg N fr x y Mg -N fr N' fr'
  • 60. libre Por el diagrama del cuerpo libre: Eje x: fr'+fr=-Ma Eje y: -Mg+N'-N=0 Sabemos que la masa m no desliza con respecto a M. Esto implica que ambos cuerpo experimentan la misma aceleración Sumando las fuerzas en la dirección del eje x, de ambos cuerpos se obtiene fr'=-(M+m)a (*) El cuerpo inferior desliza con respecto al suelo. En este caso fr'= N'. N'=N+Mg y N=mg, por tanto N'=(M+m)g. De la ecuación (*), fr'=N'=(m+M)g=-(M+m)a. Simplificando se obtiene que a=-g. Pagina 60
  • 61. Física-Mecánica Problema 7 Una partícula es proyectada a la derecha a partir de x=0 con una velocidad inicial de 9 m/s. Si la aceleración de la partícula está definida por la relación a=−0,6v3/2 , donde a y v están expresados en m/s2 y m/s, respectivamente, determine: a) la distancia que la partícula habrá recorrido cuando su velocidad es 4 m//s b) el instante en el v= 1 m/s c) el tiempo requerido para la partícula recorra 6 m. Solución Sabemos de la relación dv dt = dv dx dx dt =−0,6 v 3/2 , lo que nos da X v/s V En v= 4 m/s, con X=8/3 m ( 2 puntos) De la definición de aceleración, tenemos a= dv dt =−0,6 v 3/2 , cual al integrar se obtiene que V v/s t Si v= 1 m/s, el tiempo t=20/9=2,22 s. (2 puntos). Utilizando la ecuación X v/s V, se obtiene que v=1.44 m/s. Esta velocidad corresponde (ecuación V v/s t) para t=10/6 s=1,666 s. (2 puntos) Profesor Miguel Bustamante S Temario −0,6x=2v−3 −2 v  2 3 =−0,6t
  • 62. Problema 8 La aceleración debida a la gravedad a un altitud “y” sobre la superficie de la Tierra se puede expresar como a= −32,2 1 y 20,9 x10 6  2 donde a está expresado en ft/s2. Usando esta expresión, calcule la altura alcanzada por un proyectil disparado verticalmente hacia arriba desde la superficie de la Tierra si su velocidad inicial es a) 1800 ft/s b) 3000 ft/s c) 36700 ft/s Solución ecu 2: Y v/s V ( 2 puntos) , de cual se obtiene que 6,7298 x10 8 arctan y 20,9 x10 9 = v 2 2 − v0 2 2 (2 puntos) Si v=0, luego la altura da: Y v=20,9 x10 6 tan  v 2 1,34 x10 9  Y(1800)=50534,4 ft. (1/3 puntos) Y(3000)=140375,245 ft ( 1/3 puntos) Y(36700)=32,9x10⁶ ft. (1/3 puntos) Pagina 62 a= dv dt = dv dy dy dt =v dv dy = −32,2 1 1 y2 20,9x10 6  2 
  • 63. Física-Mecánica Problema 9 El movimiento bidimensional de una partícula está definido por las relaciones rt =32−e−t  y t=4t2e−t  , donde r se expresa en metros, t en segundos y  en radianes. Determine la velocidad y la aceleración de la partícula en t=0; en t=, ¿a qué conclusión se puede llegar respecto a la trayectoria final de la partícula? Solución Primero, calculemos la primera y segunda derivada de r(t) y (t). ṙ t=3e −t y r̈ t=−3e −t ̇t=4−8e −t y ̈t=8e −t ( 2 puntos) Sabiendo que el vector velocidad en coordenadas polares es:  vt=ṙ  rr ̇   Evaluado,  v0=3 r−3x4  =3  r−12   y  v∞=0  r6∗4  =24   ( 2 puntos) La aceleración:  a=r̈−r ̇ r2ṙ ̇r ̈  Evaluando ⃗ a(0)=(0−3(−4)) ̂ r+ (6(−4)+ 24) ̂ θ=12 ̂ r+ 0 ̂ θ=12 ̂ r y ⃗ a(∞)=(0−6x4)̂ r+ (2x0x4+ 6x0) ̂ θ=−24 ̂ r ( 2 puntos) Profesor Miguel Bustamante S Temario
  • 64. Problema 10 Se tiene un cuerpo de masa M, que sale en forma horizontal con una velocidad V, que está a una altura h, sobre el suelo. Suponiendo que la partícula de masa M, se puede en medio viscosa viene dado por la expresión  f =−  v Si M=1 kg, g=9,81 m/s2 , h=1,5 m, con V=5 m/s. Se sabe que el cuerpo cayó a 2 m del borde. Calcule: • La constante . (2 puntos) • La velocidad con que llega a ese punto.(2 puntos) • El tiempo que demora en llegar. (2 puntos) Solución Según la segunda ley: − v−m g=m a . En coordenadas: En x: − vx=m ˙ vx En y: − vy−mg=m ˙ vy De la ecuación en X se obtiene que vx t=V0x e − m t donde V0x, es la velocidad incial en x ; y en eje Y v y t= mg  e − m t −1 . Al integrar la velocidad en X:, se obtiene la posición, idem en Y. xt=∫ 0 t vx tdt=V0x m  1−e − m t  , en el eje Y, yt= −m² g  2 e − m t − mg  t m² g  2 h Si llam,amo K=/m , y remplazando los valores de lo conocido, se obtiene que, que x debe satisfacer ecu 3: y en y, K 2 g1−e −t/K −g K th=0 . Combinan do las dos igualdades, se obtiene la siguiente relación entre el tiempo y K t=0,15K+0,4 Remplazando t en función de K, en la ecuación 3 se obtiene la ecuación ecu 4: Ecuación de recurrencia de K como se observa en el gráfico, existen dos punto de encuentro en 0, y un punto que debemos determinar numéricamente. Pagina 64 V 5K1−e −t K =2 e−0,15K²0,4 K  =1−0,4 K
  • 65. Física-Mecánica Buscando el punto de encuentro, distinto de cero, es K=1.609960311444378, por tanto =1,6099 y el tiempo t=0,641 s ( Tiempo de caída ) Conocemos la constante  y t, podemos calcular en x e y: (1,782 ;1,0200) m/s Profesor Miguel Bustamante S Temario Figura 3: H(K) y G(K): tienen dos puntos de intersección
  • 66. Problema 11 Se lanza verticalmente un cuerpo de masa M, con una velocidad inicial V0. Si parte del origen, y esta sometida a una fuerza del tipo viscoso  f =−∣ v∣2  v , calcule: • La altura máxima en función de V,M y . • El tiempo para llegar a la altura màxima Solución La ecuación dinámica es: −∣vy∣2 −mg=m ˙ vy . Despejando la velocidad, se obtiene que: v y t= m g  tanatan V0  m g  −  g m t . La posición en función del tiempo se obtiene integrando la velocidad, yt = m lncosatan LV0 gm − gLT  m  L  m ln LV0 2 gm 2L − m∗lnm 2L − ln gm 2L Cuando la velocidad es cero, alcanza la altura máxima y ocurre cuando V(t)=0, ó t= atan V0 mg  mmg  si se reemplaza en la expresión de y(t), se obtiene la altura máxima hmax= m lnV0 2 gm 2 − m lnm 2 − ln gm 2 Pagina 66 V0
  • 67. Física-Mecánica Problema 12 Se tiene dos masas de valor m unidos por una cuerda inextensible. La cuerda pasa por orificio en la mesa de modo que una de las masas cuelga; la otra gira en forma circular con radio R, libre de roce. • Calcule la velocidad angular necesaria para que la masa que cuelga no suba ni baje. Solución La tensión de la cuerda es T=mg, ya que la masa está en equilibrio. La fuerza central del movimiento es T=mg=w2 Rm. Despejando w, se obtiene que w= g R Profesor Miguel Bustamante S Temario g
  • 68. Problema 13 Se tiene un disco de radio R que gira a una velocidad angular de w. Sobre el disco hay dos masas M y 5M, unidos por una cuerda inextensible. La superficie del disco con las superficie de las masas genera un roce estático y cinético (ms, mk). El peso es perpendicular a la superficie de rotación, entrando por la hoja. La distancia de la masa 5M al centro de rotación es h. Calcule: • La frecuencia W, justo antes que comience a deslizar las masas. • La tensión de la cuerda en ese instante. Solución Veamos una vista lateral de las masas Escribamos la ecuación dinámica de la masa 5M −Fr ̂ r+ N ̂ k−5Mg ̂ k+ T ̂ k=−5M w2 h ̂ r La ecuación dinámica de M −Fr' ̂ r+ N ' ̂ k−Mg ̂ k−T ̂ k=−M w2 (h+ R/ 4)̂ r Descomponiendo por las componentes Pagina 68 5M M R h R/4 R/4 h 5mg N mg N' Fr' Fr
  • 69. Física-Mecánica 1:−Fr+ T=−5Mw2 h 2:−5Mg+ N =0 3:−Fr '−T =−Mw2 (h+ R/4) 4: N '−Mg=0. Sumando las ecuaciones 1 y 4, se obtiene Ecu 5 La condición crítica de equilibrio ocurre cuando Fr=μs N . Luego, la ecuación 5 se transforma Ecu 6 Despejando w de la expresión 6 se obtiene w= √6μs g 6h+ R/4 Profesor Miguel Bustamante S Temario −Fr−Fr '=−5Mw2 h−Mw2 (h+ R/4) μs 5Mg+ μs Mg=6Mw2 h+ Mw2 R/4
  • 70. Problema 14 Se tiene el siguiente sistema de masas. Sobre la información entregada en la figura, se pregunta: ¿Se mueven las masas? Si existe aceleración, calcule la aceleración de cada masa. Solución Debemos analizar las fuerzas de cada masa por separado La masa colgando: La masa 2M sobre la masa M Pagina 70 Coeficiente de roce estático 0.8 Coeficiente de roce cinético 0.6 Roce estático:0.3 Roce cinético 0.2 Masa M Masa 2M Masa M T Mg
  • 71. Física-Mecánica La masa M Nuestra hipótesis: “El sistema está en equilibrio”. Masa colgante: T−Mg=0 Masa 2M N−2Mg=0 −Fr+ T =0 Masa Inferior: −N+ N −Mg=0−Fr' + Fr=0 De las ecuaciones anteriores tenemos: T=Mg ⇒Fr=T=Mg y Fr '=Fr Condición de inminencia de movimiento Fr⩽μs N ⇒Mg⩽μs 2Mg ⇒1⩽1.6 (La masa 2M no desliza sobre la mas M). Esto no implica equilibrio, sólo implica que el conjunto se podría mover como todo (Masa 3M). Fr '⩽μs' (3Mg)⇒Mg 1⩽3μs ' 1⩽0.9 . El sistema de masa 2M y la masa M sobre el plano se mueve como un sólo bloque de 3M. Así, podemos plantear las nuevas ecuaciones Profesor Miguel Bustamante S Temario T N -2Mg fr -fr Fr' N' -N -Mg
  • 72. −Fr' + T=3Ma T−Mg=−Ma Luego a= g 4 (1−3μk )=0.025g=0.245 m/s 2 Pagina 72
  • 73. Física-Mecánica Pregunta 15 Se proyecta una carretera para un tráfico de 120 km/h. A lo largo de uno de sus tramos, el radio de curvatura es de 270 m. La curva está peraltada como se observa en la figura de manera que no sea necesario el rozamiento para mantener el auto en la calzada Determine : • El ángulo de peralte θ • El coeficiente de roce mínimo si la carretera no estuviera peraltada. Solución Veamos las fuerzas actuando sobre el auto, cuya ecuación dinámica ⃗ w+ ⃗ N =m ⃗ a En una descomposición en coordenadas cilíndricas, nos dan las ecuaciones Eje vertical N cos(θ)−mg=0 ( 1 punto ) Eje radial −Nsin(θ)=−m v2 R (1 punto ) Combinando las ecuaciones se obtiene que tan (θ)= v2 gR =0.419 , lo que implica q=22.74ª. (1 punto) Si está horizontal, es la fuerza de roce la responsable de mantener en órbita circular. La ecuación dinámica de este cuerpo en coordenadas cilíndricas − fr ̂ r+ N ̂ k−mg ̂ k=−m v2 R ̂ r (1 punto) Profesor Miguel Bustamante S Temario W N W fr
  • 74. de lo cual se obtiene que fr =m v2 R En el momento de inminencia de movimiento, fr=msN=msmg. (1 punto) Igualando las ecuaciones dr obtiene μs mg=m V2 R ⇒μs= V 2 gR (1 punto) Pagina 74
  • 75. Física-Mecánica Problema 16 Una esfera de masa m= 5 kg unida a una barra vertical mediante dos hilos, según se observa en la figura 4. Cuando la masa gira en torno del eje de la barra, los hilos se tensan según la figura 4. Calcular: • Las tensiones de los hilos para w= 5 rad/s • La velocidad angular w, para que la tensión de la cuerda unida a B sea T=0, pero estirada. Solución En un sistema de referencia cilíndrico, la fuerza actuando sobre la masa m son la tensión T1, la tensión T2 y el peso W (ver figura 4). Aplicando la segunda ley de Newton, se obtiene ⃗ T1+ ⃗ T2+ ⃗ W =m⃗ a Descompongamos las tensiones y el peso según el sistema de referencia cilíndrico, donde el eje Z está contenido en el eje de rotación. ⃗ T1=−T1 cos(α1) ̂ r +T1 sin(α1) ̂ k con tan(α1)= 60 60 =1⇒α1=π/4 (45°) Profesor Miguel Bustamante S Temario Figura 4: Esquema de la masa girando en torno al eje
  • 76. ⃗ T2=−T2 cos(α2) ̂ r−T2 sin(α2) ̂ k con tan(α2)= 67,5 60 =1,125 ,⇒α2=48,37° ⃗ W =−mg ̂ k y ⃗ a=−m w2 r ̂ r con r=60+10 cm =70 cm =0,7 m Cada componente debe ser igual a cada la do de la ecuación, se obtiene que k: ecu 7 r: ecu 8 Combinando las ecuaciones 7 y 8, se obtienen los siguientes resultados. T2= mw 2 r− mg tan (α1) sin(α2) tan(α2)+cos(α2) Y T1= [ mw 2 r− mg tan (α1) sin (α2) tan(α2)+cos(α2) ]sin(α2) sin(α1) + mg sin(α1) En este caso, numéricamente hablando T1=98,123 N y T2=27,27 N. En el caso T2=0⇒ mw2 r− mg tan(α1) =0⇒ w= √ g r tan (α1) =3,75rad / s Pagina 76 T1 sin(α1)−T 2sin (α2)−mg=0 −T 1cos(α1)−T 2cos(α2)=−mw2 r
  • 77. Física-Mecánica Problema 17 Se tiene un masa M que desliza sin roce sobre una superficie. Se aplica una fuerza F, de modo que una masa M, (Existe roce cionético k, y estático s en la superficie superior). Calcule el valor mínimo de la fuerza F, para que la masa m está a punto de deslizar. Solución Debemos analizar las fuerzas que actúan en cada masa La masa “m” El diagrama de cuerpo libre, y descomponiendo las fuerzas en el sistema de referencia Y: N-mg=0 X: fr=ma ( 2 puntos) Profesor Miguel Bustamante S Temario Figura 1: Problema 1 M m 40° m fr N Wm
  • 78. La masa “M” Descomponiendo las fuerzas, según el sistema de coordenadas Y: N'-N-Mg=0 X:Fcos(40)-fr=Ma (2 puntos) En la inminencia de movimiento se cumple que fr=s N , como N=mg y fr=ma, se obtiene que a=s g Aplicando este resultado en el segundo cuerpo (M) Se tiene que N=N+Mg=(m+M)g En el eje X: Fcos40−s mg=M s g , del cual se desprende que F=s g mM cos40 (2 puntos) Pagina 78 -fr N' WM N 40°
  • 79. Física-Mecánica Problema 18 Determine: ● Si se mueve las masas (2M, 3/2M) ● Si acelera, calcule la aceleración. (Las poleas puntuales y no afectan el calculo). Solución Para resolver veste problema, primero se debe analizar las fuerzas que actúan en cada masa: Masa “2M” En el eje Y: T-Mg=-Ma ( 1 punto) La masa “3/2 M” En el eje X: -T+fr=-3/2Ma En el eje Y: N-3/2Mg=0 ( 1 punto) Primero debemos saber si existe un movimiento. La hipótesis que vamos a usar es que “el sistema está en equilibrio” (1 punto) Con esta hipótesis, se tiene T-2Mg=0 -T+fr=0 (1 punto) Sumando las ecuaciones, se obtiene que fr=2Mg (1 punto) . La condición de equilibrio es frs N=s 3/2Mg (1 punto). Luego, la ecuación que se obtiene es: fr=2Mg≤s 3/2 Mg o 2≤3 , lo cumple con la hipótesis, e implica que el sistema está en equilibrio. Profesor Miguel Bustamante S Temario Coeficiente de roce s=2,0, k=0,4 Masa 2M Masa 3/2M x y
  • 80. Problema 19 En la figura, los bloques de masas M1= 8 kg y M2= 6 kg esta unidos por una cuerda ideal. Los coeficientes de roce estático y dinámico entre la masa M1 y el plano inclinado son μs=0.2 y μk=0.1. El ángulo de inclinación es de θ=30°. Con la información entregada, calcule: • La aceleración de cada masa • la tensión en la cuerda. Solución Primero debemos identificar las fuerzas actuando en cada masa, pero sin la. Escribamos las ecuaciones dinámicas de cada masa. La masa queda de la forma ecu 9 La ecuación de la masa 2 es Pagina 80 ⃗ N + ⃗ w1+⃗ T=M1 ⃗ a1 30° M1 M2 g Existe roce μs=0.2 μk=0.1 30° M1 M2 g Existe roce μs=0.2 μk=0.1 N T w1 T w2 x y y s1 s2
  • 81. Física-Mecánica ecu 10 Se hace notar que las aceleraciones vectorialmente son distintas, como son las tensiones. Como cada cuerpo tiene asociado un sistema de referencia, descomponemos las fuerzas (vectores) de acuerdo al sistema de referencia asociado a cada masa. Para la masa M1, la descomposición queda de la forma ecu 11 y en la masa M2, ⃗ T=T ^ j, ⃗ w2=−M2 g ^ j , ⃗ a2=a2 ^ j Reemplazando las descomposición de los vectores en las ecuaciones 80 y 81, y separando por direcciones , se obtienen ecu 12 y ecu 13 Como la cuerda es ideal, es por tanto inextensible; es decir, la suma de s1 y s2 es constante. ecu 14 Derivando dos veces la ecuación 14, se obtiene la siguiente relación para las aceleraciones a1+a2=0⇒a2=−a1 Combinando las ecuaciones 12, 13 y 14, resolvemos un sistema del cual se obtiene que la aceleración a1 es a1= M2 g−M1 gsin(θ) M1+ M2 Con los valores del problema, el valor de esta supuesta aceleración es (g=9.8 m/s2) 1.4 m/s2, sentido positivo. Lo que implica que la fuerza apunta hacia el lado contrario. Este cálculo no resuelve el problema. Sólo indica para donde va a puntar el roce. Debemos reescribir nuevamente las ecuaciones dinámicas y descomponer las en los sistema conocidos. Profesor Miguel Bustamante S Temario ⃗ T+ ⃗ w2=m ⃗ a2 ⃗ N=N ^ j ⃗ T=T ^ i ⃗ w1=−M1 gsin(θ)^ i−M1 gcos(θ)^ j ⃗ a1=a1 ^ i x:T−M1 gsin (θ)=M1 a1 y: N−M1 gcos(θ)=0 y:T−M2 g=M2 a2 s1+s2=L
  • 82. La ecuación dinámica de la mas M1 queda ⃗ T+ ⃗ fr+ ⃗ N + ⃗ w1=M1 ⃗ a1 . La fuerza de roce, referido al sistema de referencia asociado a M1, se puede escribir como ⃗ fr=−fr ^ i . El sistema ecuaciones 12, queda escrita como ecu 15 y la ecuación de la ecuación Sin embargo, aún no podemos calcular la aceleración, ya que no sabemos si se mueve. Recordemos que una condición de equilibrio debido a la fuerza de roce es |⃗ fr|⩽μs|⃗ N| . Para resolver esta situación, vamos a adoptar una hipótesis :”El sistema esta en equilibrio”. Esta hipótesis implica que la aceleración a1=0. Las ecuaciones nos quedan x:T−fr−M1 gsin(θ)=0 y: N−M1 gcos(θ)=0 y De la ecuación inferior se obtiene que T=M2g. Reemplazando en la ecuación de la masa M1, en la dirección de x, se obtiene que fr=(M2−M1sin (θ)) g . Este es valor que debe tener el roce para esté en equilibrio. Sin embargo debe satisfacer además, |⃗ fr|⩽μs|⃗ N| . ecu 17 Veamos si la relación 17 es verdadera con los valores de este problema Hemos llegado a una contradicción. Por tanto, las masas se mueven. Como se mueven, sabemos que la fuerza de roce viene descrita por fr=μk N . Reemplazando esta relación en las ecuaciones 15 y 16, y recordando que a2=-a1 se obtiene que la aceleración a1 es Pagina 82 x:T−fr−M1 gsin(θ)=M1 a1 y: N−M1 gcos(θ)=0 fr=(M2−M1sin (θ)) g⩽μs|N|=μs M1 gcos(θ) 30° M1 M2 g Existe roce μs=0.2 μk=0.1 N T w1 T w2 x y y s1 s2 fr ecu 16 y:T−M2 g=M2 a2 y:T−M2 g=0 fr=(M2−M1sin(θ)) g=19,6⩽μs|N|=μs M1 gcos(θ)=0,2x 67,89=13,579
  • 83. Física-Mecánica a1= (M2−M1(sin(θ)+μk cos(θ)))g M1+M2 =0,92 m/ s 2 y la tensión T es T=M2 g−M2 (M2−M1(sin(θ)+μk cos(θ))) g M1+ M2 =53,28N Profesor Miguel Bustamante S Temario
  • 86. Energía Problema 1 A un cuerpo de masa m=1 Kgr se le aplica una fuerza F, de modo que éste se mantiene comprimiendo al resorte en 0.3 m. En t=0 deja de actuar la fuerza F y el cuerpo comienza a recorrer la pista ABCDEF , pasando por los puntos B y E con una rapidez de 6 m/s. Si en los tramos AB y DE son tramos rugosos y la constante de elasticidad del resorte es k=1000 N/m, calcule a) Trabajo realizado por las fuerzas conservativas en el tramo AD b) Trabajo realizado por el roce en los tramos rugosos c) Sobre la base de los datos entregados, calcule los coeficiente de roce de los tramos AB y DE. Solución: a)La fuerza conservativa en el tramo AD es el peso, el trabajo es mgh=1*9.8*1.5 J= 14.7 J b) El trabajo de la fuerza de roce es: W=Uf-Ui+Ekf-Eki. En el tramo AB el trabajo de la fuera de roce es: W=14.7-1/2*1000*(03)²+1/2*1*6²-0 J=-12.3 J=- m kMgcos(23) 1.5/sin(23) J El trabajo de DE, W=Uf-Ui+Ekf-Eki=-Mg1.5=-45 J=-m kMgCos(50) 1.5 /Sin(50) c). Los coeficientes de rocen son: m k(AB)=0.499 y m k(DE)=3.6 Pagina 86 1.5 m B C 1.7 m D E 37° A Fuerza F g 1.5 m 23° 50 ° F
  • 87. Física-Mecánica Problema 2 Se tiene una pelotita de masa m que este en el punto B, en reposo. Un alumno golpea la bolita dando una velocidad v=30 m/s y sube por un arco circunferencial de diámetro 60 m. Sobre la información entregada: ·       ¿A qué altura se desprende la bolita del arco de semicircunferencia? ·       ¿Dónde cae la bolita? (Recuerde lanzamiento de proyectiles) Solución La solución de este problema involucra tanto la energía, como las fuerzas actando sobre el cuerpo. Si la persona le dio energía inicial, esta al subir por la rampa, se transforma en potencial y cinética. Ei=1/2mv2 =Ed=1/2mvd 2 +mghd (+) donde vd es la velocidad en el momento que se desprende y hd es la altura ala cual se desprende. Realicemos un diagrama de cuerpo libre, justo en el momento que se desprende: Profesor Miguel Bustamante S Temario 2R=60 m V 2R=60 m V  hd La altura hd se relaciona con el radio R y el ángulo  por la ecuación hd =R(1+sin())
  • 88. La aceleración radial está en la misma dirección que la norma. En esta dirección tenemos: N+mgsin()=-mar Cuando se desprende el objeto, la normal es cero (N=0). La ecuación anterior queda mgsin()=mar En este punto, podemos decir que la aceleración radial ar = Vd 2 /R. Con esta relación tenemos que mgsin()=mVd 2 /R. (*) De la expresión (*) despejamos la velocidad y la remplazamos en la expresión (+). 1/2mv2 =1/2mRgsin()+mgR(1+sin()) Despejando sin(), se tiene la siguiente igualdad: sin= 1 3  v 2 gR −2=0,35 Lo que implica que =20,487º. La altura hd=1,35R. La magnitud de la velocidad a esa altura es: Vd=0,35Rg . Desde este momento, el cuerpo se comporta como un proyectil. Pagina 88 mg N  2R=60 m V  hd La altura hd se relaciona con el radio R y el ángulo  por la ecuación hd =R(1+sin()) V Rcos() O
  • 89. Física-Mecánica La posición inicial del proyectil es (-Rcos(),5R/4). La velocidad  Vd = Rg 4 sin ,cos= Rg 4  1 4, 15 4  . La posición en el tiempo del proyectil, tomando como origen el punto O, que descrito por  r t = 1 4 RG 4 t−R 15 4 ,− 1 2 g t2  Rg 4 15 4 t  5R 4  El saber donde cayó es equivalente a hacer r(t)=(x(t),0). El tiempo que demora en llegar al suelo una ves que se ha desprendido: t= 57 Rg15Rg 8g = Rg 8g 5 715=2,13 R g Luego la distancia a la que cae respecto del origen en donde cae es:  r t =−15 R 4 5715 R 64 ,0=−0.701R,0 Profesor Miguel Bustamante S Temario
  • 90. Problema 3 En el peligroso deporte del salto Bungee, un osado estudiante de Física salta desde un puente con una cuerda elástica atado a sus tobillos especial sujeta a sus tobillos. La longitud de la cuerda sin alargamiento es de 5H/7. El estudiante pesa Mg y el puente está a H por encima de la superficie de un rio “seco”. Calcule la constante elástica de la cuerda, suponiendo que es un resorte de modo que el estudiante se salve, no “mojándose”. Solución Realicemos un esquema del problema Si el origen esta en el borde del puente, la energía inicial es cero Ei=0. Cuando llega a máximo alargamiento la velocidad es cero (Energía cinética), y la energía potencial U=-mgH y la potencial elástica 1/2k(H-5H/7)2 . La energía total en este punto es 0=-mgH+1/2K(2H/7)2 . Despejando K nos da K=49mg/(2H). Pagina 90
  • 91. Física-Mecánica Problema 4 Los bloques representados en la figura, de igual masa, están unidos a los extremos de un resorte ideal de constante k. Los bloques que partieron juntos del reposo desde la posición en que x=0, se deslizan sin roce por los planos que tienen el mismo ángulo de inclinación. Si el resorte está sin deformar en la posición en que x=0, y parte del reposo determine: La posición de los bloques cuando el resorte alcanza la máxima compresión Sugerencia: Tome la altura h=0 m en el borde superior. Solución Situemos nuestro origen del sistema en la parte superior del artefacto. En este caso la energía total inicial es cero (Se supone que en x=0 el resorte no está comprimido). Cuando baja y llega a su máxima compresión, la velocidad es cero, es decir su energía cinética es cero. Sólo quedan energía potencial y cinética. En ese instante se cumple : 0=-2mgxcos()+1/2K(2xsin())2 . Despejando x de esta ecuación nos da: x= mgcos  ksin2 Profesor Miguel Bustamante S Temario x x α 
  • 92. Problema 5 Se tiene un resorte en posición vertical, con una constante elástica k=Mg/h y en presencia de gravedad está unido a una masa M. Se comprime el resorte una distancia 3h, y se suelta. Calcule: • La elongación máxima del resorte (La masa no se desprende: unido al resorte) • La velocidad que tiene la masa M, cuando pasa por el punto de elongación cero del resorte. Solución La energía inicial, es solo potencial es: ecu 18: . Cuando alcanza la elongación máxima, la energía potencial es: ecu 19: Igualando la ecuación 18 con 19, podemos encontrar x, que tiene dos soluciones: x+:(3hK-2gm)/K x-: -3h. La solución X- corresponde a la situación inicial. La solución es x+ Cuando, el resorte está en elongación x=0, sólo hay energía cinética. Pagina 92 Figura 5: Resorte comprimido, en cero elongación, elongación máxima Masa Masa 3h Masa U=−mg3h 1 2 K 3h 2 Uf= 1 2 K x 2 mgx.
  • 93. Física-Mecánica La velocidad viene dado por la expresión: v= 9h2 K m −6hg . Profesor Miguel Bustamante S Temario ecu 20: U=−mg3h 1 2 K 3h 2 = 1 2 m v 2
  • 94. Problema 6 Una masa M, esta a una altura H sobre el suelo y se suelta (parte del reposo). Sabiendo que K=2Mg/H, calcular: • Cuanto se comprime el resorte. (Si este llega a comprimir) • Donde se detiene. Solución La energía inicial, con el resorte en su elongación natural es Ui=MgH. Sólo en la zona de roce disipa la energía, existe trabajo; además comprime el resorte una distancia x. Suponiendo que se comprime el resorte, el trabajo será W =−μk Mg(H + x) ; la energía potencial del resorte será U R= 1 2 K(H −x) 2 . Luego, aplicando que el trabajo es iguala al cambio de las energía potencial y cinética y además en el punto x se detiene el cuerpo, se obtiene la siguiente igualdad ecu 21 , donde nuestra incógnita es x. Ordenando la ecuación 21 se del cual x da dos valores: x=H/2 y x=-H. En este caso, la solución es la positiva. LA fuerza que experimenta el resorte al comprimirse H/2, es ⃗ F=−K( H−H /2) ̂ i= −2Mg H H 2 ̂ i=−Mg ̂ i . Pagina 94 H M K 2 H H Coeficiente de roce estático 0.8 Coeficiente de roce cinético 0.5 g ecu 22 μk Mg( H+ x)+ 1 2 K(H + x) 2 −MgH=0 W =−μk Mg(H + x)= 1 2 K( H+ x) 2 −MgH
  • 95. Física-Mecánica La fuerza máxima que el roce podría realizar para detenerlo es Froce=μs Mg=0.8 Mg< Mg . Esto implica que se mueve. Aplicando, a partir de este punto, que el trabajo es igual al cambio de energía, se obtiene la siguiente ecuación Wroce=−μk Mgd=0− 1 2 K ( H /2) 2 = −MgH 4 que implica que d= H/2. Profesor Miguel Bustamante S Temario
  • 96. Chouqe: Impulso y conservación del momento Pagina 96
  • 97. Física-Mecánica Choque Problema 1 Dos bloques de masas m1=2.0 Kgr. Y m2 =4.0 Kgr. Se sueltan desde de una altura de 5.0 m sobre la pista sin fricción, como la que se muestra en la figura. Los bloques sufren un choque frontal elástico. • Determine las dos velocidades justo antes del choque • Determine las velocidades después del choque. • Determine la altura máxima a la cual sube cada bloque después del choque. (El choque se produce cuando la altura es H=0 m) Solución En todo el sistema no hay pérdida de energía, se conserva. Si las masas m1 y m2 están a la misma altura y parten del reposo, llegando a nivel de altura cero, lo harán con la misma velocidad. En este caso v=(2hg)1/2 =10 m/s. En el caso de este choque, se conserva el momentun y la energía. La ecuación del momentun antes del choque es:  pantes=m1 v  i−m2 v  i El momentun después del choque  Pdespues=m1 v1  im2 v2  i En este caso, el momentun se conserva, lo que implica m1 v−m2 v=m1 v1m2 v2 La conservación de la energía, nos da la ecuación: Einicial = 1 2 m1 v2  1 2 m2 v2 =Edespues= 1 2 m1 v1 2  1 2 m2 v2 2 Con esta última igualdad tenemos dos sistema de y dos incógnitas. Resolviendo el sistema nos da las siguientes soluciones v1=v=10 m/s y v1=(m1-3m2)v/(m1+m2)=-18 m/s, v2=-v=-10 m/s y v2=(3m1-m2)v/(m1+m2)=3.33 m/s Cada masa posteriormente sale en sentidos contrarios. Profesor Miguel Bustamante S Temario M1=2.0Kgr M2=4.0Kgr H H
  • 98. Como no hay roce, la energía cinética de cada masa se transforma en energía potencial. La altura será h=v2 /(2g). en el caso de la masa m1, la altura es h1=16,2 m y h2=0,55 m (g=10 m/s2 ). Problema 2 Magila el gorila de masa M, come unas bananas con insecticida. Alucina y ataca a un guardia, sobre el cual se abalanza con velocidad v (constante). El guardia, con el propósito de detenerlo, reacciona disparándole con una metralleta de proyectiles de goma de masa m = M/50.000 y cuya velocidad es u = 100v. Suponga que una vez que Magila disminuye su velocidad como consecuencia del impacto de un proyectil la mantiene constante hasta el siguiente impacto. Calcule: a. ¿Cuántos proyectiles debe disparar el guardia para detener al gorila? Suponga que las balas chocan y caen cuando alcanzan al gorila. b. Si la cadencia de tiro es T, ¿cuánto avanza Magila entre el tercer y cuarto impacto? Solución Un esquema de esta situación será: Justo antes del choque , el momentun inicial es: pi=Mv −mu Después del primer choque pf1=Mv1 . Igualando las dos cantidades, se tiene que v1=v− m M u=v− 1 50000 100v=v1− 1 500  . Para el segundo disparo, el momentun inicial antes del segundo impacto es: pi2=Mv1−mu El momentun final después del segundo choque pf2=Mv2. Igualando, se obtiene que v2=v(1- 2/500). Pagina 98 No hay roce V u
  • 99. Física-Mecánica En el tercer impacto, el momento inicial es pi3=Mv2-mu, el momento final después del tercer impacto pf3=Mv3, igualando y despejando, se tiene que la velocidad de Magila es v3=v(1- 3/500). En forma sucesiva se obtiene que la velocidad enésima es: vn=v(1-n/500); por tanto con 500 balas podemos detener a Maguila. Si la cadencia1 de Tiro es T, que la cantidad de tiros que salen por unidad de tiempo. Por tanto el tiempo entre tiro y tiro es 1/T. La velocidad después del tercer impacto es: v3=497/500u. La distancia que recorre hasta el cuarto impacto es d= 497 500 u 1 T 1 Cadencia: Repetición de un fenómeno que sucede regularmente. Profesor Miguel Bustamante S Temario
  • 100. Problema 3 Dos bloques A y B de igual masa M, están unidos mediante un resorte de constante de dureza k y largo natural L0. El sistema formado por los dos bloques descansa sobre un plano horizontal liso. El resorte está en su largo natural. De pronto una bala de masa m y rapidez v0 se incrusta en el bloque A (choque plástico e instantáneo), tal como muestra la figura. a) Calcular la rapidez v1 que adquiere el bloque A inmediatamente después que la bala se ha incrustado en él b) Cuando el resorte está comprimido el máximo los bloques viajan con la misma rapidez v2. Calcular v2 c) Calcular la distancia a entre los bloques cuando el resorte se encuentra comprimido al máximo Solución En este choque hay conservación del momentun, mas no de la energía. El mometun inicial es  pi=m v0  i . El momentun posterior al choque, instantáneo es:  pf =Mmv1  i . Como hay conservación, podemos obtener la velocidad v1,  v1= m Mm v0  i Posteriormente, se produce un segundo choque pero es elástico; el momemtun se conserva y la energía. El momentun justo antes del choque elástico es  pf =Mmv1  i . Cuando alcanza la máxima compresión el resorte, ambos cuerpos se mueven a la misma velocidad, por tanto el momentun en esa situación es  p=2MmV2  i . De la conservación de momentun pi=pf, se tiene que V2 es igual a  V2= Mm 2Mm V1 2  i= m 2Mm V0  i La energía inicial después del primer choque es cinética e igual a E1= 1 2 Mmv1 2 . Al comprimirse el resorte, la energía en ese momento es cinética y potencial elástica. E= 1 2 KL0 −a2 1/22MmV2 2 . Remplazando el valor de v2 en función de V0 y V1 en función de V0, el valor de a da: a=L0− mV0 K  M Mm2Mm Pagina 100
  • 101. Física-Mecánica Problema 4 -Se tiene una barra de largo L, de ancho H y profundidad S. Esta barra tiene un orificio circular de radio R (R<H/2 y R<L/2), coincidiendo el origen con el eje de simetría vertical y cuyo centro está a una distancia d del centro sin orificio de la barra.. Calcule el centro de masa de sistema Solución Este problema se puede pensar que es un rectángulo de la do L y H, que se ha extraído una circunferencia. La circunferencia debe tener la misma densidad del objeto real. Si la masa de objeto inicial es M, la densidad es = M LH− R 2 El centro de masa, de acuerdo a un sistema en el punto O es:  rcm= H/2 LH  i−H/2  R 2  L 2  LH  j−L/2d  R 2 LH− R 2  = L 2 − d R2 LH− R 2   j Profesor Miguel Bustamante S Temario - = H L R O d
  • 102. Problema 5 Utilizando la misma técnica, calcule el centro de masa de: Solución En este caso, la densidad es: = M LH−  2  H 2  2 En esta caso, el centro de masa va estar contenido en la linea de simetría. Sólo calculemos en el eje vertical, respecto de la esquina inferior izquierda. ycm= L/2 LH−L− 4 3 H 2   2  H 2  2  LH−  2  H 2  2  Pagina 102 L H
  • 103. Física-Mecánica Problema 6 Una bala de 300 gr con una velocidad de 330 m/s impacta con un péndulo de masa 100 kg, que cuelga desde un techo que está a 2 m. Si la bala queda incrustada en el péndulo, calcule: • La velocidad justo después del impacto. • Calcule la la altura que llega el péndulo. Solución Sabemos que el momento inicial ses  p=mv  i . Justo después del choque el momento  Pf =mMVf  i . Por conservación del momento, en el eje x, se obtiene Vf = m mM v . Una vez, efectuado el choque, la energía se conserva. La energía cinética se transforma en energí potencial 1 2 mM  M mM  2 v2 =mMgH . Luego, despejando H= m2 2gmM2 v 2 (3 puntos) Profesor Miguel Bustamante S Temario